Examen de Diagnostic 16736

Share Embed Donate


Short Description

Download Examen de Diagnostic 16736...

Description

Universidad La Salle. Facultad Mexicana de Medicina. Curso de Extensión Universitaria para la Preparación del Examen Nacional para Aspirantes a Residencias Médicas.

Nombre: Examen Diagnóstico. Número de intentos: 3. Vigencia: 6 de Febrero del 2013. Horario: 9:00 a.m. a 9:00 p.m. Programar aleatorio. (5 bloques con 20 preguntas cada uno). 1.- Femenino de 45 años es atendida en consulta consulta externa por presentar desde hace 6 meses flujo transvaginal, mucosanguinolento y sinusorragia importante, refiere dispareunia de 3 meses de evolución. AGO. G-5 P-5. Oclusión tubárica bilateral hace 6 años. E.F.: TA 130/80 MMHG, FC 80 LPM, TEMP. 36.7 ºC. Laboratorio: HB 9.7 G/DL, HTO 37 %, Se reporta Papanicolaou clase V. El siguiente paso para confirmar el diagnóstico es: a) b) c) d)

Histeroscopía. Ecosonografía. Biopsia dirigida. Colposcopía.

El examen citológico de papanicolau, realizado como examen de tamizaje en ginecología, permite establecer las características de las células que se descaman del cuello uterino. La técnica de papanicolau ha permitido disminuir la mortalidad e incidencia de cáncer invasor de cuello uterino. Se define examen de papanicolau anormal cuando se presentan alteraciones en el núcleo, citoplasma y/o la relación núcleo/citoplasma de las células examinadas. El papanicolau anormal, de acuerdo a la clasificación de bethesda abarca desde un diagnóstico citológico de ascus (siglas en inglés de células atípicas de significado no determinado), lesión escamosa intraepitelial (lei) de bajo grado (corresponde a displasia leve y cambios por papiloma virus), lesión escamosa intraepitelial (lei) de alto grado (displasia moderada, displasia severa o carcinoma in situ ) y células de cáncer invasor.

4.6. Colposcopía Ideada por Hinselmann en Alemania en 1924, llegó a América por el cono sur. Consiste en la visualización y amplificación del cuello uterino mediante un sistema binocular de lentes, entre 25 y 40 aumentos, lo cual permite la observación de las estructuras del cuello uterino mediante la asociación con imágenes preestablecidas. La colposcopía tiene una mayor sensibilidad que la citología, pero su menor especificidad. De conducir a procedimientos diagnósticos invasivos (biopsias y conizaciones) innecesarios y su mayor costo, son sus principales limitaciones. Combinadas la citología y la colposcopia brindan una seguridad diagnóstica que excede EL 95%. 4.6.1. Indicaciones de la colposcopia La colposcopía está indicada en las siguientes circunstancias (20), (21): - Pacientes con citología cervical (papanicolaou) clase III, IV o V, o sus equivalentes en los otros sistemas de clasificación. - Pacientes con citologìa clase II con atipia inflamatoria, escamosa o endocervical, o cuando se informe la presencia de coilocitos. - Pacientes con cérvix macroscópicamente normal, pero quienes presentan Sinusorragia. - Pacientes con cérvix macroscópicamente anormal, en ausencia de carcinoma evidente. - Pacientes con citología clase II persistente, pese a tratamiento de posibles causas. (Ejemplo: trichomonas). Tamizaje en cáncer ginecológico autores de la guía. Dr. Miguel Bueno Montaño Profesor asociado departamento de ginecología universidad libre ginecólogo centro médico Imbanaco profesor titular de ginecología y obstetricia universidad del valle Dr. Jaime Rubiano universidad del valle Dra. Derry Trujillo.

2.- Mujer de 25 años de edad, refiere que desde hace un par par de meses ha presentado hemorragia irregular o postcoital, actualmente actualmente presenta disuria y dolor abdominal abdominal usted debe sospechar en cervicitis por: a) Micoplasma b) Chlamydia c) Gardnerella d) Candida

4.6. Colposcopía Ideada por Hinselmann en Alemania en 1924, llegó a América por el cono sur. Consiste en la visualización y amplificación del cuello uterino mediante un sistema binocular de lentes, entre 25 y 40 aumentos, lo cual permite la observación de las estructuras del cuello uterino mediante la asociación con imágenes preestablecidas. La colposcopía tiene una mayor sensibilidad que la citología, pero su menor especificidad. De conducir a procedimientos diagnósticos invasivos (biopsias y conizaciones) innecesarios y su mayor costo, son sus principales limitaciones. Combinadas la citología y la colposcopia brindan una seguridad diagnóstica que excede EL 95%. 4.6.1. Indicaciones de la colposcopia La colposcopía está indicada en las siguientes circunstancias (20), (21): - Pacientes con citología cervical (papanicolaou) clase III, IV o V, o sus equivalentes en los otros sistemas de clasificación. - Pacientes con citologìa clase II con atipia inflamatoria, escamosa o endocervical, o cuando se informe la presencia de coilocitos. - Pacientes con cérvix macroscópicamente normal, pero quienes presentan Sinusorragia. - Pacientes con cérvix macroscópicamente anormal, en ausencia de carcinoma evidente. - Pacientes con citología clase II persistente, pese a tratamiento de posibles causas. (Ejemplo: trichomonas). Tamizaje en cáncer ginecológico autores de la guía. Dr. Miguel Bueno Montaño Profesor asociado departamento de ginecología universidad libre ginecólogo centro médico Imbanaco profesor titular de ginecología y obstetricia universidad del valle Dr. Jaime Rubiano universidad del valle Dra. Derry Trujillo.

2.- Mujer de 25 años de edad, refiere que desde hace un par par de meses ha presentado hemorragia irregular o postcoital, actualmente actualmente presenta disuria y dolor abdominal abdominal usted debe sospechar en cervicitis por: a) Micoplasma b) Chlamydia c) Gardnerella d) Candida

Según los datos de la Organización Mundial de la Salud, anualmente se detectan 89 000 000 de nuevas infecciones por Chlamydia trachomatis  en el mundo. Esta infección provoca uretritis y cervicitis, y las secuelas incluyen enfermedad inflamatoria pélvica, embarazo ectópico, infertilidad por daño tubárico, epididimitis, proctitis y artritis reactiva. Se considera principalmente un problema de salud en la mujer, en ella las manifestaciones y consecuencias son más dañinas para la salud reproductiva. Los individuos infectados con Chlamydia trachomatis  pueden portar el microorganismo por meses o años y transmitir la enfermedad a sus parejas sexuales. Su diagnóstico sigue siendo un reto, ya que quienes la padecen presentan síntomas muy leves o son portadores por tadores asintomáticos. Manifestaciones clínicas La cervicitis es la manifestación clínica más frecuente de la infección por C. trachomatis en la mujer. Sin embargo, el 70% de las mujeres infectadas no tienen síntomas, mientras que en el tercio restante las evidencias clínicas son poco específicas de infección, como flujo genital, dolor abdominal o pelviano, sangrado y/o disuria. disur ia. La presencia de disuria puede indicar una uretritis acompañante, lo que sucede en el 35% de los casos. En otras oportunidades, solo la uretra está comprometida, y la infección uretral se manifiesta como piuria o disuria con cultivo negativo (23% de los casos). El diagnóstico se realiza al examinar el hisopado endocervical, que muestra flujo amarillento o verdoso con más de 10 PMN por campo de inmersión en el examen de Gram. Este resultado define la cervicitis mucopurulenta (CMP) la cual también puede ser producida en casos de infección por gonococo o mixta (C. trachomatis y gonococo). Por lo tanto, el diagnóstico debe confirmarse mediante estudios de mayor especificidad, como las técnicas moleculares (test de ligasa, PCR), que tienen una sensibilidad del 96% aproximadamente, o la detección del antígeno por técnica de ELISA, con una sensibilidad del 75%. También se ha demostrado que las técnicas moleculares en el primer chorro de orina son específicas y altamente sensibles. Cuatro de cada diez mujeres con cervicitis no tratada adquieren enfermedad inflamatoria pelviana (EPI), con mayor riesgo de sufrir embarazo ectópico, infertilidad y dolor crónico pelviano. El riesgo de infertilidad se eleva según el número e intensidad de los episodios: alrededor del 10% después de un episodio, del 30% después de dos, y mayor del 50% si ha habido tres o más episodios. Por otro lado, el embarazo ectópico es cinco a siete veces más frecuente cuando se trata de pacientes con antecedentes de EPI.

Tratamiento Azitromicina 1 g VO dosis única Doxiciclina 100 mg VO cada 12 horas por 7 días Eritromicina 500 mg VO cada 6 horas por 7 días Ofloxacina 300 mg VO cada 12 horas por 7días Levofloxacina 500 mg VO cada 24 horas por 7 días Basado en Guías Clínicas para el manejo de las ITS , OMS, 2003 / Norma Oficial Mexicana  NOM -039-SSA2-2002 Berek J. (2002) Ginecología de NOVAK. México. Ed. Mc Graw Hill Interamericana. Pag 293.

3.- Masculino de 10 años acude a su pediatra con fiebre e inflamación dolorosa de la rodilla izquierda. Hace 2 semanas tenía la garganta irritada, y un cultivo fue positivo para estreptococo hemolítico. El niño se trató desde entonces con un ciclo de 3 días de penicilina oral. ¿Cuál de las siguientes ofrece la mejor evidencia, por si sola, del diagnóstico de fiebre reumática aguda? a) b) c) d)

Velocidad de sedimentación eritrocitaria elevada. Soplo sistólico apical. Alta valoración de anticuerpos estreptocócicos. Intervalo PR prolongado visible en el electrocardiograma.

La carditis es el síntoma diferenciador de fiebre reumática, y un soplo sistólico apical, es el hallazgo inicial más común. Los demás casos enumerados en la lista son propios de pacientes con fiebre reumática, pero también se encuentran en otros trastornos. Carditis Poliartritis Corea (Sydenham) Nódulos subcutáneos Eritema marginado Clínicas Artralgias Fiebre Antecedentes de brote reumático Laboratorio y Gabinete Elevación de reactantes de fase aguda

Prolongación del intervalo PR Evidencia de infección Estreptocócica (Grupo A): Antiestreptolisinas Exudado faríngeo PAC. Libro 1, Parte AFiebre ReumáticaPag. 3,7 Dr. José F. Guadalajara Boo • • •

Jefe de Servicio Clínico. Instituto Nacional de Cardiología Ignacio Chávez. Miembro numerario de la Academia Nacional de Medicina. Gobernador por México del american College of Cardiology.

Presidente de la Sociedad Mexicana de Cardiología. 4.- Se trata de neonato prematuro quien cursa con parto difícil, episodios de arritmia y sospecha de hipoxia-isquemia. Después del nacimiento, se encuentra letárgico y tiene periodos de apnea. Se sospecha hemorragia intracraneana. No se observa un traumatismo craneal obvio. El USG craneal identifica sangre en los ventrículos. La estructura más probable que da origen a ésta hemorragia es: a) b) c) d)

Corteza cerebral Matriz germinal Tálamo Vasos del círculo de Willis

Debe diferenciarse entre la hemorragia del neonato pretérmino y la del de término. • La del recién nacido pretérmino es la más frecuente y se produce por sangrado de la matriz germinal subependimaria. • La del neonato de término es menos frecuente y se produce por traumatismos,

malformaciones vasculares, infecciones, tumores u otras causas. Matriz germinal

• Es un tejido subependimario adyacente a los ventrículo s

laterales, altamente vascularizado, cuyos vasos muy delicados sangran fácilmente ante distintos tipos de estrés perinatal. • Es una estructura transitoria del cerebro fetal que involuciona normalmente hasta

desaparecer hacia las 32-34 semanas de gesta. Previo a su desaparición sólo persiste en el surco caudotalámico.

• Se denomina "matriz germinal" porque es el tejido que da origen a las neuronas, que

posteriormente llegan a la corteza cerebral por el proceso llamado migración. Clasificación de H. de la matriz germinal

• Grado I: Hemorragia localizada en la matriz germinal. Puede ser unilateral o bilateral. • Grado II: H emorragia que se extiende al ventrículo lateral sin producir dilatación. • Grado III: H emorragia intraventricular acompañada de dilatación ventricular aguda y

concomitante. Conviene aclarar que la hidrocefalia secundaria que se produce luego de varios días no se considera como grado III, pues se debe a dificultad en la reabsorción de LCR en las granulaciones de Paccioni, por la obstrucción producida por el sangrado. •Grado IV: C ompromete al parénquima cerebral vecino. • Esta clasificación en grados es importante y tiene valor pronóstico. A mayor grado, más

frecuentes e importantes serán las secuelas neurológicas.

Cuadro Clínico La HIV tiene generalmente un inicio súbito, a las 24 - 48 horas del nacimiento, pudiendo aparecer en un prematuro con una clínica post-parto normal y lógicamente debe esperarse su aparición en los que tienen cuadros respiratorios asociados, en los nacidos de madres muy jóvenes, en los de peso muy bajo y cuando el período gestacional es más corto, recordar que el 90% de los nacidos con una edad gestacional de menos 32 semanas hacen HIV (37,39). El cuadro habitual es de aparición de convulsiones, paro cardíaco o apnea prolongada, con toma del sensorio, fontanela abombada y caída del hematocrito; pero hay pacientes que pueden tener poca o ninguna sintomatología (48,54) y por esto es importante descartar una HIV ante la presencia de una caída del hematocrito sin explicación aparente. Se conoce que una fontanela normotensa no descarta un cuadro de HIV. Nosotros tenemos pacientes RN que llegaron a nuestro servicio para tratamiento de una hidrocefalia aparentemente congénita (sin antecedentes de ningún tipo que sugiriera que fuera secundaria) y detectamos que eran portadores de un líquido ventricular hemorrágico; por eso es nuestra práctica en la actualidad realizar una punción ventricular para estudio del LCR(tanto del aspecto como citoquímico) a toda hidrocefalia que debuta en los primeros treinta días de la vida. Diagnóstico Está bien establecido la vigilancia estrecha de los prematuros en general y en especial los que constituyen el grupo de "alto riesgo" (véase fisiopatología y patogenia). A todo

prematuro se le debe realizar estudios con ultrasonidos diarios durante la primera semana de nacido y posteriormente semanales hasta el alta. Mucho se ha discutido acerca de la investigación ideal a realizar en estos pacientes. Algunos recomendaron la Tomografía Axial Computada (TAC) por los detalles que la misma puede aportar; sin embargo tiene el inconveniente de que al niño hay que trasladarlo hacia el Departamento de Imagenología y al mismo tiempo va a recibir una cantidad enorme de radiaciones, en un paciente inmaduro, lo que puede ser perjudicial. La Ultrasonografía (US) es indudablemente el estudio ideal, pues permite realizar el diagnóstico al lado de la cama y

repetirlo las veces que se crea necesario sin efectos deletéreos sobre el paciente; debe tenerse en mente la necesidad de visualizar la fosa posterior. Para nosotros el mayor valor del US está dado por la evolutividad del mismo y claro está en los casos que ofrezca dudas se puede complementar con la TAC (11,12,37,39,40,41,48). Los estudios Ultrasonográficos, para la mayoría de los autores, permiten clasificar las HIV en cuatro grupos fundamentales (6,11,12,62): Grado I: Cuando el sangramiento está localizado en la matriz germinal, sin o mínima hemorragia intraventricular. Grado II: El sangramiento es intraventricular y ocupa entre el 10 y el 50 % del ventrículo. Grado III: El sangramiento intraventricular es mayor del 50 % y el ventrículo lateral está distendido. Grado IV: Cuando a lo anterior se asocia sangramiento intraparenquimatoso. Volpe(63) sólo reconoce los tres primeros grupos. Una vez establecido el diagnóstico de HIV se realiza seguimiento evolutivo con US, buscando la aparición signos de Hidrocefalia. James y colaboradores (30) sugieren los siguientes criterios para su diagnóstico, atrium ventricular mayor de 10 mm, Indice de Evans mayor de 0,35 y cuando el cuerpo del ventrículo lateral, en un plano sagital, a nivel del tálamo mide más de 10 mm. 5.- Femenino de 36 años. Acude a consulta por presentar cefalea, cansancio e irregularidades menstruales con ritmo de 36 a 50 x 2-3 días. No se ha podido embarazar después de 18 meses de actividad sexual regular. No tiene antecedentes importantes. EF: Campos visuales normales, tiroides aumentada de tamaño una vez y aumentada de consistencia, no tiene galactorrea. Resto normal. Laboratorio: química sanguínea, Bh y electrolitos normales. Prolactina 47 ng/dL (< 25), perfil tiroideo: TSH 18 mUI/ml, T4t: 50 nmol/L (57.9 a 154.4), T4L: 7.7 pmol/L (9 a 24), T3T: 1.06 nmol/L (1.2 a 2.9), T3L: 1.96 pmol/L (3 a 6.31) El diagnóstico es: a) b) c) d)

Hiperprolactinemia Hipertiroidismo Hipotiroidismo primario Síndrome de ovarios poliquísticos

DIAGNOSTICO Inicialmente el hipotiroidismo se diagnosticaba mediante la cuantificación por técnicas de Radio Inmuno Análisis (RIA) de las hormonas circulantes triyodotironina y tiroxina; el proceso era lento y sometido a muchos factores de error que hacían su sensibilidad y especificidad poco confiables. Posteriormente, se desarrollaron técnicas para la medición de la TSH hipofisiaria igualmente mediante el RIA lo que mejoró en forma importante la sensibilidad para el diagnóstico de esta enfermedad; sin embargo, los niveles de detección de la prueba se encontraban en el orden de 1 µIU/ml lo que hacía que la prueba no fuera sensible para valores menores de 1 µIU/ml. Debido a esto se crearon técnicas de segunda generación mediante la cuantificación de TSH por anticuerpos monoclonales y RIA, el IRMA (Immuno Radiometric with Monoclonal Antibodies) que permitió detectar valores de TSH en rangos de 0.1 µIU/ml; posibilitando desde entonces diagnosticar pacientes con

hipertiroidismo primario; pero con la limitante de que para esta técnica era imposible detectar valores de TSH menores de 0.1 µIU/ml por lo que se creo la medición de TSH mediante quimioluminiscencia o métodos enzimáticos, es decir las técnicas de tercera generación, las cuales pueden detectar valores de TSH de 0.01 µIU/ml; con lo que se logra el espectro ideal para una prueba de laboratorio que tiene la capacidad de diagnosticar tanto la hipofunción como la hiperfunción(20). Además el avance no sólo fue en la medición de TSH sino también en las hormonas tiroideas que han evolucionado simultáneamente con la TSH y ya se miden incluso las fracciones libres de hormonas y las fracciones totales, lo que ha facilitado el manejo de estos pacientes. Gracias a esta evolución en técnicas de laboratorio, el diagnóstico de hipotiroidismo primario es bastante sencillo. Niveles de TSH superiores al valor máximo de la técnica serían diagnósticos de la disfunción; pero no es tan fácil. Cuando tenemos un paciente con toda la sintomatología del hipotiroidismo y la TSH se encuentra elevada el diagnóstico es obvio; pero podemos tener pacientes con síntomas muy inespecíficos como depresión y con examen físico normal a quienes se les encuentran valores de TSH por encima del límite superior y con hormonas tiroideas normales. Se trata de un hipotiroidismo o es un valor ligeramente elevado ocasional de una persona sana(21). Igualmente tenemos otra circunstancia que ha sido descrita con mayor frecuencia: pacientes con valores de TSH en el límite superior normal y con dislipidemia a quienes se les da tratamiento con hormonas tiroideas y su dislipidemia se corrige manteniendo valores de TSH en rangos normales. Todas las circunstancias anteriores han hecho que aparezca en el hipotiroidismo primario la expresión de hipotiroidismo subclinico, que ha sido objeto de reuniones y congresos dedicados exclusivamente a este tema. La sociedad Europea de Tiroides hace algunas recomendaciones para el manejo de esta situación que se consideran útiles como guía (Tabla).

Tabla. Enfoque del paciente con disfunción tiroidea de acuerdo a los niveles de hormona estimulante de la tiroides (TSH). Si TSH 0.4 a 2.0 Si TSH < 0.4m U/L mU/L Si TSH 2.01 a 5.0mU/L Normal, Medir T3 y T4 totales o Repetir libres para diagnostico cada cinco Medir T4 libre y anticuerpos de hipertiroidismo. años antitiroideos 1. Si AAT (-) y T4 libre es normal repetir screening cada año. Si TSH es > 4.0mU/l en dos ocasiones dar tratamiento 2. Si AAT (+) y/o T4 libre esta baja o normal baja tratar si

Si TSH > 5.0 mU/L Dar tratamiento para hipotiroidismo

TSH es mayor de 3.0 mU/l y observar a los otros Tomado de Koutras DA. Subclinical hypothyroidism. En G. Hennemann, E.P. Krenning, Thyroid International Merck KGaA, Darmstadt 1999 (3), 6-9

6.- Femenino de 26 años su padecimiento actual inició hace 4 años, cuando sus ciclos menstruales empezaron a ser irregulares. Su ritmo actual es de 40-90 x 3-4. FUR: hace 3 meses. Inició vida sexual a los 23 años y no ha podido embarazarse. En la exploración física encontramos la piel ligeramente seca, hay salida de líquido blanquecino escaso a la expresión del pezón izquierdo y tiene giordano positivo derecho. El estudio que es de mayor utilidad para aclarar el diagnóstico es: a) b) c) d)

Prueba de embarazo LH, FSH y estrógenos Tomografía de cráneo Prolactina

En toda paciente con galactorrea, trastornos menstruales, hirsutismo, disminución de la libido o infertilidad, deben determinarse los niveles de PRL plasmática, entre el tercer y quinto día del ciclo menstrual, si este es regular o en cualquier momento, si presenta amenorrea u oligomenorrea.2 Si en la primera determinación se obtienen cifras elevadas debe repetirse y si se confirma nuevamente, se establece el diagnóstico de hiperprolactinemia. Una historia clínica com-pleta, con un interrogatorio y examen físico detallados, permitirá orientarnos hacia la causa de la hiperprolactinemia. La primera causa que se debe descartar antes de emprender otros estudios, es el embarazo. El uso de estrógenos, anticonceptivos orales y drogas que aumentan la secreción de PRL debe precisarse en el interrogatorio, así como la presencia de quemaduras u otras lesiones en tórax que se buscan en el examen físico. Pocas mediciones hormonales tienen el significado clínico que se observa con la prolactina. La técnica está bien estandarizada y presenta bastante confiabilidad; la muestra de sangre se puede obtener a cualquier hora del día y debido a las variaciones fisiológicas (efecto del ejercicio, alimentos, irritación de la pared costal) cuando se obtiene una cifra de prolactina por arriba de lo normal, es necesario repetir el análisis. Varios fármacos pueden producir una moderada elevación en los niveles de prolactina, sin alcanzar los valores que se encuentran en prolactinoma; las drogas más comunes son metoclopramida, fenotiazinas, risperidona, verapamil, metildopa, reserpina y los inhibidores de MAO. Exámenes: prolactina plasmática (prolactinemia), tirotropina (hormona estimulante de la glándula tiroides o TSH), test de embarazo. Prolactina menor de 100 ng/ml: baja sospecha de prolactinoma. Prolactina entre 100 y 300 ng/ml: mayor posibilidad de prolactinoma.

Prolactina superior a 300 ng/ml: alta sospecha de prolactinoma. Es precisa resonancia magnética nuclear (RNM) de hipófisis (detecta prolactinoma mayor de 3mm). En el 50% de los casos de hiperprolactinemia se detecta tumor: microadenomas o macroadenomas; la prolactina suele encontrarse entre 100 y 200ng/ml; los macroadenomas tienen efecto de masa y causan hipopituitarismo; los microadenomas no presentan efecto de masa y sólo producen hipogonadismo. Referencias: 1. Schlechte JA. Prolactinoma. N Engl J Med 2003;349:2035-2041. 2. Zárate A, Canales ES, Jacobs LS, Soria J, Daughaday WH. Restoration of ovarian function in patients with the amenorrhea-galactorrhea syndrome after long-term therapy with L-Dopa. Fertil Steril 1973;24:340. 3. Tyson JE, Carter JN, Andreassen B, Huth J, Smith B. Nursing mediated prolactin and luteinizing hormone secretion during puerperal lactation. Fertil Steril 1978;30:154. 4. Schlechte JA, Sherman BM, Chapler FK, VanGilder J. Long-term followup of women with surgically treated prolactin-secreting pituitary tumors. J Clin Endocrinol Metab 1986;62:1296-301. 5. Losa M, Mortini P, Barzaghi R, Gioia L, Giovanelli M. Surgical treatment of prolactinsecreting pituitary adenomas: early results and long-term outcome. J Clin Endocrinol Metab 2002;87:3180-3186.

7.- Femenino de 25 años, acude a urgencias por cuadro de 3 días de evolución caracterizado por dolor en cuadrante superior derecho, de inicio insidioso, intermitente, opresivo, sin irradiación y de intensidad 7/10. Ingiere paracetamol para cefalea ocasional y anticonceptivos orales desde hace varios años. El ultrasonido demuestra una imagen bien delimitada, hiperecóica de 2 cm de diámetro en el lóbulo hepático derecho. ¿El diagnóstico más probable en esta paciente es? a) b) c) d)

Carcinoma hepatocelular. Adenoma hepático. Quiste hepático. Metástasis hepática.

Los adenomas hepáticos (AH) son proliferaciones benignas de hepatocitos, que suelen presentarse en mujeres entre los 20 y los 40 años de edad y con antecedentes de toma de anticonceptivos orales. Infrecuentemente se presentan en varones, siendo la proporción de varones: mujeres de 1:11. El síntoma más frecuente es el dolor abdominal vago crónico, que si se localiza en el cuadrante superior derecho suele ser por una hemorragia intratumoral. Actualmente, las técnicas de imagen con mejor rendimiento para su diagnóstico son la TC helicoidal multifásica y la resonancia magnética (RM).

Hugh TJ, Poston GJ. Benign liver tumors and masses. Surgery of the liver and biliary tract. 3ª edición. London, 2000; tomo 2 p.1397-1422 8.- Cuando los resultados en la manometría esofágica muestran un aumento de la presión basal del esfínter esofágico inferior (EEI) junto a una disminución o ausencia de su relajación con la deglución, estamos frente a un cuadro de: a) b) c) d)

Acalasia. Esclerodermia con afectación esofágica. Enfermedad por reflujo gastroesofágico Espasmo esofágico difuso.

Aspecto radiográfico Un esofagograma puede mostrar la disminución de peristalsis, la dilatación del esófago proximal y el estrechamiento del esófago en su parte inferior. El paciente se traga una solución de bario, con fluoroscopia continua que son grabaciones de rayos X para observar el flujo del fluido a lo largo del esófago, sin que se observe el movimiento peristáltico normal del esófago. Hay un agudo estrechamiento en el esfínter esofágico inferior y reducción del diámetro en la unión gastro-esofágica. La imagen que proyecta se denomina clásicamente «en pico de loro» o «en cola de ratón». Por encima de la reducción, el esófago a menudo se observa con una dilatación de diversos grados a medida que poco a poco se va estirando en el tiempo. Por la falta de movimientos peristálticos, se suele observar en la radiografía un margen entre aire y líquido. Manometría esofágica Debido a su sensibilidad, el diagnóstico es confirmado por medio de una manometría esofágica, que mide las presiones del esófago mediante una sonda nasoesofágica y permite comparar las presiones en situación basal y durante la deglución.4 Se inserta un tubo delgado a través de la nariz, y se le instruye al paciente a deglutir varias veces. La sonda mide las contracciones musculares en diferentes partes del esófago durante el acto de la deglución. La manometría revela la falla del EEI para relajarse con cada deglución y la falta de peristaltismo funcional del músculo liso en el esófago. Para descartar complicaciones se suele acudir a una endoscopia digestiva alta.

Esquema manométrico de acalasia demostrando contracciones aperistálticas, un aumento de la presión intraesofágica y el fallo de la relajación del esfínter esofágico inferior. Debido a la similitud en los síntomas, la acalasia se puede confundir con trastornos más comunes, tales como la enfermedad de reflujo gastroesofágico, la hernia de hiato, e incluso trastornos psicosomáticos.

REFERENCIAS BIBLIOGRÁFICAS 1. García Gutiérrez A. Acalasia de esófago. http://www.sld.cu/galerias/pdf/uvs/cirured/acalasia..pdf Consultado Mayo 27, 2006.

Disponible

en:

Fareras Rozman, et al. Acalasia esofágica.Tratado de Medicina Interna. 15 Edición. España: Ediciones Harcourt; 2003. Sección 2. Cap 20.p.354-9. 9.- Masculino que acude al servicio de consulta externa presenta prurito y lagrimeo en ambos ojos, hiperemia conjuntival, fotofobia, exudado más o menos viscoso y formación papilar en la conjuntiva tarsal. ¿El diagnóstico probable es?:

a) Queratoconjuntivitis seca. b) Conjuntivitis alérgica. c) Conjuntivitis bacteriana. d) Conjuntivitis vírica. Conjuntivitis alérgica. Condición inflamatoria ocular bilateral, crónica y recurrente, que se presenta predominantemente en la infancia y con mayor incidencia en el sexo masculino. Las exacerbaciones se asocian a ciertas estaciones del año (donde el nombre de "primaveral")

en las cuales se supone que existe una mayor cantidad de alergenos en el medio ambiente; por ejemplo, el polen de las plantas. Estos pacientes frecuentemente presentan historia personal o familiar de atopia. El síntoma principal es el prurito ocular intenso, además de secreción mucoacuosa matutina, fotofobia severa y lagrimeo. Clínicamente se observa

intensa hiperemia y quemosis conjuntival bulbar, presencia de papilas gigantes en la conjuntiva tarsal, secreción mucoide abundante, ocasionalmente se observan infiltrados eosinófilos a nivel de limbo que se conocen como "puntos de Trantas" y en etapas crónicas

se aprecia acúmulo de pigmento en conjuntiva bulbar y opacificación corneal secundaria a queratopatía punteada. Es frecuente que estos pacientes presenten una sobreinfección debido al contacto de las manos con estructuras oculares por el prurito incontrolable. Se ha demostrado presencia de eosinófilos e inmunoglobulina IgE; sin embargo, el diagnóstico es esencialmente clínico. El tratamiento se divide en medidas preventivas, paliativas y antiinflamatorias. Las medidas preventivas se orientan a evitar la exposición directa al sol, utilización de visera y lentes oscuros; el tratamiento sintomático se basa en la aplicación de fomentos fríos, lubricantes y vasoconstrictores tópicos y el uso de antihistamínicos orales. Como antiinflamatorios tópicos en etapas agudas se emplean cursos cortos de esteroides. Los estabilizadores de células cebadas (cromoglicato de sodio) y antihistamínicos tópicos tienen efecto a largo plazo. Generalmente estos pacientes presentan mejoría importante en la intensidad y frecuencia de los episodios al alcanzar la adolescencia.

Referencias Bibliográficas: Miller Stephen JH. Parsons’ Diseases of the Eye. 18th ed. Edinburgh, London, Melbourne

and New York: Churchil Livigstone; 1990.

Sánchez Salorio M, Rodríguez Ares T, Alio y Sanz JL, Pita D, Fontanela JR, Grau M et al. Conjuntivitis, Edika-Med S.A. Barcelona: CIBA VISION, 1992. Foulks GN, Pavan-Langston D. Cornea and External Disease. In: Pavan-Langston D, editor. Manual of Ocular Diagnosis and Therapy. 2nd ed. Boston: Little, Brown & Co. 1991. p. 100102. 10.- 2 year old female, is taken by her mother to the emergency room complaining of intermittent abdominal pain. Medical history of 6 to 24 bloody evacuations in the last 24 hours. Vital signs: BP 110/65, CR 90 pm, RR 28 pm, tempetature 36.5 °C. The child is irritable, crying loud and in the physical exploration there is a abdominal mass in the right lower quadrant. Whath would be the most probable diagnosis? a) Meckel´s diverticulum. b) Apendicular abscess. c) Intestinal intususception. d) Acute appendicitis.

Invaginación intestinal: Ocurre cuando una porción del tracto alimentario se pliega dentro del segmento adyacente. Es la causa más frecuente de obstrucción intestinal entre los 3 meses y los 6 años de edad. El cociente varón:mujer es 4:1. Se produce de forma súbita con dolor cólico paroxístico intenso recurrente a intervalos frecuentes, el 60% de los lactantes expulsan heces que contienen sangre roja y moco, las deposiciones en confitura de grosella. Nelson, Tratado de Pediatria, 17ª Edición, Ed. Elsevier, Pág.1242-1243.

11.- Masculino de 52 años de edad, jardinero, acude a consulta debido a que presenta unas lesiones en su mano y antebrazo. La lesión inicial fue una pápula pequeña, no dolorosa que se expandió lentamente y desarrolló un área central necrótica. Posteriormente el paciente presentó numerosos nódulos subcutáneos en su antebrazo. Los nódulos aumentaron en tamaño progresivamente, refiere que uno de los nódulos se ulceró (aproximadamente 1 mes después de la aparición de las lesiones). No refiere síntomas sistémicos. También notó que los nódulos se encuentran acomodados como en cadena. ¿Cuál de los siguientes es el diagnóstico más probable?

a) b) c) d)

Aspergilosis Candidiasis Mucormicosis Esporotricosis

DEFINICIÓN La esporotricosis es una infección subaguda o crónica causada por el hongo dimorfo Sporothrix schenckii. Es una enfermedad cosmopolita y probablemente la micosis subcutánea más frecuente en todo el mundo.1, 2. Micosis subcutánea, se localiza preferentemente en cara y extremidades, afecta piel y vasos linfáticos, se caracteriza por nódulos o gomas que dan lugar a lesiones fijas verrugosas o linfangíticas, de evolución subaguda o crónica,. En raras ocasiones es extracutánea o sistémica afectando pulmón, huesos o articulaciones. EPIDEMIOLOGÍA México: Sur del D. F.,Puebla, Guanajuato,San Luis Potosí y Estado de México. Jalisco y Nayarit.  Se han comunicado epidemias familiares y en empacadores de loza. 

 

Clima templado y húmedo, promedio entre 20 y 25°C y humedad relativa superior a 90%. La mayoría de los casos se presenta a finales de otoño y principios de invierno.



Se ha aislado del suelo, detritus vegetales, madera, hojas y ramas ya sean frescas o secas, paja y zacate.



Campesinos, jardineros, floristas y carpinteros. Se presenta en igual proporción en hombres que en mujeres. Grupos de edad más afectados: niños entre 5 y 15 años en un 30% de los casos y entre los 16 y los 35 años de edad en un 50% de los casos.

 

 

El período de incubación en los casos cutáneos es de una semana a un mes. En los raros casos pulmonares se desconoce. Vía de ingreso por un traumatismo cutáneo.

PATOGENIA  



 



  

Esporotricosis cutánea. La lesión inicial es un chancro esporotricósico, constituido por una lesión nodular o gomosa, ulcerada, dos semanas después se presenta un complejo cutáneo linfangítico, siguen los linfáticos regionales, puede involucionar y presentar cura espontánea. Cuando el proceso continua se extiende por contigüidad presentando una cadena de gomas eritematovioláceas,no dolorosas que siguen los vasos linfáticos regionaleshasta el linfático mayor. CUADRO CLÍNICO Linfangítica ascendente de las extremidades Presencia de lesiones gomosas dstribuidas a lo largo del trayecto linfático, en especial de extremidades superiores, siguiendo de manera paralela la extremidad. CUADRO CLÍNICO CUTÁNEO FIJA (20 a 30%) Forma crónica No tiende a la diseminación Lesión única Verrugosa o vegetante Borde eritemato-violáceo Bordes bien limitados Cubierta con escamas y costras melicéricas Asintomática Tendencia a la curación



DIAGNÓSTICO DE LABORATORIO



Examen directo no se emplea da resultados negativos Cultivo son el mejor método. Sabouraud y micosel agar Incubación 28°C Crecimiento 5 a 8 días En medios de cultivo ricos (gelosa sangre, BH etc) se obtienen colonias levaduriformes.

    



TRATAMIENTO

Yoduro de Potasio es la terapia de elección Dosis Niños 1 a 3 g. al día Dosisn adultos 3 a 6 g. al día 20 g. de KI en 300 cc de agua Frasco obscuro (15 cc de solución (una cucharada sopera) da una concentración de 1 g). Tiempo de Tx. 3 meses promedio y continuar 2 meses más. 

Rodríguez GH, Magaña RMC, Juárez L, Arenas R. Esporotricosis cutánea diseminada: comunicación de un caso. Dermatol Rev Mex 2008;52(5):228-30. La versión completa de este artículo también está disponible en: www.revistasmedicasmexicanas.com.mx

12.- Masculino de 52 años acude al servicio de urgencias por presentar fiebre de 39.0c, exantema máculo-papuloso generalizado, incluyendo palmas y plantas. El paciente labora en el campo ordeñando vacas frecuentemente parasitadas por garrapatas. Señale la enfermedad a la que se refiere, el germen causante y el tratamiento adecuado: a) Kala-azar, Leishmaniae Donovani: Antimoniales. b) Fiebre Q, Coxiella Burnetti, Doxiciclina. c) Fiebre botonosa, Ricckettsia Conori: Doxiciclina. d) Fiebre de Malta, Brucella Mellitensis: Cotrimoxazol.

Las rickettsias son organismos coco-bacilares, Gram negativos, de 2-3 micras de diámetro, intracelulares, incapaces de crecer en ausencia de células vivas del huésped. El género Rickettsia  es ubicado taxonómicamente en la familia Rickettsiaceae , junto a otros dos géneros: Coxiella , con la especie C.burnetii, responsable de la fiebre Q, y Ehrlichia  con las especies E.chaffeensis, agente de la ehrlichiasis monocítica, y E.phagocytophila, productora de la ehrlichiasis granulocítica humana. Junto a la familia Rickettsiaceae  está la familia Bartonellaceae  con tres especies principales: Bartonella henselae, agente de la enfermedad por arañazo de gato (“cat scratch disease”); B.quintana, responsable de la angiomatosis bacilar(1), y B.bacilliformis , productora de la bartonelosis o enfermedad de Carrión (verruga peruana). Las rickettsiosis son zoonosis transmitidas desde los huéspedes o reservorios animales al hombre a través de picaduras de artrópodos diversos, que varían con cada enfermedad (piojos, pulgas, garrapatas, otros ácaros, esencialmente). La rickettsiosis por R.conorii es conocida con el nombre de fiebre botonosa o manchada del Mediterráneo o fiebre de Marsella, siendo transmitida al hombre desde el perro que constituye su reservorio por garrapatas de los géneros Amblyomma   y Riphicephalus  principalmente. En ellas el germen cumple un ciclo que incluye el pasaje transovárico a los descendientes por lo cual representan también otro verdadero reservorio del parásito. Clínicamente la enfermedad se caracteriza por la aparición en el sitio de la picadura de la garrapata de una lesión inicial indurada con centro necrótico muchas veces (“mancha negra” o “tache noir”) rodeada de aureola inflamatoria, seguida de adenopatías regionales de carácter inflamatorio en los días subsiguientes. Concomitantemente, fiebre frecuentemente alta de 39º-40ºC, malestar general, cefaleas a veces intensas, dolores musculares y articulares. Es relativamente frecuente la observación de un exantema máculo-papuloso que explica el nombre de fiebre botonosa y que puede afectar varios territorios. Es una afección endémica en Sudáfrica, Europa del Sur y Medio Este (2). El diagnóstico se confirma esencialmente por la técnica de inmunofluorescencia indirecta (IFI) empleando láminas que contienen antígenos de R.conorii  y utilizando, siempre que sea posible, sueros pareados obtenidos al inicio del cuadro y 20-30 días después para investigar la seroconversión. La histopatología de las lesiones iniciales o “taches noires” fue estudiada en detalle por Montenegro y colaboradores en 1983(3). En otro trabajo posterior, Montenegro y

colaboradores(4) demuestran en ratones inoculados con R.conorii  la importancia crucial de la inmunidad celular con respecto a la humoral en el control de la infección experimental y reducción del índice de mortalidad. El ratamiento de elección es dicloxacilina, alternativas macrólidos y quinolonas (ciprofloxacino). Bibliografía: 1 . Sampaio SAP, Rivitti EA. Dermatologia. São Paulo: Artes Médicas, 1998: 1155. 2 . Harris RL, Kaplan SL, Bradshaw MW, Williams Jr, Temple W. Boutonneuse fever in american travelers. J Infect Dis 1986; 153:126-8. 3 . Montenegro MR, Mansueto S, Hegarty BC, Walker DH. The histology of “taches noires” of boutonneuse fever and demonstration of Rickettsia conorii  in them by immunofluorescence. Virchows Arch (Pathol Anat) 1983; 400:309-17. 4 . Montenegro MR, Walker DH, Hegarty BC. Infection of genetically immunodeficient mice with Rickettsia conorii  . Acta Virol 1984; 28:508-14. 5. Conti Díaz IA, Rubio I, Somma Moreira RE, Pérez Bormida G. Rickettsiiosis cutáneo-ganglionar por Rickettsiaconorii  en el Uruguay. Rev Inst Med Trop (São Paulo), 1990;

13.- Masculino de 19 años tacleado mientras jugaba football se presenta con dolor severo en rodilla izquierda. A la EF la rodilla se encuentra edematizada y el paciente tiene dolor a la palpación directa del aspecto lateral de la rodilla. Cuando se flexiona la rodilla 30°, la aducción pasiva ocasiona dolor en la misma área, y la pierna puede ser aducida más que la pierna contralateral. Cajón anterior, cajón posterior y Lachman negativos. ¿Cuál es el sitio más probable de lesión? a) b) c) d)

Ligamento cruzado anterior Ligamento colateral lateral Menisco lateral Ligamento cruzado posterior

Se produce por un trauma en varo al golpearle en la parte externa de la pierna por debajo de la rodilla o al caerle otro jugador o contusión en la cara interna de la rodilla. Presenta dolor y generalmente sensación de parestesia en el territorio del nervio ciático poplíteo externo por distensión, que puede ser una parestesia transitoria en traumas de menor energía o una lesión definitiva con pie caído en traumas de alta energía. El paciente presenta dolor, derrame leve a moderado y equimosis en cara lateral de la rodilla.

El diagnostico se hace al encontrar: 1. Bostezo en varo en 30 grados de flexión : para evaluar el ligamento colateral lateral, el dolor es en trayecto del ligamento. Puede ser de diferente magnitud: •

Grado I: muy discreta apertura de la interlinea comparado con lado contra lateral.



Grado II: apertura de 5 mm de la interlinea articular.



Grado III: apertura de 10 mm de la interlinea articular.

2. Bostezo en varo en extensión completa: para evaluar esquina postero lateral, capsula postero lateral y ligamento fibulo poplíteo. Las estructuras laterales tienen un potencial de cicatrización menor que las estructuras mediales. En las lesiones grado I y II se debe inmovilizar y proteger el apoyo durante seis semanas. Se debe tener un alto índice de sospecha y en los casos grado III y donde se presente compromiso de la capsula postero lateral, se debe realizar cirugía para reparo primario en las primeras dos semanas, pues el reparo primario funciona mejor que cualquier reconstrucción anatómica.

14.- Masculino de 25 años, que 10 días después de acudir a una despedida de soltero, comienza con inflamación de rodilla derecha y de ambos tobillos, conjuntivitis bilateral, aftas orales y erosiones superficiales no dolorosas en el glande. El disgnóstico más probable es: a) b) c) d)

Enfermedad de Reiter Enfermedad de Still. Infección gonocócica. Infección por Staphylococcus Aureus.

Proceso inflamatorio estéril de la membrana sinovial, precedido o precipitado por una infección que ocurre fuera de la articulación.

DIAGNOSTICO: HISTORIA SÍNTOMAS GENERALES MANIFESTACIONES MÚSCULO-ESQUELÉTICAS Artralgias, artritis aditiva o migratoria Monoartritis u oligoartritis asimétrica Articulaciones grandes que sostienen peso: rodillas, tobillos y caderas Dactilitis o dedos en “salchicha” Afección axial: articulaciones S-I y columna lumbar Entesopatía, tenosinovitis •



































MANIFESTACIONES GENITO-URINARIAS Uretritis, balanitis circinada (es importante señalar que las lesiones son indoloras) , prostatitis Cervicitis, cistitis, enfermedad pélvica inflamatoria MANIFESTACIONES CUTÁNEAS Y DE MEMBRANAS MUCOSAS Queratodermia blenorrágica Eritema nodoso Distrofia ungueal Úlceras orales











MANIFESTACIONES OCULARES Conjuntivitis y uveítis. MANIFESTACIONES G-I 

BIBLIOGRAFÍA RECOMENDADA: Klippel JH, Stone JH, Crofford LJ, White PH, editors. Primer on the rheumatic diseases. 13th ed. New York: Springer-The Arthritis Foundation; 2008. Martínez-Elizondo P, editor. Introducción a la Reumatología. 4a ed. México: Colegio Mexicano de Reumatología A.C./Intersistemas S.A. de C.V.; 2008. Firestein GS, Budd RC, Harris ED Jr, McInnes IB, Ruddy S, Sergent JS, editors. Kelley’s Textbook of Rheumatology. 8th ed. Philadelphia: Saunders Elsevier; 2009.

15.- Masculino de 56 años que llega al servicio de urgencias con dolor torácico de más de 12 hrs. de evolución, que inicia en forma súbita, mejora al estar sentado, no tiene antecedentes de importancia, habrá que descartar de primera instancia: a) b) c) d)

TEP Enfermedad ácido péptica Cardiopatía isquémica Lesión de grandes vasos

El manejo de un paciente con este tipo de dolor, requiere una evaluación de la severidad, localización y características peculiares de dicho dolor. Muy pocos síntomas suponen una urgencia tan obligatoria como lo es el dolor torácico. Tanto el médico como el paciente saben que la isquemia miocárdica puede ser causa de muerte súbita, pudiendo generar ansiedad en ambos. La importancia y dificultad en la valoración del dolor torácico radica en la multitud de causas posibles y en el diferente pronóstico según la patología subyacente. Al problema diagnóstico inherente a un síndrome esencialmente clínico, se añade la dificultad de etiquetar el dolor en poco tiempo (ayudados sólo por la clínica, una Rx de tórax y un ECG), dada la importancia de iniciar con prontitud el tratamiento más adecuado en los pacientes con patología potencialmente letal. Aunque el dolor o la molestia precordial constituye una de las manifestaciones principales de cardiopatía, es muy importante recordar que puede originarse no sólo en el corazón, sino también en: estructuras cardiacas intratorácicas como la aorta, la arteria pulmonar, árbol broncopulmonar, pleura, mediastino, esófago y diafragma; tejidos del cuello o la pared torácica, incluidos piel, músculos torácicos, región cervicodorsal, uniones costocondrales, mamas, nervios sensoriales o médula espinal y órganos abdominales como estómago, duodeno, páncreas o vesícula biliar; además de dolor artificial o funcional. Manifestaciones clínicas en cardiopatía isquémica Estas se pueden agrupar en cuatro grandes grupos o categorías, las cuales son: : en donde la obstrucción del riego arterial no es lo suficientemente persistente como para causar muerte del tejido muscular cardíaco; hay tres tipos que son la angina estable, la de prinzmetal y la inestable. : en este caso la obstrucción del riego arterial es lo suficientemente duradera o persistente como para causar necrosis tisular isquémica del miocardio. : son pacientes que generalmente han sufrido uno o más ataques cardíacos y han sobrevivido a ellos, pero continúan teniendo problemas cardíacos debido a que la parte del miocardio que no muere se hipertrofia para suplir las necesidades del cuerpo y esto a su vez causa un aumento de la demanda cardíaca debido al aumento de los componentes estructurales de las células cardíacas, trayendo más problemas porque no

se podrá suplir adecuadamente al corazón debido a la obstrucción coronaria. Estos pacientes constituyen el 50% de los que reciben trasplantes cardíacos. : Es el paro cardíaco en el cual se presentaron síntomas en una hora antes de la muerte, o no se presentaron nunca. Causas: aterosclerosis coronaria, estenosis aórtica, hipertensión sistémica, comúnmente arritmias letales (asistólicas y fibrilación ventricular). BIBLIOGRAFIA: 1. Goldman L., Braunwald E. Molestias torácicas y Palpitaciones. En Isselbacher KJ., Braunwald E., Wilson JD., Fauci AS., Kasper DL., eds. Harrison, Principios de Medicina Interna. McGraw-Hill. Interamericana de España. 1994. 2. Braunwald E. Tratado de Cardiología. Interamericana. Mcgraw-Hill. 1993. 3. Harkins SW. Geriatric pain. Pain perceptions in the old age. Clin Geriatric Med 1996. 4. Coto lópez, A., Morales JM., Gutierrez Rodero, F., Gonzalez E., .Dolor Torácico. Manual de diagnóstico y terapéutica médica. Gutierrez Rodero F y García Díaz JD. 2ª ed. Madrid, 1990; pag. 165-172. 5. Durán Serantes, M., Calderón de la Barca Gázquez, J.M., Romero Moreno M., Martinez Guillén, J., Montero Pérez, FJ., Jimenez Murillo, L., Cardiopatía Isquémica ( I ): Angor. Protocolos de actuación en Medicina de Urgencias. Jimenez Murillo L y Montero Pérez FJ. Mosby/Doyma Libros SA. Barcelona 1996; pág. 51-56. 6. James H. Chesebro. La clínica del dolor torácico en el Servicio de Urgencias: abordaje de los pacientes y relación coste-eficacia. Grandes temas de la cardiología: avances hacia el cambio de siglo. 1998, American College of Cardiology. 7. Tresch DD, Aronow Ws. Clinical manifestations and clinical diagnosis of coronary artery disease. Clin Geriatr Med. 1996. 8. Owens, G.M.: Chest pain. Primary Care, 1986. 13; pág: 55-61. 9. Rutherford, J.D.; Braunwald, E.: Diagnóstico diferencial del dolor precordial. En: Braunwald E: Tratado de Cardiología, 4ª edición. Interamericana McGraw-Hill, Madrid, 1993; pág: 1448-1449. 10. Williams, E.S.: Approach to the patient with chest pain. En: Kelly WN, ed. Textbook of Internal Medicine. Filadelfia, J.B. Lippincott Company, 1989; pág. 374-379.

16.- Femenino de 34 años que inicia con debilidad muscular, episodios frecuentes de ptosis, diplopia y fatiga generalizada. Refiere dificultad a la masticación y deglución. La prueba más específica para corroborar su diagnóstico es: a) Electromiograma de fibra muscular aislada. b) Determinación de anticuerpos anti-receptor de acetilcolina. c) TAC torácico. d) Electromiograma con estimulación repetitiva.

La miastenia gravis es una enfermedad autoinmune que se caracteriza por presentar debilidad muscular fluctuante y fatiga de distintos grupos musculares. La miastenia gravis afecta a individuos de todas las edades, con una predilección por mujeres entre los 20 y 40 años. Los músculos oculares, faciales y bulbares son los más frecuentemente afectados por la enfermedad. Los pacientes con miastenia gravis manifiestan empeoramiento de la debilidad muscular, con infecciones intercurrentes, fiebre y agotamiento físico o emocional. La infección respiratoria (bacteriana o vírica) es la causa más frecuente de provocación. La presencia de anticuerpos contra receptores de acetilcolina en un paciente con manifestaciones clínicas compatibles con la miastenia gravis confirma el diagnóstico. El tratamiento de esta entidad es controvertido y debe ser individualizado, ya que no existe un régimen terapéutico uniforme para todos los pacientes. Entre los tratamientos disponibles testacan los fármacos anticolinesterásicos, los corticosteroides, la plasmaféresis, la inmunoglobulina, los inmunosupresores y la timectomía. REFERENCIAS 1. Drachman DB. Myasthenia gravis. N Engl J Med 1994;330:1797-8102.2. Engel AG. Myasthenic syndromes. In Engel AG, Franzini-Armstrong C, eds. Myology. 2 ed. New York: McGraw-Hill;1994. P. 1798-835. 3. Engel AG. Ohno K. Milone M, Sine SM. Congenital myasthenic syndromes caused by mutations in acetylcholine receptor genes. Neurology 1997;48 (Suppl 5):S28-35. 4. Steinman L, Mantegazza R. Prospects for specific inmunotherapy in myasthenia gravis . FASEB J 1990;4:2726-31. 5. Massey JM. Treatment of acquired myasthenia gravis. Neurology 1997;48 (suppl 5):S4651.

17.- Masculino de 46 años de edad que inicia con dolor intenso en fosa renal izquierda ante la sospecha de litiasis renoureteral se realizan estudios de laboratorio y gabinetem, los Rx demuestran cálculos radio lúcidos, los cuales están relacionados a: a) b) c) d)

Calcio Estruvita Ácido úrico Cistina

-Bruce E. Jarrell, R. Anthony Carabasi, Nacional Medical Series for Independent Study. Wiliams & Wilkins, 3 rd Edition: 451-475. Los cálculos que sepueden presentar en la vía urinaria son formados por calcio, ácido úrico y cistina, los únicos radiolucidos son los de ácido úrico que representan un reto diagnóstico. 18.- Masculino de 24 años presenta dolor y tumefacción del testículo derecho. Su médico solicita un ultrasonido que revela una masa testicular de 2 x 2.5 cm. Se realizan una exploración inguinal y una orquiectomía. El estudio histopatológico reveló un seminoma puro. Una tomografía computadorizada de tórax, abdomen y pelvis mostró dos ganglios retroperitoneales de 3 cm que están aumentados de tamaño. La biometría hemática, la química sanguínea y los marcadores tumorales están todos dentro de los límites normales. ¿Cuál de los siguientes seria la mejor conducta? a) b) c) d)

Extirpación quirúrgica de toda la enfermedad Braquiterapia Observación Radioterapia

Allen R. M. MMS Medicina Interna. 5ª. Edición. National Medical Series. Mc. Graw Hill. 2006. (capítulo 4IX G 2). El cáncer testicular es el más común en varones adultos jóvenes variedades más frecuentes son seminomas y tumores de células germinales no seminomatosos; ambos sor rabies aun en etapas avanzadas. Los seminomas son muy sensibles a la radioterapia; por tanto, los pacientes enfermedad de etapa II (la limitada a testículo y a ganglios por abajo del diafragma) pueden tratarse con < bajas de radiación. Debido a la toxicidad de la médula ósea producida por la radioterapia mediastínica, y está indicada la radiación profiláctica del mediastino. En este contexto, los ganglios linfáticos retroperito aumentados de tamaño denotan la existencia de enfermedad metastásica y está indicado el tratamiento.

19.- Femenino de 64 años que acude al servicio de urgencias por referir dolor precordial, nausea y malestar general, ante un probable infarto agudo al miocardio, ¿Cuál de los siguientes marcadores tiene mayor sensibilidad para realizar éste diagnóstico? a) CPK b) Transaminasa c) Troponina d) Deshidrogenada láctica

Es muy conocida la falta de especificidad de la CPK. La elevación de la actividad de esta enzima se produce tanto en el infarto de miocardio (IAM) como en afecciones caracterizadas por un grado variable de necrosis muscular. Por ello en los últimos años se han realizado, y se siguen realizando, considerables esfuerzos para encontrar nuevos marcadores analíticos que sean capaces de diferenciar estos cuadros. Entre ellos, la determinación de la isoenzima miocárdica de la CPK de forma cuantificada (CPK-MB-Masa) resulta también poco específica ya que se eleva también en caso de necrosis de músculo estriado; está descrita su elevación en traumatismos, rabdomiolisis, convulsiones 1, miopatías agudas y crónicas 2, insuficiencia renal en diálisis 3 e incluso en el ejercicio intenso4,5; y lo mismo podemos afirmar de la mioglobina 6. Más recientemente se han determinado las troponinas T e I en sus isoformas específicas de músculo cardiaco, que tienen una secuencia diferente de aminoácidos a las de músculo estriado no cardiaco, lo que permite el desarrollo de inmunoensayos específicos. El complejo de las troponinas T, C, e I está estrechamente unido al filamento de tropomiosina. La T se encarga de la unión a tropomiosina; la C es iniciadora de la contracción tras unirse al calcio, y la I se llama así por ser inhibidora de la contracción en reposo 7. Esta especificidad ha sido demostrada en varios trabajos que muestran valores normales de troponina-I en procesos que cursan con necrosis de músculo estriado 2,8. Además de ser muy específicas, las troponinas son altamente sensibles en el infarto de miocardio. Mair y col encuentran una sensibilidad del 100% para el diagnóstico si se hace la determinación de troponina-I pasadas 6 horas del comienzo del dolor 9. La cinética de estos marcadores en pacientes con IAM puede resumirse de esta forma: 1. La CPK total comienza a elevarse a las 4-8 horas del comienzo de los síntomas, y permanece elevada hasta que se normaliza a las 48-72 horas 10. 2. La isoenzima MB de la CPK (CPK-MB) aparece en el suero tres horas después del comienzo del IAM, con un pico máximo a las 18-20 horas, alcanzando valores 16 veces superiores al normal. A partir de este punto desciende lentamente y persiste elevada al menos 2 días11. 3. La troponina-I se eleva a partir de las 2-3 horas del comienzo de los síntomas, con un valor máximo a las 16 horas. Desciende bruscamente hasta las 48 horas, y a partir de entonces se produce un lento descenso; puede detectarse todavía el 7º-8º día 11. 4. La mioglobina es la primera que se eleva. Da las cifras más altas de sensibilidad en el plazo de dos horas del comienzo del dolor10 con respecto a la troponina-I y la CPK-MB. Alcanza su pico a las 8 horas y desciende bruscamente a valores normales a las 18 horas 11 Utilidad de la troponina-I, CPK-MB y mioglobina en el diagnóstico del infarto de miocardio y de los procesos de necrosis muscular de origen no cardiaco Use of troponin-I, CPK-MB and myoglobin in the diagnosis of myocardial infarct and   processes of muscular necrosis of non-cardiac origin  J.I. Ibáñez1, R. Sobrado 1, M. Rivero2, J.M. Olite3, I. Idoate3, I. Berrozpe1, E. Arina1, L. Metola 1, J. Sesma1

1. Unidad de Urgencias. 2. Servicio de Medicina Interna. 3. Servicio de Bioquímica Clínica. Hospital Virgen del Camino Pamplona.

20.- Paciente masculino de 42 años, con antecedente de alcoholismO Y DM tipo 2, hospitalizado por TCE. Durante su estancia intrahospitalaria inicia con fiebre en picos (38.5º-39º C), escalofríos, aparición de soplo, la biometría hemática muestra leucocitosis (16,700) se descarta IVU y neumonía. Se sospecha endocarditis infecciosa: La conducta inicial es: a) b) c) d)

Tomar hemocultivos seriados Ecocardiograma transtorácico Hemocultivo a través de catéter Retiro de catéter intravascular

La endocarditis infecciosa es una infección microbiana que se localiza sobre las válvulas cardíacas o sobre el endocardio mural. A pesar que la mayoría de estas infecciones son causadas por bacterias, se considera más apropiado denominarla en forma global como endocarditis infecciosa debido a que también puede ser producida por hongos, rickettsias o clamidias. La endocarditis bacteriana es la forma más reconocida y se clasifica en aguda o subaguda según su presentación clínica. La endocarditis micótica se detecta casi exclusivamente en drogadictos o en pacientes con prótesis valvulares. Hemocultivos con antibiograma Se recomiendan tomar tres muestras a la llegada del paciente (preferiblemente sin tomar antibióticos), las muestras de sangre separado por un intervalo mínimo de 1 h, en un período de 24 horas; al día siguiente debe repetirse el mismo proceder. Debe realizarse una correcta antisepsia previa de la piel y cambiar de aguja cuando se vaya a verter la sangre en los frascos. El cultivo de sangre se realiza en un tubo especializado para hemocultivos automatizados.  Se prefiere dejar al paciente, si el estado clínico así lo tolerara, hasta un mínimo de 72 horas sin antibióticos, antes de tomar la muestra, pero si el paciente por su situación de sepsis avanzada y repercusión hemodinámica no se le puede retirar el antibiótico entonces entonces se utiliza el frasco de hemocultivo automatizado con concentración mínima inhibitoria del antibiótico, precisando en la orden el ó los tipos de antibióticos que se están empleando en la terapéutica antimicrobiana.  Si al realizar los hemocultivos automatizados estos todos vienen negativos a pesar de que la sintomatología del paciente no mejora se pueden indicar los hemocultivos automatizados

con medios de cultivos especiales para gérmenes de crecimiento lento, por ejemplo en medio de Sabureao.  A estos frascos se le añade sangre venosa, entre 5-10 mL en adultos. Se llevará la muestra lo más rápido posible al laboratorio de Microbiología. La toma de muestra se realizará independientemente de la temperatura del paciente o cuando se presentan los escalofríos y no en pico febril.  Para confirmar criterio de curación de la infección previo al egreso hospitalario se realizan 6 hemocultivos automatizados una semana después de suspendidos los antibióticos. La endocarditis infecciosa (EI) es una enfermedad en la que uno ó varios mi-croorganismos infectan el endocardio, las válvulas ó las estructuras relaciona-das, generalmente sobre una lesión ante-rior: cardiopatía reumática en otros tiempos, cardiopatías congénitas en la actualidad. Puede ser súbita y aguda, pero más frecuentemente es subaguda y larvada lo que retrasa el diagnóstico. Tiene morbilidad y mortalidad impor-tante a pesar de los tratamientos antimi-crobianos y de la difusión de su profi-laxis entre los niños susceptibles. Nue-vos grupos de riesgo son los pacientes sometidos a cirugía cardiaca, los niños que precisan catéteres intravasculares, especialmente neonatos, los inmunodeprimidos y los adictos a drogas por vía intravenosa. La ecocardiografía- Dop-pler aporta actualmente criterios básicos para el diagnóstico y seguimiento. Pue-den ser útiles a niños nuevos recursos en el tratamiento y una profilaxis precisa. 21.- Femenino de 34 años con diagnóstico de hirsutismo asociado a ovario poliquístico, usted elige el siguiente fármaco para su tratamiento por ser el más adecuado: a) Clomifeno b) Estrógenos c) Acetato de ciproterona d) Corticoide

El acetato de ciproterona parece ser más efectivo que otros fármacos para el hirsutismo en mujeres causado por la producción ovárica excesiva de andrógenos Una de las causas de hirsutismo (crecimiento piloso excesivo) en mujeres es la hiperproducción de andrógenos a partir del ovario. Varios fármacos pueden utilizarse para contrarrestar los efectos del andrógeno. El acetato de ciproterona es un fármaco antiandrogénico. Los efectos adversos informados con su uso fueron aumento de peso, depresión, fatiga, síntomas mamarios y disfunción sexual. La revisión de los ensayos encontró que el acetato de ciproterona parece ejercer un efecto en el hirsutismo similar a otros fármacos utilizados para el tratamiento del hirsutismo por exceso de andrógenos. No existen pruebas suficientes para comparar los efectos adversos de las opciones de tratamiento.

Van der Spuy ZM, le Roux PA. Acetato de ciproterona para el hirsutismo (Revisión Cochrane traducida). En: La Biblioteca Cochrane Plus, número  4, 2007. Oxford, Update Software Ltd. Disponible en: http://www.update-software.com. (Traducida de The  Cochrane Library, 2007 Issue 4. Chichester, UK: John Wiley & Sons, Ltd.).

22.- Gestante de 9 semanas la cual tiene contacto con un menor que, 6 días más tarde, desarrolla un cuadro de exantema y síndrome general infeccioso sugerente de infección por virus de rubéola. En el primer control serológico gestacional se detectó la negatividad de la IgG específica. ¿De las siguientes afirmaciones la correcta es:? a) La aparición de IgM materna positiva constituye indicación para la determinación de la IgM fetal. b) No existe posibilidad de contagio dado que el niño ya no se hallaba en fase de eliminación viral. c) En este período de la gestación, el riesgo de la infección y de afectación embrionaria es mínimo. d) La existencia de IgM fetal negativa excluye en este caso la posibilidad de transmisión transplacentaria.

INFECCIONES EN EMBARAZADA VACUNACIONES: CONTRAINDICADAS: Parotiditis Rubéola Sarampión Fiebre amarilla NO se RECOMIENDA: gripe, poliomielitis, hepatitis B. SÍ SE PUEDEN DAR (cuando estén indicadas): Fiebre tifoidea Rabia Tétanos Tos ferina (ver pregunta mir, que pone que no) -

-

-

-

-

-

-

-

-

TOXOPLASMOSIS: 50% de transmisión. Si grave (en 1er T, pero es menos frecuente): aborto, parto pretérmino, muerte fetal intraútero. Dx: lo de siempre. Calcificaciones cerebrales. TTO: En casos de seroconversión ESPIRAMICINA (depresión medular, ac. folínico) ht final del embarazo. -

-

Si infección fetal: PIRIMETAMINA Y SULFADIAZINA en ciclos de 3 semanas alternando con la espiramicina ht final del embarazo.

RUBEOLA: Contagio >80% si la madre se contagia en 8 primeras semanas. SORDERA CONGÉNITA. Acs 2 años supraglótica brusco (horas) alta si si sentado si no nasal normal o aumentada corto

Laringotraqueobronquitis viral 6m a 3 años subglótica gradual baja no no decúbito no si, perruna ronca aumentada largo

Croup leve: se recomienda la ingesta de líquidos, antipiréticos, humidificación de secreciones, observación domiciliaria de los signos y síntomas de empeoramiento.4 Queda a criterio médico el uso de Dexametasona 0.15 mg/Kg dosis única, vía oral o Prednisolona 1 mg/Kg vía oral durante 3 días. Si existiere intolerancia oral se puede utilizar la vía intramuscular.4,6 Croup moderado: Se puede utilizar Budesonide (2 mg.) nebulizado, L-adrenalina 1/1000 nebulizada, más dexametasona oral (0.5 mg cada 8 horas), según la evolución.4 Croup severo: lo importante es evitar la necesidad de intubar al niño, se puede aplicar dexametasona por vía parenteral, sumada a adrenalina nebulizada (hasta 3 aerosoles seguidos) y Budesonide nebulizado de apoyo. En caso de no haber respuesta favorable se procederá a entubar al paciente.4 Complicaciones: Ocurren en una minoría de los casos y son generalmente secundarias a la entubación: neumotórax, neumomediastino, superinfección bacteriana, estenosis subglótica, syndrome tóxico producido principalmente por Staphilococo aureus.8

CUADRO CLÍNICO El cuadro inicial es el de una rinofaringitis e incluye irritación nasal, coriza, fiebre generalmente menor de 39 °C, tos seca y odinofagia, con poca o nula afección del estado general; 24 a 48 horas después aparece disfonía, la tos se hace "crupal" (traqueal, perruna

o en ladrido) y se presentan en forma gradual estridor inspiratorio creciente (estridor laríngeo) y signos de dificultad respiratoria de intensidad variable. En la exploración física son evidentes la disfonía, el estridor laríngeo que en ocasiones es audible a distancia, la disminución del murmullo vesicular y datos de dificultad respiratoria.1-6 Forbes describe la progresión del cuadro clínico de la siguiente manera: Etapa 1: Fiebre, ronquera, tos crupal y estridor inspiratorio al molestar al paciente. Etapa 2: Estridor respiratorio continuo, participación de músculos accesorios de la respiración con retracción de costillas inferiores y de tejidos blandos del cuello. Etapa 3: Signos de hipoxia e hipercapnia, inquietud, ansiedad, palidez, diaforesis y taquipnea. Etapa 4: Cianosis intermitente, cianosis permanente, paro respiratorio. Por lo general la enfermedad es leve, en pocas ocasiones progresa más allá de la etapa 1, más del 95% reciben tratamiento ambulatorio; del 5% que requiere hospitalización sólo del 1 al 1.5% requieren medidas de apoyo ventilatorio (intubación endotraqueal o traqueostomía). La duración del padecimiento es muy variable, desde tres a siete días en casos leves, hasta siete a catorce días en casos graves.

BIBLIOGRAFIA 1- Behrman R, Kliegman R, Jasón H. Tratado de Pediatria Nelson. 17º Edición. España. Elsevier 2004:1508-1503 2- Behar RR. Prevención de las infecciones respiratorias agudas. Presente y futuro. La Habana Cuba. Hospital Pediátrico Universitario William Soler Septiembre 2002; Boletín N° 2 3- Mendoza A, Mejías H, Schmidt G. Educación Continua en Pediatría. Croup. Revista de Pediatría. 2002; 40(2):46-51 4- Basanta A. Croup. Anales de la Pediatria 2003; Monog. 1(1):55-61 5- Kimpen Jann L. Management of respiratory syncytial virus infection. Curs Opin Infect Dis 2001; 14:323-8.

6- Hall C. Respiratory Sincytial and Parainfluenza Virus. N Engl J Med 2004; 344:1917-28 7- Wright R, Pomerantz W, Curia J. New Approaches to respiratory infections in children. Bronchiolitis and croup. Emerg Med Clin North Am 2002;20:93-114 8- Muñiz A. Croup. Emedicine [en línea] 2004 [fecha de acceso 10 de marzo del 2006]; 11:287-9. 25.- Masculino de 44 años acude a consulta con estudios de laboratorio que reportan hipercalcemia el paciente se refiere asintomático. El resto de sus exámenes de laboratorio muestran elevación de parathormona, baja en fósforo, elevación de cloro,BUN y creatinina normales. El calcio urinario está elevado. ¿La etiología más probable en éste paciente es? a) b) c) d)

Mileoma múltiple Hiperparatiroidismo primario Hipervitaminosis Sarcoidosis

Al grupo de síndromes que tienen como característica común la secreción excesiva y no controlada de hormona para-tohormona (HPT) por una o más paratiroides funcionantes se le conoce como hiperparatiroidismo primario (HPP). La causa más frecuente es el adenoma y le sigue en frecuencia la hiperplasia. El cuadro clínico del HPP afecta principalmente al sistema óseo, al riñón y al sistema gastrointestinal. El cuadro clínico se caracteriza por debilidad muscular, fatigabilidad fácil, manifestaciones gastrointestinales y depresión. Cuando hay afección renal el paciente presenta hipercalciuria (25%), litiasis renal recidivante (25%) y/o nefrocalcinosis (20%). Cuando hay resorción subperióstica, tumores pardos en huesos largos o en el maxilar inferior y a nivel del cráneo, las lesiones osteolíticas le dan el aspecto de “sal y pimienta”. En los exámenes de laboratorio hay hipercalcemia, hipofosfatemia, fosfatasa alcalina elevada (fracción ósea), elevación de HPT (90%), osteocalcina y desoxipiridinolinas; así como calciuria e hiperfosfaturia. Cuando hay afección del sistema gastrointestinal hay úlceras y hemorragias. Por lo tanto, como podemos observar, se trata de una paciente con HPP clásico con afección del sistema óseo y del riñón, sin manifestaciones gastrointestinales aparentemente, el cual al someterse a tratamiento quirúrgico y médico adecuados hubo mejoría inmediata que ha continuado hasta la fecha actual. Hiperparatiroidismo primario Ma. Esther Gutiérrez Díaz Ceballos,1 Héctor A. Rodríguez Martínez,1 Evelyn M. Torres Acosta,1 Humberto Cruz Ortiz1 1 Unidad de Patología y Servicio de Endocrinología del Hospital General de México y de la Facultad de Medicina, UNAM

26.- Masculino de 32 años que inició con disfonía y disfagia, es referido al servicio de endocrinología donde es confirmado el diagnostico de nódulo tiroideo único, ¿El estudio recomendado es? a) b) c) d)

Gammagrama tiroideo Tomografía computada de cuello Biopsia por aspiración con aguja fina Repetir Pruebas de función tiroidea

El nódulo tiroideo solitario se define como el crecimiento localizado de la glándula tiroides, usualmente es benigno, la prevalencia es del 4 al 7% en la población general. Aunque el cáncer tiroideo es el tumor endocrino más común sólo representa el 1% de todos los cánceres y 5% de todos los nódulos tiroideos. El estudio clínico diagnóstico y terapéutico debe iniciarse con historia clínica completa, exploración física y exámenes de laboratorio que incluyan un perfil tiroideo para evaluar función de la glándula . La citología tiroidea por aspiración (CTA) es el principal procedimiento diagnóstico en los pacientes con nódulo tiroideo solitario, por ser capaz de diferenciar lesiones benignas de las malignas, sus principales ventajas son: segura, reduce costos de atención médica, selecciona mejor los pacientes que serán sometidos a tratamiento quirúrgico y se realiza en pacientes ambulatorios. Torres AP, Hernández SE, Caracas PN, Serrano GI et al Diagnóstico y tratamiento del nódulo tiroideo. Rev Edocrinol Nutr 2000; 8 (3): 87-93.

27.- Masculino de 47 años con datos positivos de alcoholismo crónico, al cual se le diagnóstica pancreatitis aguda ¿Cuál es la primera medida terapéutica a adoptar? a) Iniciar antibióticos de amplio espectro. b) Administración de inhibidores de la bomba de protones. c) Ayuno absoluto d) Aspiración nasogástrica.

El 80 % de los pacientes con pancreatitis aguda se tratan mediante medidas de sostén, suspensión de la vía oral, hidratación intravenosa, analgésicos y alimentación parenteral cuando la vía oral se restringe más allá de la semana.

La indicación de la suspensión de la vía oral se basa en el dolor y la intolerancia digestiva. No debe ser prolongada y se debe restablecer secuencialmente luego de 48 horas sin dolor. La alimentación parenteral no tiene ningún sentido si el restablecimiento de la vía oral se realiza dentro de los primeros 7 días. De no ser así se sabe que esta patología grave provoca una agresión severa que determina un estado hípercatabólico por lo tanto debe implementarse soporte nutricional para evitar la desnutrición y las complicaciones que ella trae consigo (alteración de la modulación de la respuesta inflamatoria, translocación bacteriana, inmuno supresión, etc.) El 60 % de esta enfermedad presenta hipermetabolismo (1,5 veces el metabolismo basal), el 40 % son normo o hipometabólicos. Según la Conferencia de Consenso de Nutrición de la Agresión (Francia 1998), los triglicéridos no son contraindicados a menos que el paciente presente una hipertrigliceridemia importante. La necesidad de aporte debe ser de 0,25 a 0,30 g/Kg. La suplementación de micronutrientes fundamentalmente antioxidantes (vitaminas A, C, E y selenio) y zinc están indicadas. La suplementación en base a glutamina, inmunomoduladores, nuevas emulsiones lipídicas en base a aceite de oliva necesitan aun ser confirmadas para la pancreatitis severa. La vía enteral se debe privilegiar ya que no solo es mas fisiológica sino que también presenta menor costo y complicaciones habiendo actualmente estudios que muestran un menor índice de morbimortalidad en estos pacientes ya que la integridad de la barrera intestinal limitaría la sobreinfección bacteriana pero fundamentalmente la fúngica ( Kalfarentzos 97, Windsor 98, Pupelis 2000) Referencias Banks PA, Freeman ML; Practice Parameters Committee of the American College of Gastroenterology. Practice guidelines in acute pancreatitis. Am J Gastroenterol . 2006 Oct;101(10):2379-400. Frossard JL, Steer ML, Pastor CM. Acute pancreatitis. Lancet . 2008;371:143-152.

28.- El diagnóstico para un hombre de 44 años que ha presentado un cuadro de diarrea con una duración de más de dos semanas, pero que no se extiende por más allá de cuatro, es: a) b) c) d)

Diarrea aguda Diarrea persistente Diarrea crónica Diarrea acuosa

La clasificación de la diarrea de acuerdo al tiempo de duración de este síntoma la divide en: aguda, con duración de menos de 14 días, crónica con duración de más de 4 semanas, y persistente ha la que tiene una duración menor del mes, y mayor a las dos semanas.

Kasper DL, Braunwald E, Fauci AS, Hauser SL, Longo DL, Jameson JL. Harrison´s Principles of Internal Medicine. McGraw Hill. 16 Ed. 225 p.

29.- Masculino de 2 años y medio años de edad, diagnosticado con sífilis congénita tardía, una de las manifestaciones de esta infección es: a) Hepatomegalia b) Rinorrea c) Queratitis e) Pseudoparálisis de Parrot La sífilis congénita tardía se manifiesta después de los 2 años de vida, el síntoma más frecuente es la queratitis, que es parte de la tríada de Hutchinson (queratitis, hipocausia y dientes en tonel o Hutchinson), así como deformidades óseas (tibias en sable, frente olímpica, engrosamiento clavicular), rodilla de clutton, paresia juvenil y tabes dorsal. La hepatomegalia, rinorrea, roseola sifilítica y la pseudoparálisis de parrot son manifestaciones de la sífilis precoz que se manifiesta en los primeros dos años de vida. Evans HE, Frenkel LD: Congenital Syphilis. Clin Honeyman: Manual de Dermatología, 2da ed. 1988; pg. 46.

Perinatol

1994;

21(1)

Lukehart SA, Holmes KK: Sífilis. en: Principios de Medicina Interna. 12 ed. Ed. Harrison. Cap. 128, pg. 765.

30.- A 55-year-old female is brought to the emergency room with a sudden onset of severe chest, back, and abdominal pain that began 20 minutes ago. The pain is described as being very sharp with a tearing-like character. Her previous medical history is notable for hypertension treated with hydrochlorothiazide. On examination, her blood pressure is 145/95 mm Hg in the right arm and 119/75 mm Hg in the left arm; pulse, 105 beats/minute; temperature, 37.3°C.; and respirations, 17 breaths/minute. A chest X-ray (CXR) demonstrates a widened mediastinum, and electrocardiography (ECG) reveals nonspecific ST and T wave changes. What is the most appropriate next step in the management of this patient? a) b) c) d)

Emergent surgery Thoracic computed tomography Aortography Intravenous labetalol

DISECCIÓN DE AORTA: Los predictores clínicos de mayor sensibilidad para el diagnóstico son: dolor torácico súbito e intenso, ausencia o diferencia de pulsos y ensanchamiento mediastinal. •

El manejo médico está dirigido a disminuir la presión arterial media y de manera muy importante, a disminuir el aumento de la presión pulsátil que golpea directamente la aorta (dp/dt). •

Las medidas a tomar para este objetivo incluyen reposo absoluto, control de las cifras de presión arterial, administración de oxígeno y monitorización completa no invasiva. •

Los objetivos iniciales de reducción de las cifras de tensión sistólica deben alcanzar los 100-120 mm Hg o menos, manteniendo la perfusión a los órganos vitales. •

Los medicamentos antihipertensivos de elección son los beta bloqueadores IV, combinados con nitroprusiato de sodio. Una terapia alternativa a esta combinación, es el uso de labetalol que tiene efecto bloqueador alfa y beta. •

Khan IA, Chandra N. Clinical, Diagnostic, and management perspectives of aortic dissection. Chest 2002; 122: 311-328. •

31.- Masculino de 42 años de edad con diagnóstico de herpes genital activo y recurrente, presenta de manera súbita numerosas máculas eritematosas, pápulas, vesículas. Las manos, pies y cara están extensamente afectadas. A la exploración física minuciosa demuestra que muchas de las lesiones tienen una aparencia de “tiro al blanco”. Algunas afectan labios y mucosa oral. El diagnóstico más probable es: a) b) c) d)

Eritema multiforme Granuloma anular Pénfigo Necrolisis tóxico epidérmica

El eritema multiforme es una enfermedad de la piel y las mucosas que se manifiesta con lesiones eritematosas y de tipo vesículo-ampollar. Las lesiones Vesículo-ampollares y erosivas a nivel de la cavidad oral y la piel pueden ser causadas por un amplio grupo de patologías. La etiología de las mismas también, puede ser muy variable, desde una causa traumática o química por contacto, hasta una causa autoinmune.

Las lesiones producidas por el eritema multiforme son circulares y pueden presentarse como anillos concéntricos (se las conoce como lesiones de tiro al blanco). Estas lesiones también pueden asociarse con otras condiciones médicas tales como infección por herpes, infección estreptocócica, tuberculosis (TB), o como una reacción a sustancias químicas o medicamentos.

Gavaldá-Esteve C, Murillo-Cortés J, Poveda-Roda R. Eritema multiforme. Revisión y puesta el día RCOE, 2004, Vol 9, Nº4, 415-423 32.- Femenino de 43 años portadora de DM tipo II, e HTAS, es ingresada al servicio de Medicina Interna por cetoacidosis. Posterior a su recuperación metabólica inicia con fiebre, cefalea, dolor facial, disminución del nivel de conciencia y enrojecimiento nasal con lesión negruzca en fosa nasal derecha. El diagnóstico más probable en ésta paciente es: a) b) c) d)

Endocarditis por S. aureus. Carcinoma epidermoide. Infección por M. tuberculosis. Infección por Mucor.

Mucormicosis es el nombre común dado a varias diferentes enfermedades causadas por hongos de la orden de los Mucorales. Muchas diferentes especies han sido implicadas como agentes de síndromes clínicos similares.

Los Mucoraceos son hongos que se encuentran en todas partes y son comunes habitantes de materia en descomposición. Por ejemplo, Rhizopus sp. Frecuentemente puede ser recuperado de pan mohoso. Por su rápido crecimiento y prolífica capacidad de formar esporas, inhalación de conidias debe ser una experiencia cotidiana. La presencia de esporas Mucorales en cinta adhesiva no estéril se demostró fue la fuente de mucormicosis cutánea primaria. Aun cuando estos hongos crecen en muchos nichos ecológicos, la infrecuencia de enfermedad debida a estos organismos da fe de su baja potencial virulencia en el huésped humano. En contraste a la amplia distribución de este hongo, la enfermedad en humanos esta limitada, en muchos casos a población con severo inmunocompromiso, diabetes mellitus o trauma.

Manifestaciones clínicas En 1973 Meyer  y Armstrong 12 categorizaron las diferentes presentaciones clínicas de mucormicosis, considerando el órgano involucrado y haciendo una división en seis entidades: rinocerebral, cutánea, gastrointestinal, pulmonar, diseminada y formas misceláneas. Existe una fuerte asociación entre la entidad subyacente y la forma de presentación. La mucormicosis rinocerebral se presenta más frecuentemente en pacientes diabéticos con acidosis, y debido al advenimiento de quimioterapias cada vez más potentes, se aprecia con más frecuencia en pacientes leucémicos con neutropenia prolongada, en aquellos que reciben múltiples esquemas de antibióticos y corticoesteroides, así como en pacientes con trasplantes de órganos. En pocas ocasiones se ha documentado esta forma invasora en personas sin enfermedades subyacentes. La forma pulmonar puede presentarse en pacientes con leucemia y neutropenia; la mucormicosis gastrointestinal se observa más comúnmente en pacientes con desnutrición calórico-proteica y en prematuros, y la diseminada en pacientes con déficit inmunológico grave como trasplantados, leucémicos y nefrópatas tratados con deferoxamina. 1,2 La infección rinocerebral es la presentación más frecuente y característica de mucormicosis, siendo la rinoorbitaria y la mucormicosis paranasal estadios tempranos de esta. La infección generalmente inicia en senos paranasales o paladar duro, y se extende a senos adyacentes con diseminación a través de senos etmoidales y zona retroorbitaria; puede tener acceso al cerebro a través del ápex orbitario, lámina cribosa y por vía vascular. Una costra necrótica sangrante en paladar o en mucosa nasal y un drenaje ocular de pus negruzco orientan al diagnóstico. Puede haber una progresión rápida y presentarse el deceso en pocos días o ser indolente si la enfermedad subyacente se logra controlar. Inicialmente puede haber dolor facial, cefalea, fiebre y algún grado de celulitis orbitaria, conforme progresa la invasión de la órbita la pérdida de la función del II, III, IV y VI nervios craneales puede ocurrir, así como también puede haber pérdida de la función de músculos extraoculares, proptosis, quemosis progresiva, congestión nasal, epistaxis y

letargia. La disfunción de nervios craneales, especialmente el V y VII, ocurre de manera tardía manifestando ptosis y midriasis, lo cual es un factor pronóstico grave, en caso de trombosis retiniana, hay pérdida de la visión y puede haber afección intraocular. El compromiso cerebral es la complicación más seria y puede manifestarse como: infarto, absceso, trombosis del seno cavernoso, hematoma subdural y necrosis del lóbulo frontal. 2 Otras complicaciones son trombosis de arteria carótida interna y vena yugular, e incluso se ha reportado infarto de miocardio por oclusión de coronaria y aborto séptico. Puede haber presentaciones crónicas y secuelas tardías a pesar del tratamiento aparentemente satisfactorio por lo cual siempre debe realizarse un seguimiento del paciente a largo plazo BIBLIOGRAFIA: • Mayo Clinic. Pulmonary Diseases • Mandele?Douglas. Infections Diseases • Pennington. Respiratory infections,

Diagnosis and Management

• Ronald B. george., Richard W. Light, Michael A. Matthay.

Chest Medicine 3era. edición.

• Scott E Davis. Neumonía Micótica. Clínicas médicas de Norteamérica 1997; 5: 1092?1094.

33.- Paciente masculino de año y medio de edad, refieren los padres que desde que inició a caminar aproximadamente a los 15 meses se ha presentado con cojera. Al realizar la exploración llama la atención que presenta marcha en Trendelenburg. Su sospecha principal de diagnóstico es: a) Artritis séptica de cadera. . b) Enfermedad neurológica que afecta al desarrollo psicomotor. c) Luxación congénita de cadera. d) Enfermedad de Perthes.

La luxación congénita de cadera (L.C.C.) llamada también Enfermedad Luxante de la Cadera, es la malformación ortopédica de gravedad más frecuente del ser humano. Corresponde a una displasia articular que se produce por una perturbación en el desarrollo de la cadera en su etapa intrauterina antes del tercer mes de vida fetal. La sub-luxación o la luxación se produce después del nacimiento, en los primeros meses de vida extra-uterina  y como una consecuencia de la displasia. No hay que confundirla con la verdadera luxación intrauterina de la cadera y que corresponde a una afección teratológica y que se acompaña habitualmente con otras alteraciones congénitas cardíacas, renales, de la columna vertebral, de los pies, etc. Esta luxación intra-uterina es de muy difícil tratamiento, de mal pronóstico y alcanza no más del 5% del total de las luxaciones congénitas de la cadera. La L.C.C. es progresiva. Se nace con displasia y, si no es adecuadamente tratada, progresa a subluxación y luxación, representando el 95% de las L.C.C. (el otro 5% son las luxaciones teratológicas ya mencionadas). Por esta razón, el diagnóstico precoz (primer mes de vida) es de extraordinaria importancia, ya que tratada en este momento se logran caderas clínica, anatómica y radiológicamente normales. LUXACIÓN En este período la sintomatología es más llamativa. Cuadro Clínico Hay retardo en la iniciación de la marcha, que puede ser claudicante, describiéndose el signo de Trendelemburg, marcha de pato en la luxación bilateral. La niña es incapaz de realizar la abducción total de la cadera o caderas. Puede observarse asimetría de pliegues. Acortamiento aparente del miembro inferior afectado. Contractura de aductores que limita la abducción de la cadera a 20° o menos (Normal 45° o más). Contractura del psoasiliaco. El trocánter mayor es anormalmente prominente encontrándose por encima de la línea de Nelaton Roser (línea que se extiende desde la espina iliaca anterosuperior hasta la tuberosidad del isquion). La rotación interna está aumentada a 60° o más debido al aumento de la anteversión por encima de 25°. El signo de Trendelemburg se da por debilidad del glúteo medio al acercarse sus inserciones pelvitrocantéricas; cuando la paciente se sostiene sobre una pierna, la imposibilidad de los abductores para elevar la pelvis del lado opuesto cayendo ésta, produce el signo positivo.

Signo del pistón (Dupuytren). Radiología Para el estudio de los cuadrantes de Ombredanne, el arco de Shenton. La anteversión del cuello. Hipoplasia del cótilo y de la cabeza femoral. La resonancia magnética para el estudio de la cápsula y sus alteraciones: Deformidad en reloj de arena, adherencias. El limbo, etc.

Cirugía: II cirugía ortopédica y traumatología Autor: Universidad Nacional Mayor de San Marcos (Lima). Facultad de Medicina. Escuela Académico Profesional de Medicina Humana. Departamento Académico de Cirugía Publicación: Lima: UNMSM, 2000 Descripción: 407 Serie: (Cirugía; 2) ISBN: 9972-46-102-5 Otros autores: Salaverry García, Oswaldo, 1959-, ed. Tema: Traumatología; Ortopedia

34.- Femenino de 55 años de edad, que acude a consulta externa refiere rubor, tumefacción y rigidez de las articulaciones interfalángicas dístales desde hace tres meses, sin presencia de otras molestias articulares. ¿Cuál de los siguientes diagnósticos es más probable? a) b) c) d)

Osteoartritis erosiva Artritis reumatoide Espondilitis anquilosante Esclerodermia

Allen R. M. MMS Medicina Interna. 5ª. Edición. National Medical Series. Mc. Graw Hill. 2006. (capítulo 10 V E 1). Típicamente, la osteoartritis erosiva afecta las articulaciones interfalángicas dístales en mujeres de edad madura. Es improbable que esos síntomas articulares dístales prominentes sucedan en pacientes con artritis reumatoide o con lupus eritematoso diseminado sin molestias articulares más generalizadas. No hay pruebas que indiquen espondilitis anquilosante o esclerodermia.

35.- Masculino de 50 años con dolor precordial relacionado al esfuerzo, de corta duración, de 4 meses de evolución y con ECG en reposo normal, el siguiente estudio de elección es: a) b) c) d)

Ecocardiograma de reposo Prueba de Talio- Dipiridamol Ecocardiograma con estrés farmacológico Prueba de esfuerzo con protocolo de Bruce

El propósito fundamental de la prueba es el de demostrar la existencia de isquemia miocárdica en los subgrupos de población con mayor prevalencia de cardiopatía isquémica o bien en los subgrupos en donde la prueba se efectúa a manera de evaluación del tratamiento en pacientes ya conocidos con cardiopatía isquémica. Existe otro subgrupo de población en quienes este estudio ayuda a determinar la clase funcional en la que se encuentran e incluye a pacientes con valvulopatías o con insuficiencia cardiaca de cualquier etiología. Protocolos de esfuerzo. El protocolo más empleado es el de Bruce sobre treadmill, aunque existen otros protocolos  y su elección dependerá de las condiciones del individuo. Los protocolos discontinuos  son los que alternan periodos de esfuerzo que se intercalan con periodos de reposo de duración similar, se emplean en escasas circunstancias.

Los  protocolos continuos  son los que no interrumpen el esfuerzo una vez iniciado hasta finalizada la prueba, permiten mejor adaptación física y psicológica y es posible adaptar la intensidad de forma individualizada para que la prueba tenga una duración de 6 a 12 minutos. Los protocolos máximos  son los que se suspenden debido a la sintomatología del paciente, a los signos registrados durante la prueba o se alcanzan valores máximos de FC y VO 2. Los  protocolos submáximos  son los que se suspenden cuando el sujeto alcanza un nivel determinado de carga, habitualmente el 85% de la FC máxima teórica (que se encuentra entorno a los 170 lpm). En la práctica diaria, el nivel de carga (VO 2) se expresa en forma de trabajo externo (MET  ó equivalentes metabólicos) que corresponden a 3,5ml/kg/min de VO 2, lo que permite comparar protocolos entre sí (cada protocolo dispone de fórmulas para realizar el cálculo de los METS), el error que cometen en el cálculo de los METS es mayor en protocolos discontinuos. Emplear la FC como único criterio para determinar el esfuerzo máximo es erróneo, por lo que deberían tenerse en cuenta otros criterios, como es la percepción subjetiva por parte del paciente mediante la escala de Borg (tabla I). Esta dificultad en la predicción del esfuerzo máximo es lo que limita la realización de pruebas submáximas a la determinación de la condición física de sujetos aparentemente sanos. Tabla I. Escala de Percepción del esfuerzo de Borg. Escala de 15 grados Escala de 10 grados Valor Percepción Valor Percepción 6 No se siente nada 0 Nada 7 0,5 Muy muy leve Muy muy leve 8 1 Muy leve 9 Muy leve 2 Leve 10 11 Considerablemente leve 3 Moderada 12 4 Algo fuerte Moderadamente dura 13 5 Fuerte o intensa 14 6 Dura Muy fuerte 15 7 16 8 Muy dura 17 9 Muy muy fuerte (submáxima) 18 Muy muy dura 10 19 20 Esfuerzo máximo * A la izquierda la escala original de esfuerzo percibido en 15 grados (de 6 a 20) y a la derecha la más nueva de 10 categorías.

Tabla II. Indicaciones clásicas de la ergometría. I. Fines diagnósticos A. Pacientes sintomáticos. 1. Dolor torácico: a) Típico b) Atípico 2. Clínica de equivalentes isquémicos. A. Pacientes asintomáticos. 1. Con alteraciones en el ECG sugestivas de isquemia. 2. Con alta probabilidad de padecer Cardiopatía Isquémica (paciente con múltiples factores de riesgo) 3. Cuando convenga descartar con cierta seguridad CI. 4. Con sospecha de CI silente. 5. Sedentarios que inician programa de actividad física. 6. Para estudio funcional de ciertas arritmias. II. Con fines valorativos y pronósticos. 1. Seguimiento de paciente con CI conocida. 2. Tras IAM. 3. En exámenes prelaborales o laborales. 4. De la eficacia del tratamiento: Médico. Cateterismo y angioplastia. Quirúrgico. 5. Respuesta de la Tensión Arterial. 6. En valvulopatías o miocardiopatías. 7. Estudio de arritmias y trastornos de la conducción aurículo-ventricular. 8. En cardiopatías congénitas. Bibliografía 1.

Gibbons RJ (Edit.). ACC/AHA 2002 Guideline Update of Exercise Testing. 2002 American College of Cardiology Foundation and American Heart Association ACC/AHA; 2002 [Acceso 1-4-06]. Disponible en: 2. Guidelines for cardiac exercise testing. ESC Working Group on Exercise Physiology, Physiopathology and Electrocardiography Eur Heart J 1993; 14: 969-988. 3. Fernando Arós Aros F, Boraita A, Alegria E, Alonso AM, Bardaji A, Lamiel R el al. Guías de práctica clínica de la Sociedad Española de Cardiología en pruebas de esfuerzo. Rev Esp Cardiol 2000; 53 (8): 1063-94 4. Chaitman B. Las pruebas de esfuerzo. En: Braunwald E, editor. Tratado de Cardiología. Medicina Cardiovascular. 4ª ed. Madrid Mc-Graw-Hill-Interamericana de España; 1993. p. 177-197.

5. 6. 7. 8. 9. 10. 11. 12. 13.

Schlant RC, Friesinger GC 2nd, Leonard JJ. Clinical competence in exercise testing: a statement for physicians from the ACP/ACC/AHA Task Force on Clinical Privileges in Cardiology. J Am Coll Cardiol 1990; 16: 1061-5 Reyes Lopez de los M, Iñiguez Romo A, Goicolea de Oro A, Funes Lopez B, Castro Beiras A. El consentimiento informado en cardiología. Rev Esp Cardiol 1998; 51: 782796. Fletcher GF, Flipse T, Malouf J, Kligfield P. Current status of ECG stress testing. Curr Probl Cardiol. 1998 Jul; 23(7): 353-423. Alegría Ezquerra E, Alijarde Guimerá M, Cordo Mollar JC, Chorro Gascó FJ, Pajarón López A. Utilidad de la prueba de esfuerzo y de otros métodos basados en el electrocardiograma en la cardiopatía isquémica crónica. Rev Esp Cardiol 1997; 50: 6-14 Wasserman K, Hansen JE, Sue DY, Whipp BJ, Casaburi R. Principles of exercise testing and interpretation . 2ª ed. Philadelphia: Lea & Febiger; 1994. p. 95-111. American college of Sports Medicine. Guideliness for exercise testing and prescription. 5ª ed. Baltimore: Williams & Wilkins; 1995. Borg GA. Psychophysical bases of perceived exertion. Med Sci Sports Exerc 1982; 14: 377-381. Froelicher VF, Umann TM. Exercise testing: clinical applications. En: Pollock ML, Schmidt DH, editors. Heart disease and rehabilitation . 3ª ed. Champaign, IL: Human Kinetics, 1995; p.57-79. Myers J, Froelicher VF. Exercise testing. Procedures and implementation. Cardiol Clin. 1993; 11(2): 199-213.

Weiner DA, McCabe C, Hueter DC, Ryan TJ, Hood WB Jr. The predictive value of anginal chest pain as an indicator of coronary disease during exercise testing. Am Heart J 1978; 96: 458-462. 36.- Masculino de 30 años de edad adicto a las drogas intravenosas presenta debilidad del hemicuerpo derecho y cefalea en un periodo de dos días. La exploración revela un individuo mal nutrido y afebril con hemiparesia derecha leve. ¿Cuál de los siguientes es el diagnóstico más probable? a) b) c) d)

Absceso cerebral Endocarditis bacteriana Meningitis por virus de la inmunodeficiencia humana (VIH) Meningitis criptocócica.

Los adictos a drogas intravenosas están propensos a sufrir bacteriemia, que a su vez puede producir absceso cerebral y disfunción neurológica progresiva. Los pacientes con aquél, por lo común se encuentran afebriles a no ser que haya endocarditis acompañante u otro origen endovascular de infección. Los adictos a drogas intravenosas tienden a presentar endocarditis bacteriana y pueden cursar con déficit neurológicos en forma apoplética debido a embolia séptica del cerebro. Sin embargo, por lo regular tienen fiebre. La meningitis por VIH produce cefalea y datos de irritación meníngea, pero no se presenta déficit neurológico focal. La meningitis criptocócica se manifiesta con conducta alterada y cefalea y los pacientes están afebriles. Sin embargo, es raro que haya datos de seudoapoplejía. Por último, el uso de drogas intravenosas puede provocar embolia de cuerpo extraño pero con problemas neurológicos apopléticos. Un émbolo puede llegar al cerebro por un cortocircuito cardíaco de derecha a izquierda o bien por una malformación arteriovenosa pulmonar si la inyección es venosa. El émbolo puede entrar a la circulación cerebral de manera directa en caso de inyección intracarotídea. Referencias: Nath A. Brain abscess and parameningeal infections. In: Goldman L, Ausiello D, eds. Cecil  Medicine . 23rd ed. Philadelphia, Pa: Saunders Elsevier; 2007: chap 438 Allen R. M. MMS Medicina Interna. 5ª. Edición. National Medical Series. Mc. Graw Hill. 2006. (Capítulo 11 XVI B 1-2). 37.- Paciente de 52 años de edad con diagnóstico de neumonía adquirida en la comunidad, quien recibió tratamiento antibiótico a base de Ceftriaxona. No tuvo respuesta adecuada, y en el estudio diagnóstico para determinar la causa, se encontró un derrame pleural del 60%, el cual se puncionó. En el estudio citoquímico y bacteriológico de este líquido, se reportan cocos grampositivos en la tinción de Gram, y un pH de 7. Con esto, usted considera como indispensable: a) b) c) d)

Iniciar cobertura con vancomicina. Colocación de sonda endopleural. Intubación orotraqueal y ventilación con volúmenes altos. Ingreso a Unidad de Terapia Intensiva.

Los hallazgos del citoquímico y tinción son característicos de empiema. El drenaje del mismo es la maniobra terapéutica base para el tratamiento y resolución del mismo. Kasper DL, Braunwald E, Fauci AS, Hauser SL, Longo DL, Jameson JL. Harrison´s Principles of Internal Medicine. McGraw Hill. 16 Ed. 1536 p.

38.- Un estudiante universitario de 20 años de edad acude a consulta debido a tos seca, fiebre, cefalea y dolor muscular durante las últimas 2 semanas. Refiere que sus compañeros de casa han desarrollado síntomas similares. Niega el uso de drogas ilícitas y no es homosexual. Su temperatura es de 38.2°C, FC 90lpm, FR 18x’. Se auscultan murmullo vesicular bilateral. Una RX de tórax muestra opacidades intersticiales multifocales. Presenta leucocitosis y un test de aglutinina frío +. ¿Cual de los siguientes es el patógeno más probable? a) b) c) d)

Bacterias anaeróbicas Mycoplasma pneumoniae Pneumocystis carinii Streptococcus pneumoniae

En el caso antes mencionado, se trata de una neumonía llamada atípica que se presenta por norma en pacientes jóvenes y que han estado en lugares de convivencia con otras personas, la imagen radiográfica de esta neumonía es de una neuropatía intersticial como se indica. 1.- Fishman AP, , Fishman JA, Grippi MA, Kaisser LR, Señor RM. Pulmonary Diseases and disorder. 3a. Edición McGraw-Hill, EUA, 2006. 2.- Fraser, R ; Neil, C; Parè, P; Diseases of the Chest, Third Edition, Editorial Elsevier, 2005. 3.- Murray and Nadels; Textbook Respiratory Medicine, Vol 1-2, Elsevier editorial, 2005. 39.- Masculino de 5 años de edad, con antecedente de neumonía por Pneumocystis, la profilaxis de ésta patología en éste paciente, se encuentra indicada cuando las cifras de linfocitos ajustados son de: a) b) c) d)

< 1500 Células/µl < 750 Células/µl < 500 Células/µl < 200 Células/µl

La NPC en niños y en el embarazo La NPC es infección diagnóstica de SIDA en un alto porcentaje de niños, sobre todo en el primer año de vida. Los niños menores de 1 año con CD4+ por debajo de 1500// l t ienen un 90% de riesgo de padecerla. Las manifestaciones clínicas, el diagnóstico y el tratamiento no difieren de los del adulto. Para la prevención deben seguirse las siguientes recomendaciones:













Se aconseja la profilaxis primaria en todos los niños menores de 1 año hijos de madre VIH+ La profilaxis debería comenzar a las 4-6 semanas de vida y suspenderse en caso de que posteriormente no resulten estar infectados. Los niños infectados y aquellos cuyo estado de infección por VIH se desconozca deberán seguir recibiendo tratamiento profiláctico durante el primer año de vida. En los mayores de esta edad se considerará continuar con la profilaxis atendiendo al recuento de linfocitos CD4+ según la edad: de 1 a 5 años, cuando el recuento sea < 500 células/l o el por cent aj e < 15%; en niños de 6 a 12 años, cuando el r ecuent o sea inferior a 200 células/ l o el por cent aj e < 15%. No se ha estudiado la seguridad de interrumpir la profilaxis en niños infectados con el VIH y sometidos a tratamiento antirretroviral. Los niños con un historial que incluya episodios de NPC deberán ser tratados de por vida con quimioprofilaxis para evitar casos de recurrencia.

NCP en el embarazo Como ya se ha dicho, la quimioprofilaxis anti-NPC debe administrarse a las mujeres embarazadas al igual que al resto de adultos y adolescentes. En este caso, el agente profiláctico recomendado sigue siendo el cotrimoxazol, con dapsona como alternativa. Debido a la posibilidad teórica de una posible teratogenicidad asociada a la exposición a los fármacos durante el primer trimestre de gestación, durante dicho período puede considerarse la alternativa de pentamidina en aerosol debido a que dicho agente no se absorbe sistémicamente por lo que el feto en desarrollo no sufre exposición al fármaco. Lectura recomendada: Neumonía por Pneumocystis carinii en niños infectados por el virus de inmunodeficiencia humana (VIH). Gac Med Mex 2004; 140 (1): 59-70 40.- Masculino de 28 años acude a su consultorio con reporte de exámenes de laboratorio con los siguientes resultados. Hemolisinas bifásicas + (o anticuerpos de DonathLandsteiner) productoras de hemoglobinuria paroxística a frigore. La Entidad responsable de los resultados de este paciente es: a) b) c) d)

Sífilis. Leucemia linfática crónica. Mieloma múltiple. Lupus eritematoso diseminado.

Razones por las que se realiza el examen Este examen se realiza algunas veces cuando el médico sospecha de un diagnóstico de criohemoglobinuria paroxística. Valores normales La ausencia de anticuerpos es lo normal. Significado de los resultados anormales Los resultados anormales indican la presencia de criohemoglobinuria paroxística (PCH), un trastorno que ocurre cuando la exposición a bajas temperaturas hace que el sistema inmunitario produzca anticuerpos que destruyen los glóbulos rojos. Estos anticuerpos se denominan anticuerpos de Donath-Landsteiner. A medida que las células son destruidas, la parte de los glóbulos rojos (hemoglobina) que transporta el oxígeno es eliminada en la orina. La criohemoglobinuria paroxística es un síndrome adquirido. Algunas veces, los anticuerpos se presentan con una infección viral (por ejemplo, sarampión  y paperas) o con sífilis; sin embargo, en algunos casos, el trastorno no está relacionado con una enfermedad y la causa se desconoce. K. Holmes, P. Mardh, P. Sparling et al (eds). Sexually Transmitted Diseases, 3rd Edition. New York: McGraw-Hill, 1999, chapters 33-37.

41.- Femenino de 28 años G5 P2 A3 tiene una historia de abuso de sustancias prenatal. A las 37 sdg tiene una parto vaginal obteniéndose una neonato pequeño para la edad gestacional, de género masculino con péqueñas aperturas palpebrales, pliegues epicánticos, con cara aplanada, filtrum hipoplásico y borde del vermillon delgado. Estos hallazgos son característicos en neonatos cuyas madres tuvieron abuso prenatal de cual de las siguientes sustancias: a) b) c) d)

Tabaquismo Marihuana Alcohol Narcóticos

EFECTOS DEL ALCOHOL SOBRE EL RECIEN NACIDO 1 - Efectos somáticos. En 1967 Lemoine y colaboradores describieron las anomalías observadas en hijos de madres alcohólicas; posteriormente, en 1973, Jones, Smith y colaboradores las denominaron síndrome alcohólico fetal (SAF). Las principales características observadas en los niños con

SAF son las siguientes: En el 50 a 80% hubo retraso del crecimiento intrauterino, microcefalia, apertura palpebral estrecha, nariz corta y respingona, mandíbula hipoplásica y labio superior fino. También se ha asociado a cardiopatías congénitas, anomalías en extremidades y en la columna vertebral. BIBLIOGRAFIA: •





Bell GL, Lau K. Problemas perinatales y neonatales por abuso de sustancias. Clin Pediatr Nort America 1995, 2:247-266. Byrd RS, Howard CR. Children´s passive and prenatal exposure to cigarrette smoke. Pediatr Annals 1995: 24(12): 644-645 Cruz M, Bosch J. Síndromes Pediatricos. Barcelona, Espaxs Publicaciones Médicas1998, 534-535

Eyler FD, Behnke M. Desarrollo temprano en lactantes con exposición a drogas. Clin Perinatol 1999; 1: 105-149.

42.- Femenino de 28 años de edad, acude a consulta prenatal de rutina G5 P4, con 28 SDG, refiere que no ha sentido movimiento fetal durante los últimos 2 días. Su embarazo ha sido complicado debido a que padece hipertensión crónica, para lo cual le indicaron tabletas de alfa-metildopa 2 veces al día. Al examen, su FU es de 30cm, y las maniobras de Leopold demuestran que el feto se encuentra en situación transversa. Su TA es 145/85mmHg. No se encuentra latido cardiaco con el Doppler. ¿Cuál de los siguientes es el paso más apropiado a seguir en el manejo? a) b) c) d)

Realizar un test sin estrés Amniocentesis USG Beta-HCG (cuantitativa)

Probable óbito: Sintomatología y diagnóstico Signos funcionales : No se perciben movimientos fetales por 12-24 horas. Disminución o ausencia de síntomas y/o signos como nauseas vómito, hipertensión, albuminuria) Paraclínicos:  *ecografía: diagnóstico precoz y exacto: Doppler. *Radiología: hay 3 signos: +deformación del cráneo +curvatura y torsión de la columna +presencia de gas en el feto *líquido amniótico: puede estar meconiado, o sanguinolento Signos locales:  en los senos hay secreción calostral, sangrado leve y oscuro por vagina, el feto se vuelve blando a la palpación, fetocardia (-), puede haber detención y/o disminución de la altura uterina, bajo peso corporal, entre otros.

Bibliografía: 1. OBSTETRICIA, Schwarcz R, editorial El ateneo, 2003. 2. Sociedad española de ginecología y obstetricia, junio 2002. 3. OBSTETRICIA CLÍNICA, Llaca V, edición 2000, capítulo 24; Pág, 315-316.

43.- Masculino de 4 años llevado a consultar por presentar exantema máculopapular de inicio en cara, posteriormente en tronco, extremidades y nalgas, con adenopatía cervical y retroauricular, refiere la madre que ha presentado fiebre no cuantificada y catarro ¿El diagnóstico más probable de este paciente es? : a) Varicela. b) Rubéola c) Sarampión. d) Exantema súbito La también llamada Tercera enfermedad, era conocida antiguamente como Sarampión alemán y considerada como una variante del Sarampión o la Escarlatina. También se la conoce como Sarampión rojo. En 1914 se teorizó sobre su origen viral y recién en 1938 lo confirmaron Hiro y Tosaka. En 1940, durante una epidemia en Australia, un oftalmólogo reportó 78 casos de cataratas congénitas en bebés nacidos de madres infectadas en el primer trimestre del embarazo: fue el primer dato reconocido del Sindrome de Rubeola Congénita (SRC). Agente etiológico: Es un rubivirus (RNA) de la familia togavirus, aislado en 1962 por Parkman y Weller. Se clasifica de dos maneras: 1) Post-natal: Es una enfermedad infectocontagiosa, común en la infancia y juventud. Da síntomas generales leves: exantema máculopapular difuso, estado febril y adenopatías suboccipitales, postauriculares y cervicales posteriores (que son características de esta patología, antecediéndola entre 5 a 10 días). El 25 a 50% cursa en forma subclínica. En niños mayores y adultos (mujeres fundamentalmente), puede dar poliartralgias. Aunque en general no deja secuelas, puede complicarse con encefalitis o trombocitopenia.

Esta forma se transmite por contacto directo o por gotitas de Pflügge. Predomina a fines del invierno y comienzos de primavera. En comunidades pequeñas o cerradas, durante un brote suelen padecerla todos los susceptibles. El período de incubación varía entre 14 y 21 días. El contagio se produciría desde pocos días antes del exantema y hasta 7 días después del inicio del mismo (aunque estudios hechos en voluntarios, mostraron presencia del virus rubeola en secreciones nasofaríngeas desde 7 días antes y hasta 14 días después).

Manifestaciones clínicas: es una enfermedad infecciosa leve que evoluciona de manera subclínica u oligosintomática en alrededor del 50% de los niños pequeños. Los síntomas prodrómicos, como aumento de volumen doloroso (adenomegalia) en ganglios retroauriculares, cervicales posteriores y occipitales son más frecuentes entre los adolescentes y adultos jóvenes. El exantema puede ser también el signo inicial, con máculas  y pápulas rosadas pálidas que aparecen inicialmente en la cara y cuello y se generalizan en 2 a 3 días. El exantema puede ser intensamente eritematoso y descamarse finamente. Los síntomas generales, como fiebre, cefalea, mialgias, artralgias y ocasionalmente artritis, son más frecuentes entre las mujeres adolescentes y adultos jóvenes. Las articulaciones más afectadas son las muñecas y las interfalángicas.

2) Congénita: Más del 25% de los fetos cuya madre padeció rubeola durante el primer trimestre del embarazo, son afectados. Puede producir muerte uterina o aborto espontáneo. Las patologías asociadas con esta enfermedad son: oftalmológicas (microoftalmía, cataratas, coriorretinitis), cardiológicas (conducto arterioso persistente, estenosis periférica de la arteria pulmonar, etc.), auditivas (sordera neurosensorial) y neurológicas (microcefalia, meningoencefalitis, retraso mental). Además presentan retardo del crecimiento, hepatoesplenomegalia, ictericia y lesiones similares a las purpúricas. Muchas de las rubeolas maternas pueden cursar en forma asintomática y los casos leves del feto, se diagnostican varios meses o años después del nacimiento (sordera parcial o manifestaciones cardíacas leves). Después de la vigésima semana de gestación, la infección fetal es casi nula. Los lactantes con SRC pueden eliminar el virus en sus secreciones y orina durante un año o más, afectando así a otras personas susceptibles. De todas maneras, esta susceptibilidad se ha mantenido antes y después de la vacuna, en un 10 a 20%.

Bibliografía: 1. Report of the Committee on Infectious Diseases, 25h edition, Red Book 2000. American Academy of Pediatrics. 2. Fifth (human parvovirus) and sixth (herpesvirus 6) diseases. Koch WC. Curr Opin Infect Dis 2001, Jun; 14 (3): 343-356. 3. Primary human herpesvirus 8 in immunocompetent children. Andreoni M, Sarmati L, Nicastri E, El Sawaf G, El Zalabani M, Uccella I, et al. JAMA 2002 Mar 13; 287 (10): 1295-300. Varicella vaccine update. AAP. Pediatrics 2000, Jan 105: 136-141.

44.- Se trata de masculino de 2 años con otitis media aguda recurrente quien recibió tratamiento con amoxicilina en el mes previo, el tratamiento de elección en éste paciente es: a) Amoxacilina+ clavulanato a 40 mgkgdía b) Amoxacilina+ clavulanato a 90 mgkgdía c) Azitromicina v.o 10mg kg día d) Ceftriaxona IM 50 mgKgdía

, el tratamiento inicial debería ser sintomático. Si a las 48-72 horas persiste o empeora la clínica se debe iniciar antibioterapia dirigida frente al S. pneumoniae , por su elevada frecuencia en nuestro medio y la baja tasa de curación espontánea. Se administrará amoxicilina a las dosis estándar recomendadas (40-50 mg/kg/día), ya que tienen una buena actividad frente a S. pneumoniae sensible o con resistencia intermedia a la penicilina. La duración del tratamiento antibiótico no está claramente establecida, aunque se ha observado que con 5-7 días se logra la resolución de la infección. Con una pauta corta se consigue, además, disminuir el riesgo de resistencias bacterianas, del número total de antibióticos pautados y del coste económico.

En caso de fracaso terapéutico, es decir, reaparición o persistencia de signos y síntomas de OMA, se debe administrar un antibiótico no sólo efectivo frente al S.  pneumoniae resistente a penicilina sino también frente a patógenos productores de betalactamasa: amoxicilina-clavulánico a dosis altas (80-90 mg/kg/dia de amoxicilina y 10 mg/kg/dia de ácido clavulánico). Hasta que no se disponga de formas comerciales adecuadas (relación amoxicilina/clavulánico= 8/1), esta dosificación se puede alcanzar mezclando amoxicilina y amoxicilina-clavulánico BIBLIOGRAFÍA 1. Rosenfeld RM. Método basado en la evidencia para tratar la otitis media. Clin Ped  Nort (ed esp) 1996; 6: 1075-1092. 2. Arnold JE. El oído. En: Behrman RE, Kliegman RM, Arvin AM, ed. Nelson. Tratado de  Pediatría.l5ª edición. McGraw-Hill-Interamericana de España, 1997; 2258-2270. 3. Pitkaranta A, Virolainen A, Jero J, Arruda E, Hayden F. Detección de infecciones por rinovirus, virus respiratorio sincitial y coronavirus en la otitis media aguda mediante la reacción en cadena de la polimerasa de transcriptasa inversa. Pediatrics (ed esp.) 1998; 6: 85-89. 4. Palva T, Pulkkinen K. Mastoiditis. J Laryngol Otol 1991; 105: 765-766.

45.- Femenino de 40 años de edad acude al servicio de urgencias quejándose de calambres en las piernas y parestesias en los dedos de las manos. Un año antes se le sometió a una operación del cuello, pero no está segura qué fue lo que se le hizo. El signo de Chevostek es positivo: la percusión sobre el nervio facial por delante de la oreja desencadena una contracción espasmódica del labio superior. ¿Cuál de los siguientes trastornos sugiere un signo positivo? a) b) c) d)

Hipercalciemia Hipocalciemia Acidosis Hiperpotasemia

SINTOMATOLOGÍA Neuromuscular: la hipocalcemia aguda se manifiesta por parestesia (hormigueo y adormecimiento de los dedos y región peribucal) y calambres o contracturas musculares. La sintomatología subclínica de tetania, evidenciada por el signo de Chvostek, (ocurrencia de espasmo facial, especialmente del orbicular de los labios, al percutir el nervio facial a mitad de distancia entre la comisura labial y el oído). El signo de Trousseau es un espasmo doloroso del carpo, que se presenta luego de mantener por tres minutos una presión >20 mmHg por encima de la sistólica, siendo un signo más de tetania. La hipocalcemia crónica se presenta con irritabilidad, confusión, demencia e incluso, en infantes, como retardo mental. También se reportan movimientos coreicos, distonías y

convulsiones. Se ha reportado calcificación de ganglios basales en la radiografía de cráneo, que no es reversible al tratamiento. Cardiovascular: prolongación de la fase de potencial de acción y por lo tanto prolongación del segmento ST en el ECG. En casos de severa deficiencia se presentan arritmias, hipotensión o falla cardiaca; la hipocalcemia aumenta la cardiotoxicidad de los digitálicos. Pulmonar: broncoespasmos y laringoespasmos vistos, sin embargo, con poca frecuencia. Dermatológica: piel seca, uñas quebradizas y caída del cabello como signos no específicos en la hipocalcemia crónica LECTURAS RECOMENDADAS: 1. Carlstedt F, Lind L. Hypocalcemic syndromes. Crit Care Clin 2001; 17:139-153. 2. Gibbs M, Wolfson A, Tayal V. Electrolyte disturbances. En: Rosen´s Emergency Medicine. Concepts and Critical Practice. J Marx, Hockberg. R, Walls R, et al (eds). Fith edition. Mosby. St Louis,1998. 3. Kapoor M, Chan GZ. Fluid and electrolyte abnormalities. Crit Care Clin 2001; 17:503-529. 4. Lind L, Carlstedt F, Rastad J, et al. Hypocalcemia and parathyroid hormone secretion in critically ill patients. Crit Care Med 2000; 28:93-99. 5. Lo CY. Postthyroidectomy hypocalcemia. J Am Coll Surg 2003; 196:497-498. 6. Marx SJ. Hyperparathyroid and hypoparathyroid disorders.N Engl J Med 2000; 343:1863-1875. 46.- Femenino de 26 años, acude al servicio de consulta externa refiriendo presentar desde hace varias semanas temblor fino distal, sensación de angustia, sudoración palmar, ha perdido peso en los últimos meses. Sin antecedentes de importancia. Exploración física: T-A 130/86 mm hg, peso 54 kg., talla 160 cm., exoftalmos, sudoración palmar, piel húmeda  y caliente, FC 110 lpm, abdomen con ruidos peristálticos incrementados, con aumento en el número de evacuaciones. En este paciente va a encontrar una disminución de:

a) b) c) d)

TSH. Cortisol en orina de 24 hrs T4 total. T3 r.

El bocio tóxico difuso (BTD) constituye la forma más frecuente de hiperfunción de la glándula tiroidea (70 % de los casos), que puede aparecer a cualquier edad, aunque por lo general aparece entre la tercera y cuarta década de la vida. Esta enfermedad es más frecuente en la mujer, donde se observa un predominio de 7:1 en relación con los hombres en regiones no bociógenas. Esta relación se reduce en las zonas de bocio endémico. Los factores genéticos desempeñan un papel esencial en la etiología y existe una predisposición familiar a esta enfermedad de Graves-Basedow. El BTD se caracteriza por la presencia de hipertiroidismo, bocio difuso y elástico, oftalmopatía, dermopatía, acropaquia tiroidea y onicolisis. Es importante el diagnóstico y tratamiento precoz del hipertiroidismo para evitar complicaciones, principalmente las cardiovasculares. TABLA I 1. Piel: • Piel fina, caliente y sudorosa. • Prurito. • Pelo f ino y frágil. Onicolisis.

2. Sistema cardiovascular:

• Taquicardia, palpitaciones y fibrilación auricular. • Insuficiencia cardiaca, angor pectoris, disnea

de esfuerzo, vasodilatación.

• Disminución de la respuesta a la digital.

3. Aparato digestivo: • Hiperdefecación.

• Disfunción hepática: hipertransaminasemia.

4. Aparato locomotor:

• Debilidad y atrofia de la musculatura proximal. • Osteoporosis. • Aumento de la maduración ósea en niños. • Hiperreflexia, temblor distal, mioclonias.

5. Sistema nervioso: • Irritabilidad, nerviosismo e insomnio. • Psicosis, hipercinesia.

6. Otros:

• Pérdida de peso a pesar de la polifagia. • Intolerancia al calor. • Alteraciones menstruales y disminución de la

fertilidad en mujeres.

• Pruebas de función tiroidea:

a) Determinación de hormonas tiroideas:  – L-tiroxina (T4) que circula en plasma unida en su mayoría a la proteína transportadora (TBG), y menos del 0,1% libre. Aunque sólo esta pequeña porción está libre, es la concentración de T4 libre más que la T4 total la que indica la actividad tiroidea; por lo

tanto, en la mayoría de los casos sólo es necesario determinar la T4 libre, que estará elevada en los casos de hipertiroidismo. – L-triyodotironina (T3): se produce por la desyodación periférica de T4; es regulada por factores independientes de la función tiroidea. En algunos pacientes hipertiroideos la concentración de T3 está elevada cuando no lo está la de T4. TSH, producida por células de la adenohipófisis, que controla la función tiroidea por acción directa positiva y que es controlada a su vez por la TRH hipotalámica con efecto positivo estimulador. En los casos de hipertiroidismo primario la TSH estará inhibida por el sistema de retroalimentación debido al exceso de hormonas tiroideas periféricas (T4 y T3). La TSH tiene una mayor sensibilidad para el diagnóstico del hipertiroidismo que la T4 libre. Sin embargo, no es del todo específica, es decir, una TSH baja no siempre indica hipertiroidismo. – TRH sintetizada a nivel hipotalámico. No se suele determinar en la práctica clínica. Existen algunos casos en los que la concentración de T3 yT4 se encuentran en el límite superior de la normalidad y persiste la sospecha de hipertiroidismo. En estos casos la administración de TRH no produce ningún in cremento en los niveles de TSH en los hipertiroidismos primarios (test de TRH para TSH). Así pues, para el diagnóstico de un hipertiroidismo utilizaremos básicamente los niveles de TSH y T4 libre.  – 

 – 

Hipertiroidismo: clínica, diagnóstico y tratamiento I. M.ª RECHE MOLINA, B. VALERA, C. HIDALGO, L. LEÓN, G. PIÉDROLA Servicios de Medicina Interna y Endocrinología. Hospital Universitario Virgen de las  Nieves. Granada 

47.- Masculino de 42 años alcohólico con cirrosis y ascitis. Es hospitalizado por agitación y comportamiento extraño. ¿Cuál de los siguientes hallazgos es el más útil para hacer el diagnóstico de encefalopatía hepática? a) b) c) d)

Ictericia Asterixis de las manos Hemangiomas superficiales Signo de la ola positivo

ENCEFALOPATÍA Alteración en el estado mental, en el comportamiento y en el sueño que progresa a la desorientación y al coma. Indica una insuficiencia hepática severa. Fisiopatologia Las toxinas son inactivadas por el hígado pero entran a la circulación portal por los cortos circuitos. Toxinas no son inactivadas y excretadas. Las toxinas se encuentran elevadas en el líquido cerebroespinal. •





 – 

 – 

 – 





TOXINAS. Amonio. Neuroexcitatorio y depresor del sistema nervioso central. Aminoácidos aromáticos. Son precursores de neurotransmisores. Si incrementan en sangre la encefalopatía no progresa. Acido gammaaminobutírico (GABA). Incrementado significativamente en la encefalopatía. Benzodiacepinas endógenas. Compiten por los receptores de GABA y barbitúricos.  – 



 – 

 – 



 – 



 – 

 – 

CUADRO CLINICO El diagnóstico de la encefalopatía hepática depende de la existencia de una enfermedad hepática, desde la insuficiencia hepática aguda y la toxicidad por fármacos, hasta las enfermedades crónicas, tales como la hepatropatía alcohólica o la cirrosis. En la historia clínica se hace énfasis especial en los antecedentes de enfermedades del hígado diagnosticadas con anterioridad, una historia de alcoholismo o de hepatitis. Transfusiones de sangre previas o abuso de drogas por vía intravenosa pueden sugerir una hepatitis crónica B o C. La utilización de fármacos hepatotóxicos, como la metildopa, la nitrofurantoina o la isoniácida, puede ser causa de una hepatopatía crónica, mientras que una dosis alta de paracetamol puede ser causante de una necrosis hepatocelular fulminante. El cuadro clínico incluye tres elementos: cambios en el estado mental, hedor hepático y asterixis. El hedor hepático se refiere al aliento fétido del paciente. La asterixis es un temblor por sacudidas, irregular y bilateral de las manos, debido a una interrupción momentánea y brusca del tono muscular de los antebrazos. El examen físico se centra en la búsqueda de los estigmas característicos de las enfermedades hepáticas: ictericia, nevus en araña, ginecomastia, atrofia testicular, venas distendidas en la pared abdominal (cabeza de medusa) y ascitis. La exploración neurológica durante los estadíos precoces suele mostrar apraxia de construcción y dificultad para escribir. Otros hallazgos físicos son la rigidez de las extremidades e hiperreflexia. En el estadío de coma profundo suele haber pérdida del tono muscular y disminución de los reflejos tendinosos profundos.

RESUMEN: •















Petequias. Rinofima. Contractura de Dupuytren (aponeurosis palmar). Telangiectasias en el tronco. Ascitis. Asterixis. Eritema palmar. Atrofia testicular.

Blibliografía: Shakelford’s. Surgery of the alimentary tract.

5a. Ed. 2002. Tomo 3.

Feldman´s. Gastroeneterology. 2002. Perez. Anatomía y fisiología del hígado. Univ. Católica de Chile. 2005. Bratiz. Serum laboratory test in cirrhosis. Journal of Hepatology. Slovakia. 2005. Paradis. Glycomics. Journal of hepatology. Ireland. Agosto 2005. 48.- Masculino de 36 años de edad acude a consulta por presentar nausea y vómito, aumento en el número de evacuaciones de consistencia aguada, (diarrea sin productos patológicos), afebril, T.a 110.70 mmhg, refiere que su hijo presentó los misma sintomatología casi al mismo tiempo que el, como antecedente refiere haber comido juntos en un restaurante hace aproximadamente 4 hrs ¿De los siguientes microorganismos cual es el de mayor probabilidad de ocasionar éste cuadro? a) Salmonella enteriditis. b) Shigella sonnei. c) Staphylococcus aureus. d) E. Coli La contaminación de alimentos por S. aureus , está asociada con una forma de gastroenteritis que se manifiesta clínicamente por un cuadro caracterizado por vómitos (76% de casos) y diarrea (77% de casos). El corto período de incubación de 1-6 horas orienta a la sospecha de enfermedad producida por ingestión de una o más enterotoxinas preformadas en el alimento que ha sido contaminado con cepas de S. aureus productor de la misma. Son raramente observados signos de toxicidad sistémica, tales como fiebre e hipotensión En general, es un cuadro autolimitado que típicamente se resuelve en 2448 horas desde el inicio.

No está claro si se desarrolla en humanos inmunidad a largo plazo, pero anticuerpos frente a una SE no necesariamente confieren inmunidad frente a la intoxicación por S. aureus,  ya que existe múltiples SE capaces de producir enfermedad. En algunos casos, anticuerpos producidos frente a una SE confieren protección cruzada contra otra SE, ya que algunas comparten epítopes. Todas las SE son capaces experimentalmente en primates de producir emesis, y no se registra enterotoxemia, excepto en dosis muy altas, probablemente debido a su dificultad para atravesar mucosas. El 99% de casos de intoxicación alimentaria por enterotoxinas Estafilocóccicas está asociado a S. aureus   y ocasionalmente se reportan casos por Staphylococcus epidermidis. Las cepas estafilocóccicas enterotoxigénicas aisladas de alimentos implicados en brotes de infección son mas a menudo lisadas por fagos del grupo lll, y menos frecuentemente simultáneamente por los grupos l y lll. Tratamiento. Como para la mayoría de enfermedades trasmitidas por alimentos autolimitadas, las medidas de sostén son la base del tratamiento. No está indicado tratamiento con antimicrobianos. Referencias bibliográficas. - Manual of Clinical Microbiology. Murray, P. 1995 6th edition. - Principles and Practice of Infectious Diseases. Mandell, Douglas, Bennett. 1995. 4th edition. - Dinges M, Orwin P, Schlievert P. 2000. Exotoxins of Staphylococcus aureus. Clinical Microbiology Reviews, vol 13; 16-34

49.- Masculino de 55 años que acude consulta por presentar dolor ocular y fotofobia intensa. En la exploración ocular se observa ojo rojo principalmente alrededor del limbo corneal, pupila en miosis y depósitos blanquecinos en endotelio. Se realiza la toma de presión ocular la cual es de 10 mm Hg (normal >16). ¿El diagnóstico probable corresponde es? a) b) c) d)

Queratitis. Conjuntivitis infecciosa. Uveítis. Glaucoma agudo.

La uveítis se define como la inflamación de la úvea, lámina intermedia del ojo que se encuentra entre la esclerótica  y la retina, la cual aporta la mayor parte del suministro sanguíneo a la retina. La uveítis es una de las causas del ojo rojo.1 Suele acompañarse de disminución de la agudeza visual, de curso lento y progresivo y dolor.

No produce secrecciones externas lo que la distingue de otras causas de ojo rojo, como la blefaritis, conjuntivitis  y el chalazión. Por extensión se llama uveítis a cualquier tipo de inflamación del interior del ojo. Suele cursar con cierta intolerancia a la luz y requiere de un exhaustivo oftalmológico. Puede afectar a uno o ambos ojos.

Tipos: Se suele clasificar en cuatro tipos: anterior , intermedia , posterior y panuveítica'. •







Anterior. Es la más frecuente, entre el 70 y 90 por ciento de las uveítis. Es una inflamación del iris del ojo, córnea y cuerpo ciliar,2 provocada a veces por una enfermedad autoinmune como la artritis reumatoide o la espondilitis anquilosante, aunque en la mayoría de los casos su causa es desconocida .1 Cursa con ojo rojo, conjuntiva irritada, dolor y pérdida de visión parcial. Intermedia o parsplanitis. Es la inflamación de la pars plana, área estrecha ubicada entre el iris y la coroides. Generalmente es un proceso leve que afecta a los hombres jóvenes y no se asocia a ninguna otra enfermedad. Es posible que haya una asociación con la enfermedad de Crohn  y con la esclerosis múltiple. A veces se complican produciendo hemorragias o depósitos de material inflamado en la pars plana.1 Posterior. Es la inflamación de la coroides o (coroiditis ). Si se afecta también la retina se llama coriorretinitis. Puede producir pérdida de visión de intensidad variable, dependiendo del tamaño y la localización de la cicatrización. Si se afecta la parte central de la retina, denominada mácula, la visión central se deteriora. Panuveítis: se ve afectada toda la úvea, es decir, los segmentos anteriores y posteriores del interior del ojo.

Olitsky SE, Hug D, Smith LP. Disorders of the Uveal Tract. In: Kliegman RM, Behrman RE, Jenson HB, Stanton BF, eds. Nelson Textbook of Pediatrics . 18th ed. Philadelphia, Pa: Saunders Elsevier; 2007: chap.628.

50.- Which one of the following conditions results in prologation of the partial thromboplastin time (PTT), but not the prothrombin time (PT)? a) Varicela hemorrhage as a result of cirrosis b) Menorrhagia resulting from von Willebrand’s disease c) Therapy with broad-spectrum antibiotics d) Therapy with coumarin for phlebitis

Enf. Von Willebrand

Manifestaciones

 Epistaxi s

60 %

 Hemorragia transvaginal

50 %

 Equimos is

40 %

 Gingivor ragias

35 %

 Hematomas

5%

 Hemartr osis

3%

EvW  Pruebas de

escrutinio

T. Hemorragia

Prolongado

C. Plaquetaria

Normales, excepto 2B

TTPa

Normal o prolongado

TP

Normal

Gpo AB0

25 % bajo en “0”



1.-Lee GR, Foerster J, Lukens J, Paraskevas F, Greer JP and Rodgers GM. Wintrobe’s clinical haematology; 10th Edition, Lippincott Williams & Wilkins, United

States of America, 1999. •





2.-Williams WJ. Manual Williams de hematología 5a Edición McGraw-Hill Interamericana, México, 1997.





3.-Beutler E, Lichtman MA, Coller BS, Kipps T. Hematology. 5th International Edition. United Stated of America, 1995.



4.-Ruiz Argüelles GJ. Fundamentos de hematología; 2ª. Edición, Editorial Médica Panamericana, México, 1998.

51.- Femenino de 19 años que acude a consulta externa por presentar un exantema cutáneo rojo difuso; fiebre de 39.4°C y diarrea leve acuosa. Como antecedentes refiere infección de garganta por la que se le administró sulfametoxazol. Comenzó sus menstruaciones hace tres días. En la exploración física se encuentran cambios eritematosos difusos de la piel con descamación temprana. La boca y las conjuntivas están eritematosas. ¿Cuál de los siguientes explica todo el proceso? a) b) c) d)

Bacteriemia por Salmonella  Síndrome de choque tóxico (TSS) Tuberculosis Mononucleosis por virus de Epstein-Barr

Allen R. M. MMS Medicina Interna. 5ª. Edición. National Medical Series. Mc. Graw Hill. 2006. (Capítulo 8 VII D 1 a, 2 b). La bacteriemia por Salmonella, el síndrome de choque tóxico (TSS), la tuberculosis y la mononucleosis de Epstein-Barr pueden acompañarse de fiebre, pero la presencia de exantema descamativo difuso sugiere TSS, reacción farmacológica grave (p. ej., síndrome de Stevens-John-son); enfermedad de Kawasaki, o escarlatina. El exantema cutáneo relacionado con salmonelosis es muy sutil y evanescente (manchas de color rosa). La tuberculosis no se caracteriza por afección cutánea difusa y de mucosas o diarrea acuosa. Si bien la alergia al sulfametoxazol puede producir eritema cutáneo y de mucosas, no causa diarrea. 52.- En la infección por virus de la inmunodeficiencia humana por lo común la linfadenopatía difusa en una persona clínicamente sana suele ser un signo de: a) b) c) d)

Linfoma Sarcoma de Kaposi Tuberculosis No indica infección o tumor específicos

Allen R. M. MMS Medicina Interna. 5ª. Edición. National Medical Series. Mc. Graw Hill. 2006. (capítulo 8 VIII G 1 b). La linfadenopatía difusa en una persona infectada por virus de la inmunodeficiencia humana (VIH) que se encuentra clínicamente bien suele ser signo de que no hay infección específica o tumor. Aunque todas esas respuestas pueden ser ciertas, las personas con múltiples ganglios aumentados de tamaño, tuberculosis o trastornos malignos tienden a encontrarse enfermas. Con mayor frecuencia también experimentan pérdida de peso y fiebre. Es más probable que el linfoma se presente con afección orgánica en pacientes infectados con VIH que en otros. El sarcoma de Kaposi puede producir afección linfática, pero en general sólo se encuentra en etapa tardía de la enfermedad con lesiones cutáneas y mucosas extensas. La linfadenopatía moderada es un dato común en infección por VIH en etapa media. Se desconoce su causa exacta, pero la desaparición de la linfadenopatía prolongada puede preceder al deterioro clínico. La sífilis produce adenopatía local o difusa en pacientes con infección por VIH o sin ella. Sin embargo, esta adenopatía siempre se acompaña de algún otro dato de sífilis. 53.- Acude a consulta un adolescente de 20 años la cual refiere que desde hace varios meses presenta astenia, cansancio, pérdida de apetito y dificultades para concentrarse en los estudios. Al interrogatorio refiere que a perdido interés en los estudios, frecuenta menos a sus amigos, con pesimismo en la mayoría de sus actividades. ¿Cuál es el diagnóstico más probable? a) b) c)  d)

Anorexia nerviosa Trastorno de ansiedad. Depresión mayor Distimia.

LA DISTIMIA La distimia es un estado de ánimo crónicamente deprimido, menos grave que la depresión y que no cumple los criterios para una depresión mayor, o lo hace sólo en períodos muy cortos. Su evolución suele ser de más de dos años. Se caracteriza por un abatimiento prolongado del estado de ánimo en que el sujeto distímico se describe a sí mismo como “triste” o “desanimado”, perdiendo el interés por las cosas y viéndose a menudo como inútil  y poco interesante. Posee síntomas persistentes o intermitentes, de intensidad más leve comparación a la depresión mayor. Aiskal (1983) la define como “mal humor” y se caracteriza porque “el individuo está habitualmente triste, introvertido, melancólico, excesivamente consciente, incapaz de alegría y preocupado por su insuficiencia personal”.

Los criterios de diagnóstico de Distimia son los que a continuación se detallan: A. Estado de ánimo crónicamente depresivo la mayor parte del día de la mayoría de los días, manifestado por el sujeto u observado por los demás, durante al menos 2 años. Nota: En los niños y adolescentes el estado de ánimo puede ser irritable y la duración debe ser de al menos 1 año. B. Presencia, mientras está deprimido, de dos (o más) de los siguientes síntomas: 1. Pérdida o aumento de apetito 2. Insomnio o hipersomnia 3. Falta de energía o fatiga 4. Baja autoestima 5. Dificultades para concentrarse o para tomar decisiones 6. Sentimientos de desesperanza C. Durante el período de 2 años (1 año en niños y adolescentes) de la alteración, el sujeto no ha estado sin síntomas de los Criterios A y B durante más de 2 meses seguidos. D. No ha habido ningún episodio depresivo mayor durante los primeros 2 años de la alteración (1 año para niños y adolescentes); por ejemplo, la alteración no se explica mejor por la presencia de un trastorno depresivo mayor crónico o un trastorno depresivo mayor, en remisión parcial. Nota: Antes de la aparición del episodio distímico pudo haber un episodio depresivo mayor previo que ha remitido totalmente (ningún signo o síntoma significativos durante 2 meses). Además, tras los primeros 2 años (1 año en niños y adolescentes) de trastorno distímico, puede haber episodios de trastorno depresivo mayor superpuestos, en cuyo caso cabe realizar ambos diagnósticos si se cumplen los criterios para un episodio depresivo mayor. E. Nunca ha habido un episodio maníaco, un episodio mixto o un episodio hipomaníaco y nunca se han cumplido los criterios para el trastorno ciclotímíco. F. La alteración no aparece exclusivamente en el transcurso de un trastorno psicótico crónico, como son la esquizofrenia o el trastorno delirante. G. Los síntomas no son debidos a los efectos fisiológicos directos de una sustancia (p. ej., una droga, un medicamento) o a enfermedad médica (p. ej., hipotiroidismo). H. Los síntomas causan un malestar clínicamente significativo o deterioro social, laboral o de otras áreas importantes de la actividad del individuo. Como vemos, la Distimia presenta síntomas más o menos similares a la Depresión mayor, pero se diferencian entre sí respecto a:

Tipo de evolución: el estado de ánimo depresivo es crónico (no presenta intervalos libres de síntomas o mejorías significativas) y dura por lo menos 2 años. Severidad de los síntomas: los mismos suelen ser leves o moderados, sin una alteración significativa de las relaciones familiares, sociales y laborales del individuo.

- Diagnostic and Statistical Manual of Mental Disorders, 4° edition (DSM-IV TR). American Psychiatric Association. American Psychiatric Press, 2000. - Kaplan and Sadock's Synopsis of Psychiatry, 9° edition. Lippincott Williams & Wilkins Press, 2003.

54.- Acuden a consulta los padres de un menor, que cuenta con cinco años de edad, lleva varias noches despertándose agitado como si hubiera soñado algo que le angustia. Cuando acuden a su lado por la noche, el niño les mira y dice palabras que no tienen ningún significado. Al cabo de un rato vuelve a dormirse y por la mañana no recuerda nada de lo ocurrido. El diagnóstico más probable que presenta éste paciente es: a) b) c) d)

Pesadillas Disomnia. Terrores nocturnos. Sonambulismo.

Criterios para el diagnóstico de F51.5 Pesadillas (307.47) A. Despertares repetidos durante el período de sueño mayor o en las siestas diurnas, provocados por sueños extremadamente terroríficos y prolongados que dejan recuerdos vividos, y cuyo contenido suele centrarse en amenazas para la propia supervivencia, seguridad o autoestima. Los despertares suelen ocurrir durante la segunda mitad del período de sueño. B. Al despertarse del sueño terrorífico, la persona recupera rápidamente el estado orientado y despierto (a diferencia de la confusión y desorientación que caracterizan los terrores nocturnos y algunas formas de epilepsia). C. Las pesadillas, o la alteración del sueño determinada por los continuos despertares, provocan malestar clínicamente significativo o deterioro social, laboral o de otras áreas importantes de la actividad del individuo. D. Las pesadillas no aparecen exclusivamente en el transcurso de otro trastorno mental (p. ej., delirium, trastorno por estrés postraumático) y no se deben a los efectos fisiológicos directos de una sustancia (p. ej., drogas, fármacos) o de una enfermedad médica.

Criterios para el diagnóstico de F51.4 Terrores nocturnos (307.46) A. Episodios recurrentes de despertares bruscos, que se producen generalmente durante el primer tercio del episodio de sueño mayor y que se inician con un grito de angustia. B. Aparición durante el episodio de miedo y signos de activación vegetativa de carácter intenso, por ejemplo, taquicardia, taquipnea y sudoración. C. El individuo muestra una falta relativa de respuesta a los esfuerzos de los demás por tranquilizarle. D. Existe amnesia del episodio: el individuo no puede describir recuerdo alguno detallado de lo acontecido durante la noche. E. Estos episodios provocan malestar clínicamente significativo o deterioro social, laboral, o de otras áreas importantes de la actividad del individuo. F. La alteración no se debe a los efectos fisiológicos directos de una sustancia (p. ej., drogas, fármacos) o de una enfermedad médica. 55.- En la unidad de Medicina Familiar recibe a un paciente de 64 años de edad que asegura ser hipertenso de más de cinco años de evolución, sin agudizaciones además de ser portador de una fibrilación auricular, motivo por los cuales recibe propranolol en dosis de 100 MG cada mañana. Al revisar su electrocardiograma usted espera encontrar: a) Ausencia de onda P, complejo QRS normal e intervalos R-R diferentes. b) Ausencia de onda P, complejo QRS ensanchado, diferencia constante de los intervalos RR. c) Presencia de onda P, complejo QRS ensanchado y eje rotado a la derecha. d) Ritmo nodal con disociación A-V

Guadalajara J. Cardiología. Sexta Edición 151 – 152 La fibrilación auricular es la arritmia cardiaca más frecuente. La despolarización caótica y desordenada de las aurículas por múltiples ondas en simultánea, trae como consecuencia que se pierda la función mecánica de la contracción auricular. Estos dipolos de activación múltiple y desordenados alcanzan al nodo A-V y penetran en él, algunos pasan hacia el Haz de His mientras que otros no lo alcanzan, debido a la penetración parcial del nodo por conducción decreciente completa, esto se conoce como, conducción oculta; así, la rápida penetración de los estímulos auriculares favorece la aparición de conducción oculta, la cual afecta en forma impredecible el periodo refractario del nodo. El trazo electrocardiográfico típico es la ausencia de onda P, complejo QRS normal e intervalos R-R diferentes.

56.- Acatempa, Gro. Es una comunidad de 100,000 personas. Durante 2001 hubo 1,000 defunciones por todas las causas. Durante el mismo año se registraron un total de 300 casos de Infartos Agudos al Miocardio y 60 defunciones por esta misma causa. La tasa de mortalidad específica para Infarto Agudo al Miocardio en Acatempa Gro. es de : a) b) c) d)

20% 20 por 100,000 60 por 100, 000 10 por 100,000

La tasa de mortalidad para cualquier enfermedad especifica, puede expresarse para toda la población o para cualquier subgrupo de edad, raza o sexo. Se calcular dividiendo el número de muertes causadas por la enfermedad especifica entre la población total y se expresan como muertes por 100, 000 habitantes por año. ( TMe= 60/ 100, 000 por 100, 000) Morton R. F. Bioestadística y Epidemiología, Interamericana, 3ª. Ed. 1993; pág: 22.

57.- Se trata de un recién nacido al que se decide colocar una sonda orogástrica, al darnos cuenta de que dicha sonda no avanza y apreciar en una radiografía toracoabdomial la ausencia de aire en el intestino, ¿Sospechamos de?: a) Atresia de esófago tipo II y V b) Atresia de esófago tipo III y IV c) Hernia diafragmática congénita d) Atresia de esófago tipo I y II. . Consiste en la interrupción de la luz esofágica con o sin comunicación con la vía aérea. No  existe aún una definición precisa del término "long gap". De acuerdo a nuestra experiencia 

se trata de aquellas AE, independientemente del tipo, caracterizadas por una distancia  entre cabos esofágicos lo suficientemente grande como para imposibilitar la realización de  una anastomosis primaria término-terminal.

De acuerdo a nuestro criterio, basado en la clasificación de Ladd, las atresias de esófago pueden clasificarse en los tipos I, II, III, IV y V 1). La Tipo I es la atresia de esófago aislada sin fístula traqueoesofágica o “atresia pura”. No tiene comunicación con la vía aérea. Representa aproximadamente el 5% de los casos observados y es la segunda en frecuencia. Puede diagnosticarse con más facilidad en el embarazo, que cursa típicamente con polihidramnios y ausencia de imagen gástrica en las ecografías prenatales. Al nacer, los neonatos presentan el abdomen excavado por falta de pasaje de aire al intestino. En todos los casos ambos cabos esofágicos se encuentran muy separados entre sí.

La Tipo II es una forma muy rara de atresia de esófago que representa del 1 al 3% de los casos. Existe una fístula traqueoesofágica desde el cabo superior del esófago a la tráquea cervical. Al igual que en las Tipo I, no hay pasaje de aire al intestino distal y ambos extremos se encuentran muy alejados entre sí. La Tipo III es la forma observada en el 90% de los pacientes. Comúnmente se la conoce como sinónimo de atresia de esófago. Se caracteriza por una bolsa esofágica comunicación entre el extremo distal esofágico y la tráquea (fístula tráqueoesofagica al cabo inferior), que puede nacer a cualquier altura de la tráquea y excepcionalmente, también en los bronquios. En estos casos el embarazo no cursa típicamente con polihidramnios y suele verse una imagen gástrica normal en las ecografías prenatales. Al nacer los neonatos suelen presentar el abdomen distendido por el gran pasaje de aire que existe desde la vía aérea hacia el estómago e intestino. La distancia entre ambos cabos es por lo general inferior a dos o tres vértebras. La coexistencia de atresia de esófago Tipo III con atresia duodenal, intestinal o malformaciones anorrectales constituye un cuadro de urgencia; el gran flujo de aire desviado hacia el sistema digestivo a través de la fístula traqueoesofágica inferior provoca distensión abdominal severa y eventualmente, incluso, perforación intestinal. La Tipo IV se caracteriza por una fístula tráqueoesofagica en ambos cabos esofágicos. Su observación es poco frecuente. Representa menos del 2% de los casos. La fístula superior suele encontrarse por accidente cuando se efectúa la corrección quirúrgica inicial. En ocasiones pasa inadvertida por el cirujano y su diagnóstico es efectuado en forma tardía. La Tipo V no es en realidad una atresia de esófago. El esófago se encuentra permeable y es de buen calibre. En estos pacientes lo que se observa es una fístula traqueoesofágicaaislada, generalmente de ubicación cervical o torácica alta. El diagnóstico se realiza excepcionalmente en el periodo neonatal ya que los síntomas suelen aparecer en la infancia.

Bibliografía: Urgencias en Pediatría, Interamericana.McGraw – Hill. Capítulo: Urgencias Médico Quirúrgicas, Sección XXIII, pág. 774-778. Operative Pediatric Surgery. Moritz M. Ziegler. International Edition, pág. 349- 354. Cirugía Pediátrica, Ashcraft - Holder Interamericana.McGraw – Hill pág. 257 a 277.

58.- Masculino recién nacido que a la exploración física presenta peso 3.900kg, talla 55cm, ausencia de lanugo y vérmix gaseosa, la sospecha diagnóstica en éste paciente es: a) Prematuro b) Término c) Postmaduro d) Hipotrófico Definición: Se define como recién nacido postérmino a cualquier niño nacido más allá de 42 semanas de gestación, cualquiera que sea su peso. El recién nacido postmaduro es el producto de una gestación postérmino o gestación prolongada que presenta signos de postmadurez.

Signos más comunes: Peso normal o disminuido y talla aumentada. Piel seca y descamada, puede ser apergaminada. Uñas largas y quebradizas. Manos tumefactas con descamación precoz e intensa. Abundante cabello. Lanugo y vérmix caseoso ausentes. Depósitos de grasos disminuidos. Coloración verde, marrón o amarilla de la piel y uñas por impregnación de meconio. Actitud vivas y facies despierta. BIBLIOGRAFÍA: · García-Monroy L., Embarazo Prolongado en: Nuñez-Maciel E. et al. Tratado de Ginecología  y Obstetricia, Volumen II Cap.50. México, Ed. Cuéllar, S.A de C.V; 2001. pp. 687-694. · Matute GMM. Inicio del trabajo de parto. En: Nuñez-Maciel E. et al. Tratado de Ginecología y Obstetricia, Volumen I Cap.10. México, Ed. Cuéllar, S.A de C.V; 2001. pp. 187194. · Clinicas Obstétricas y Ginecologicas Vol 1:2002. McGraw-Interamericana. · Álvarez-Ponce VA., Lugo-Sánchez AM., Álvarez-Sánchez AZ., Muñiz-Rizo ME. Comportamiento del embarazo prolongado en el servicio de cuidados perinatales. Rev Cubana Obstet Ginecol 2004.

59.- Femenino de 34 años la cual labora en lavandería, hace varios años presenta enrojecimiento de los pliegues proximales de las uñas de varios dedos de las manos, que ocasionalmente le supuran. ¿Cuál es el diagnóstico más probable? a) Dermatitis de contacto. b) Liquen plano. c) Dermatoficia. d) Paroniquia candidiásica crónica.

Infección por Candida del lecho ungueal que se presenta con mayor frecuencia como una onicolisis asociada con paroniquia, aunque también se observa la destrucción completa del lecho ungueal y la erosión de la zona distal y lateral de las uñas de los dedos, sin distrofia ungueal total. La perionixis candidiásica se trata de la inflamación con eritema, edema, dolor y salida ocasional de pus blanquecino escaso y espeso del reborde ungueal de uno o varios dedos de la mano. Con frecuencia, se acompaña de onicopatía (discoloración, onicolisis, distrofia total, etc.).

Predisponentes: Todas las labores manuales que generen humedad. (muy frecuente en amas de casas, trabajadores de restaurantes, lavanderas, etc.)

BIBLIOGRAFÍA: 1. Crespo Erchiga V. Proto colo diagnóstico de contaminantes. En "Micología para dermatólogos" Ed.

Janssen, Madrid, 1994, pp:49-70. 2. Crespo V, De Luís B, Delgado V, Crespo A y Vera. Espectro etiológico de las onicomicosis en nuestro medio. CO7. II Congreso Nacional de Micología. Santiago de Compostela. Junio, 1994. 3. Crespo Erchiga V, Delgado Florencio V y Martínez García S. Micología dermatológica. Ed. M.R.A. Barcelona, 2006. 4. Daniel III CR. The Diagnosis of Nail Fungal Infection. Editorial Arch Dermatol 1991;127:1566-1567. 5. Delgado Florencio V. Protocolo de identificación de dermatofitos. En "Micología para dermatólogos".

Ed. Janssen, Madrid, 1994, pp:27-41. 6. Delgado Florencio V. Estrategia en el diagnóstico y tratamiento de las micosis superficiales. Ed. Aula Médica, Madrid, 1994. 7. Delgado V, Abad Romero-Balmas J, Armijo Moreno M y Dulanto F. Scopulariopsis brevicaulis como agente de onicomicosis. Actas Deermo-Sif. 1976; 9-10:693-700. 8. English MP. Nails and fungi. Br J Dermatol 1976; 94:697-701. 9. Fevilhade de Chauvin M. Onicomicosis. Dermatología práctica. 1994; 9:1-2. 60.- El siguiente diagnóstico tiene como característica la aparición de cilindros hemáticos en el sedimento urinario: a) Daño tubular. b) Cualquier lesión de la nefrona. c) Lesión a cualquier nivel de las vías urinarias. d) Daño glomerular severo.

La presencia de cilindros hemáticos o sus derivados indica hemorragia dentro de la neurona que puede ser el resultado de una lesión glomerular como la que se ve en la nefritis hemorrágica aguda, o una enfermedad manifestada por necrosis vascular o del penacho vascular como ocurre en la Periarteritis nodosa púrpura de Henoch-Schonlein, endocarditis bacteriana subaguda. También la necrosis tubular renal acompañada de inflamación intersticial puede producir hematuria y formación de cilindros hemáticos. Asimismo, la nefrosis hemoglobinúrica resultante de la administración de sangre incompatible y la hemólisis intravascular intensa pueden producir hallazgos similares Bauer. J. D.; Toro. G... v Ackermann. P. G. (eds.): Brav's Clinical Laboratorv Methods. ed. 6, 'St. ouisC,. V: M O S ~C~o. Bibliografía nefrológica: FARRERAS-ROZMAN. Medicina Interna; Harcourt Brace España S.A. GUARDIA J, GRAU JM, NET A. Medicina Interna Fundamental; Springer Verlag Iberica S.A. ARTHUR GREENBERG. Tratado de enfermedades renales. Harcourt Brace. 61.- Posterior a un trabajo de parto con expulsión normal, y tras una hora aproximada en periodo de alumbramiento en el que se practicó masaje uterino y se incrementó moderadamente la dosis de oxitocina, no aprecian signos de desprendimiento placentario, se indica una extracción manual de placenta, que resulta imposible por no existir plano de separación entre la placenta y la pared uterina. ¿Cuál es el diagnóstico más probable? a) Engatillamiento placentario. b) Placenta succenturiata con cotiledón aberrante. c) Placenta circunvalata d) Placenta adherente por acretismo placentario. Se denomina a la placenta como acreta cuando ésta se implanta en zonas donde la decidua es deficiente o anormal y por tanto hay una infiltración del miometrio por vellosidades coriales; esta infiltración puede ser focal, parcial o total. A su vez esta condición se subdivide en acreta, increta y percreta. La placenta increta y percreta infiltran todo el espesor de la pared miometrial, en la percreta además las vellosidades, perforan la serosa  y llegan en algunas ocasiones a infiltrar órganos vecinos, especialmente la vejiga. La PA está limitada a la superficie miometrial. La frecuencia de presentación del AP varía entre 10 y 48 por 10.000 partos. (Oishi A 1999, Hung TH 1999, Zaki ZM, 1998).

En las mujeres con acretismo placentario se han visto factores de riesgo, dentro de los cuales se encuentran: 1. Edad y multiparidad: La presentación AP, aumenta con la paridad de la paciente y la edad, siendo muy rara en primíparas. 2. Placenta previa: esta se ha encontrado en el 30% de los casos de PA. Igualmente se ha visto PA en el 9.3% de las pacientes con placenta previa (Miller DA, 1997). 3. Cesárea anterior, o cirugías uterinas previas: Se ha visto este antecedente en el 25% de los casos. En el 29% de los casos la PA estaba implantada en la cicatriz uterina y solo en el 5% la placenta estaba implantada en otro sitio. (Miller Da, 1997). 4. Dilatación y legrado, en el 25% de los casos. 5. Infección uterina previa, remoción manual de la placenta, leiomiomas y otras anomalías uterinas: La asociación con estas entidades es inconstante. 6. Niveles anormalmente elevados de feto-proteína y de b-HCG, en el segundo trimestrre. (Hung TH, 1999). Una placenta adherente o penetrante no es fácil de diagnosticar antes del alumbramiento. Después de éste, se manifiesta como retención placentaria y sangrado uterino. El diagnóstico generalmente se realiza, después de intentar la extracción manual de la placenta. Las manifestaciones clínicas propias de la placenta adherente, de la placenta acreta y de la placenta increta, consisten en una manifiesta dificultad o imposibilidad para la expulsión o extracción de la placenta. Como consecuencia de la atonía parcial y de la hemostasis insuficiente en las zonas de despegamiento placentario, se producirá una hemorragia más o menos grave que en nada se diferenciará de la hemorragia de la atenía uterina. Y no será solamente al intentar el alumbramiento artificial que se pondran de manifiesto las razones íntimas de la retención placentaria; alumbramiento que será engorroso en. la placenta ahderente e imposible en las variedades acreta e increta. En varias ocasiones puede no existir hemorragia y en estos casos la única manifestación de este estado morboso será la prolongación del período del alumbramiento. La placenta adherente, como toda placenta retenida, es pronto presa de un proceso infeccioso sin embargo se han señalado casos de placentas retenidas asépticamente durante muchos mese3. al cabo de los cuales han sido expulsadas sin causar trastorno alguno; pero hay que hacer observar que en estas enfermas se ha tratado de retensión de mebranas por abortes ovulares.

1.

Arredondo-Soberon F, Sabella V, Garza-Leal J, Valente PT. Placenta increta en primer trimestre de embarazo. Ginecol Obstet Mex 1995; 63: 279-81. 2. Cantanzarite V, Stanco L, Schrimmer S et al. Managing placenta previa/accreta. Contemp Obstet Gynecol 1996; 41: 66-95.

3. Ecker JL, Sorem KA, Soodak L, et al. Placenta Increta Complicating a FirsTrimester Abortion A case report. Journal Reproductive Medicine. 1992; 37-10. 4. Finberg G, William J. Placenta accreta: prospective sonographic diagnosis in patients with placenta previa / accreta. Contemp Obstet Gynecol 1996: 41: 66-95. 5. Gist RS, Voung V, Brody S, Rees P, Landry AD. Placenta increta occurring in a bligter ovum. South Med J. 1996; 89(5): 545-7. 6. Harden, MA, Walters MD, Valente PT Postabortal hemorahage due to placenta increta: A case report. Obstet Gynecol. 1990; 75: 523. 7. Hudon L, Belfort MA, Broome DR. Dosis and management of placenta percreta: A review. Obster Gynecol survey 1998; 53: 509-517. 8. Hung TH, Shau WY, Hsieh CC, et al. Risk factors for placenta accreta. Obstec Gynecol 1999; 93: 545-50. 9. Kinoshita T, Ogawa K, Yusumizu T, Kato J. Spontaneous rupture of the uterus due to placenta percreta at 25-weeks´ gestation: a case report J Obster Gynaecol Res 1996; 22: 125-8. Kirkinen P, Helin-Martikainen HL, Vanninen R, Patanen K. Placenta accreta: imaging by gray-scaleand contrast enhanced color Doppler somography and magnetic resonance imaging. J Clin Ultrasound 1998; 26: 90-4.

62.- Femenino de 19 años que acude a consulta refiriendo presentar 6 meses sin regla. Se manifiesta preocupada por su sobrepeso, ha estado a dieta y ha perdido 6 kg en 8 meses. Actualmente pesa 46 Kg. con talla de 1,65 mts. Signos vitales dentro de sus parámetros normales. Se realiza prueba de embarazo con resultado negativa. ¿Cuál es la causa más probable de su amenorrea? a) b) c) d)

Disgenesia gonadal. Síndrome de ovario poliquístico. Adenoma hipofisario. Hipogonadismo hipogonadotropo.

El hipogonadismo femenino se caracteriza por un fallo gonadal debido a la alteración del propio ovario o secundario a un fallo hipotálamo-hipofisario. Este fallo de la gónada puede ocurrir en distintos momentos de la vida y por causas diversas, lo que va a condicionar una presentación clínica diferente. El ovario posee dos funciones relevantes: producir gametos femeninos (oogénesis) y secretar hormonas esenciales en la regulación de la función reproductora y que influyen en la diferenciación y el desarrollo de los órganos sexuales (hormonogénesis). Los mecanismos celulares y moleculares responsables de la producción de oocitos y hormonas por la gónada femenina son parcialmente independientes entre sí. Sin embargo, ambas funciones son llevadas a cabo de modo concertado gracias a la acción de un complejo

sistema de control que implica interacciones múltiples entre el hipotálamo, la hipófisis y el ovario. Para la adquisición adquisición de la capacidad reproductora a partir de la pubertad se requiere una adecuada formación e integración funcional durante las etapas tempranas del desarrollo. Los elementos que componen este eje reproductor son: 1) el sistema neuronal hipotalámico responsable de la producción del neuropéptido GnRH; 2) las células gonadotropas de la hipófisis anterior que secretan LH (hormona luteinizante) y FSH (hormona folículo estimulante), y 3) el ovario. Hay que destacar que la formación de la gónada y de los elementos el ementos hipotálamohipofisarios del eje gonadotropo se llevan a cabo de modo independiente en etapas tempranas del desarrollo embrionario, produciéndose posteriormente su integración funcional en circuitos de retroalimentación positivos y negativos, esenciales para una correcta función reproductora. FALLO GONADAL PRIMARIO O HIPOGONADISMO HIPERGONADOTRÓFICO Incluye a aquellos pacientes en los que la producción y acción de los esteroides están reducidas. En los primeros 4 años de vida y a partir de los 9-10 años, la hipófisis, al faltarle la retroalimentación negativa de estos esteroides, aumenta la producción de gonadotropinas. Sin embargo, la elevación de las gonadotropinas no puede normalizar la función gonadal. Las causas pueden ser congénitas y adquiridas. Etiología común a ambos sexos Alteraciones congénitas del receptor de las gonadotropinas  Recientemente se han clonado y mapeado los genes de los receptores de as gonadotropinas (LH-R y FSH-R) (20), estando ambos en 2p21, y se han identificado diversas mutaciones y deleciones que conducen a una disminución de la función de estos receptores, independiente de las gonadotropinas (hipogonadismo hipergonadotropo); en otros casos se produce un aumento de la función de estos receptores. Mutaciones que ocasionan pérdida de la función del receptor de la LH  Hasta la fecha se han encontrado seis sustituciones de una base del gen del LH-R que conducen a dos mutaciones nonsense o sin sentido que ocasiona un codón de parada, y a cuatro mutaciones missense o mutaciones con error de sentido en los que cambia un aminoácido en la proteína. La mayoría son homocigotos y se considera que el trastorno es A-R. Clínicamente, existe una forma grave y una menos grave, lo que tiene relación con la actividad del receptor. En las mujeres hay amenorrea, falta de respuesta del ovario a la LH  y respuesta normal a la FSH con desarrollo folicular normal. Mutaciones que ocasionan pérdida de la función del receptor de la FSH  Son más raras. Clínicamente las mujeres homocigotas presentan disgenesia ovárica e hipogonadismo hipergonadotropo, mientras que las mujeres heterocigotas son fenotípicamente normales.

BIBLIOGRAFÍA: 1. Cañete R, Jiménez L. Ontogenia y diferenciación gonadal. Gónadas. Actualizaciones en Endocrinología. McGraw-Hill Interamericana. Capítulo 1, 2000. 2. Barrio R, Ezquieta B. Mecanismos genéticos de la l a diferenciación sexual: sus alteraciones. Gónadas. Actualizaciones en Endocrinología. McGraw-Hill Interamericana. Interamer icana. Capítulo13, 2000. 3. De Roux N, Morel Y, Hardelin Har delin JP. Genetic anomalies of the gonadotropic axis. Rev Prat Pr at 1999; 49 (12): 1277-1282. 4. Tena-Sempere M, Aguilar E, Pinilla L. Fisiología del eje hipotálamo-hipofiso-ovárico. Gónadas. Actualizaciones en Endocrinología. McGraw-Hill Interamericana. Capítulo 2, 2000. 5. Labarta JI y cols. Hipogonadismo hipergonadotrópico. En Pubertad normal y patológica, 2. º curso de postgrado. Valladolid, 1996. 6. Veldhuis JD. Neuroendocrine mechanisms mediating awakening of the human gonadotropic axis in puberty. Pediatr Nephrol 1996; 10 (3): 304-317. 7. Hopwood NJ. Pathogenesis and management of abnormal puberty. Spec Top Endocrinol Metab 1985; 7: 175-236. 8. Minagawa M, Yasuda T, Niimi H J. Spinal and femoral bone mass accumulation during normal adolescence: comparison with female patients with sexual precocity and with hypogonadism. hypogonadism. Clin Endocrinol Metab 1996; 81 (3): 1248-1253. 9. Park KH, Lee SJ, Kim JY, Kim JY, Bai SW, Kim JW. A concomitant decrease in cortical and trabecular bone mass in isolated hypogonadotropic hypogonadism and gonadal dysgenesis. Yonsei Med J 1999; 40 (5): 444-9. 63.- En el cunero, se observa que un neonato presenta asimetría de pliegues glúteos. A la EF la cadera, una de ellas puede ser fácilmente luxada posteriormente con un “click” y regresada a su posición normal con un sonido parecido. La familia se encuentra preocupada porque el primer hijo tuvo t uvo el mismo problema. ¿Cuál es el diagnóstico más probable? prob able? a) b) c) d)

Enfermedad de Legg-Perthes Artritis séptica Parto traumático Displasia de cadera

La falta de relación normal en las estructuras que forman una articulación se conoce como luxación. En el caso de la cadera, la cabeza del fémur no encaja debidamente en su cavidad (acetábulo). El desarrollo del acetábulo será normal siempre que la cabeza del fémur permanezca en posición correcta y el esfuerzo muscular sea adecuado. Siempre que esto no se cumpla, se producirá una alteración entre el cótilo y la cabeza femoral, dando lugar a una alteración con el resultado de luxación de la cadera. DIAGNÓSTICO: EXPLORACIÓN FÍSICA a) Maniobras de Ortolani y Barlow - click audible-. Ortolani: el click se produce cuando la fóvea de la cabeza del fémur encuentra la cresta cotiloidea caminando sobre ella. La limitación a la abducción desaparece, esto es signo de reducción. Barlow: se realiza la aducción de la cadera, que tras una ligera presión longitudinal sobre el fémur, produce una luxación. La prueba de Ortolani traduce una luxación y la de Barlow una cadera luxable. b) Asimetría de pliegues -no valorable en luxaciones bilaterales-. c) Limitación a la abducción. d) Dismetría.

• García Portabella, M.

Luxación congénita de cadera antes de los tres meses de edad. 2001. • Garcia-Siso Pardo, J. M. Displasia del desarrollo de la cadera. (Parte I). Peditría Rural y Extrahospitalaria. Vol. 32. Nº 304 Págs. 481-491. 2002. • Graf, R.

Classification of hip joint dysplasia by means of sonography. sonography . Arch Orthop Trauma Surg 102:248-255, 1984.

• Edeiken, J.

Luxación congénita de cadera. Diagnóstico Radiológico de las Enfermedades de los Huesos. Ed. Médica Panamericana. Págs. 388-392. 1977.

64.- Femenino de 2 años, es llevada al servicio de urgencias por presentar dolor abdominal intermitente. Antecedentes: evacuaciones sanguinolentas en número de 6 en 24 horas de 48 hrs de evolución. Exploración física: T/A 110/65mmHg, FC 90 x´, FR 28 x´, Temp 36.5°, irritabilidad, llanto fácil, se palpa masa dolorosa a nivel de colon ascendente. En este paciente el diagnóstico más probable es: a) b) c) d)

Divertículo de Meckel. Invaginación intestinal. Absceso apendicular. Apendicitis aguda

Invaginación intestinal: Ocurre cuando una porción del tracto alimentario se pliega dentro del segmento adyacente. Es la causa más frecuente de obstrucción intestinal entre los 3 meses y los 6 años de edad. El cociente varón: mujer es 4:1. Se produce de forma súbita con dolor cólico paroxístico intenso recurrente a intervalos frecuentes, el 60% de los lactantes expulsan heces que contienen sangre roja y moco, las deposiciones en confitura de grosella. Nelson, Tratado de Pediatria, 17ª Edición, Ed. Elsevier, Pág.1242-1243 65.- Femenino de 27 años se diagnostica con insuficiencia suprarrenal primaria en éste caso podemos encontrar las siguientes alteraciones de laboratorio: a) Hipercalcemia e hipernatremia b) Hipernatremia e hiperglucemia c) Hipocalcemia y leucopenia d) Hipercalemia e hiponatremia En la insuficiencia suprarrenal primaria se observa disminución en los niveles séricos de sodio, cloruro y bicarbonato, mientras que el potasio sérico se eleva. Esta se debe a los efectos combinados de déficit de aldosterona, reducción del filtrado glomerular y acidosis. Y sobre todo durante el estrés puede haber disminución en los niveles de glucosa.



Datos de laboratorio

1. Hiponatremia: los niveles séricos bajos de sodio se debe a su pérdida por la orina por déficit de aldosterona  y al desplazamiento del sodio hacia el compartimento intracelular. Esta pérdida de sodio extravascular reduce el volumen plasmático y acentúa la hipotensión. 2. Hiperkalemia: aumento de los niveles séricos de potasio. Se debe a los efectos combinados del déficit de aldosterona, la reducción del filtrado glomerular y la acidosis.

3. Hipocortisolemia: los niveles de cortisol y aldosterona son bajos y no aumentan con la administración de ACTH. 4. Hipercalcemia: aumento de los niveles séricos de calcio. Ocurre en un 10-20% de los pacientes de causa desconocida. 5. Cambios electrocardiográficos: suelen ser inespecíficos, aunque con lentificación generalizada del trazado. 6. Hemograma: puede haber anemia normocítica, linfocitosis relativa y eosinofilia moderada. 7. Prueba de estimulación de ACTH: prueba principal que confirma el diagnóstico de insuficiencia suprarrenal, al evaluar la capacidad de las suprarrenales para producir esteroides, que suelen estar ausentes o disminuidos tanto en sangre como en orina tras la estimulación de ACTH. 8. Determinación de la ACTH: en la insuficiencia suprarrenal primaria o Enfermedad de Addison, la ACTH y sus péptidos afines, están elevados en plasma ante la pérdida del mecanismo de retroalimentación del eje hipotálamo-hipófisariosuprarrenal. 9. hipertermia: la hormona del hipotálamo no controla la homeostasis

Williams GH, Dluhy RG. Enfermedades de la corteza suprarrenal. En Jameson JL (ed): Harrison. Endocrinología. 1a ed. Madrid. MacGraw-Hill España, 2006: 137-138.

66.-Mujer de 42 años. Acude a consulta por cefalea y acúfenos. Tiene antecedente de DM en padre. EF: peso 78 kg, estatura 1.62, perímetro abdominal 108 cm, TA: 140/100. Laboratorio: glucosa de ayuno 116, prueba de tolerancia a la glucosa, resultado a las 2 horas de 189. Los diagnósticos que se establecen en la paciente son: a) b) c) d)

Obesidad y diabetes mellitus Síndrome metabólico con intolerancia a la glucosa Obesidad e intolerancia a la glucosa Síndrome metabólico y diabetes mellitus

GLUCOSA: /=150 mg/dL ( >/= 1,7 mmol/L) · HDL col 0,9 mmol/L) en hombres o < 39 mg/dL 30 kg/m2 y/o cintura/cadera > 0.9 en hombres, > 0.85 en mujeres · Microalbuminuria >/= 20 μg/min o albúmina/creatinina _ 30 mg / g Por su parte, los criterios clínicos del NECP-ATPIII para el diagnóstico de síndrome metabólico son ligeramente distintos (1) · Obesidad abdominal: CC: > 82 cm (mujer) y > 102 cm (hombre) · Tiglicéridos >150 mg/dL · HDL col < 50 (mujer) o < 40 (hombre) · Presión arterial >/= 130 / >/= 85 mm Hg · Glucosa en ayunas >/= 110 mg/mL

Nótese que el NECP-ATPIII establece valores más bajos para la presión arterial y la ADA para la glucosa en ayunas >/= 100 mg/dL)

67.- Se trata de femenino de 75 años que desde hace dos semanas presenta ictericia. Hace dos días se añade confusión mental. A la EF FR 20, FC 110, TA 90/60, Temperatura 39°, no responde a comandos verbales, pero se aleja del dolor que se provoca al palpar el hipocondrio derecho y epigastrio. Los datos clínicos en el caso anterior son sugestivos de: a) b) c) d)

Coledocolitiasis Cáncer de páncreas Cirrosis Colangitis

Diagnóstico El diagnóstico de colangitis se basa en la asociación de signos y síntomas de infección con los propios de una obstrucción biliar. La presentación clásica es la aparición de dolor en hipocondrio derecho o epigastrio junto a fiebre, generalmente alta, con escalofríos, e ictericia (tríada de Charcot). Cuando se añade confusión mental y shock (sepsis) se denomina Pentada de Reynolds, que se observa con menor frecuencia, pero habitualmente se asocia con una colangitis supurada grave. Sin embargo, la correlación entre la clínica típica, las formas atípicas y la presencia de pus en la vía biliar es pobre y en muchos casos de colangitis faltan algunos de estos rasgos. Algunos enfermos, sobre todo de edad avanzada, pueden tener confusión mental o shock sin fiebre, o existir una leucocitiosis con desviación izquierda como única manifestación de la infección, por lo que se debe sospechar la existencia de una colangitis subyacente, sobre todo en enfermos de edad con algunas de estas manifestaciones. El laboratorio muestra hallazgos de obstrucción biliar con aumento variable de la bilirrubina y enzimas de colestasis. Es habitual encontrar leucocitosis con desviación izquierda, siendo por lo general las cifras más altas, en torno a 20.000 por mm 3 , reflejo de las formas más graves. La ecografía es la técnica de elección para detectar la existencia de obstrucción biliar por su elevada eficacia y versatilidad, pudiéndose detectar también complicaciones de la colangitis, como el absceso hepático. Dentro de la colangitis se pueden establecer unas formas leves, generalmente de buen pronóstico, y unas formas graves, donde se concentra la mayor parte de la mortalidad. Las primeras se manifiestan como cuadros febriles sin signos de afectación sistémica que se autolimitan espontáneamente o bajo tratamiento médico en 24-48 horas. Las formas inicialmente graves son las que asocian confusión mental, hipotensión, shock o fracaso renal. La edad es un factor constante de aumento de la morbimortalidad.

Referencias bibliográficas: 1. Bilhartz LE, Horton JD. Gallstone disease and its complications. En: Gastrointestinal and liver diseases. Sleisenger and Fordtran. Filadelfia: WB Saunders Co., 1998; 948-972. 2. Chung-Mau L, Chi-Leung L, Lai ECS, Sheuns-Tat F, Wong J. Early versus delayed laparoscopic cholecystectomy for treatment of acute cholecystitis. Ann Surg 1996; 223: 37-42. 3. Hamy A, Visset J, Likholatnikov D, Lerta F, Gibaud H, Savigny B et al. Percutaneus cholecystostomy for acute cholecystitis in critically ill patients. Surgery 1997; 121: 398401. 4. Harris A, Chong Hen Chang A, Torres-Viera C, Hammett R, Carr-Locke D. Meta-analysis of antibiotic prophylaxis in endoscopic retrograde cholangiopancreatography (ERCP). Endoscopy 1999; 31: 718-724. 5. Hermann RE. Surgery for acute and chronic cholecystitis. Surg Clin North Am 1990; 70: 1.263-1.275. 6. Koo Kp, Thirlby RC. Laparoscopic cholecystectomy in acute cholecystitis. What is the optimal timing for operation? Arch Surg 1996; 131: 540-545. 7. Lai ECS, Mok FPT, Tan ESY, Lo CM, Fan ST, You KT et al. Endoscopic biliary drainage for severe acute cholangitis. N Engl J Med 1992; 326: 1.582-1.586. 8. Marton KI, Doubilet P. How to image de gallbladder in suspected cholecystitis. Ann Int Med 1988; 109: 722-727. 9. Van den Hazel SJ, Speelman P, Tytgat GNJ, Dankert J, Van Leeuwen DJ. Role of antibiotics in the treatment and prevention of acute and recurrent cholangitis. Clin Infect Dis 1994; 19: 279-286. 10. Westphal J-F, Brogard J-M. Biliary tract infections. A guide to drug treatment. Drugs 1999; 57: 81-91. 68.- Masculino de 46 años de edad con cirrosis hepática tuvo dolor abdominal generalizado durante 24 h sin náuseas ni vómitos. Su temperatura es de 38.3°C y ha tenido distensión abdominal con onda de líquido claro. Hay hipersensibilidad difusa en la palpación abdominal. En la paracentesis se obtuvo líquido transparente con 816 leucocitos/mm 3 (85% polimorfonucleares, 15% linfocitos). La tinción de Gram no muestra bacterias. ¿Cuál de los siguientes diagnósticos es más probable? a) b) c) d)

Enfermedad ulcerosa péptica Pancreatitis Peritonitis primaria Colecistitis

El diagnóstico más probable es peritonitis primaria. Aunque es difícil diferenciar la peritonitis primaria (espontánea) por rotura de víscera hueca y contaminación peritoneal, la presencia de fiebre y la leucocitosis la ascitis sugiere alguna clase de infección peritoneal. La pancreatitis se caracteriza por dolor localizado intenso (mesoepigástrico), que se irradia a la espalda. En general, las náuseas y vómitos no se relacionan con pancreatitis aguda. El dolor abdominal en caso de colecistitis se sitúa en el cuadrante superior derecho y suele haber náuseas y vómitos. El absceso hepático tiende a ser un trastorno subagudo sin datos peritoneales prominentes. En caso de ascitis crónica infectada, las enfermedades ulcerosas pépticas son causa poco probable. Allen R. M. MMS Medicina Interna. 5ª. Edición. National Medical Series. Mc. Graw Hill. 2006. (capítulo 8 V E 1 a). 69.- Masculino de 31 años de edad que desde hace tiempo tiene episodios de ojo rojo con ausencia de secreción. Refiere leve sensación de cuerpo extraño y una carnosidad que crece y ya alcanzó la córnea. A la exploración se observa un tejido de neoformación nasal que invade 1mm de la córnea. El diagnóstico más probable es: a) b) c) d)

Carcinoma intraepitelial de conjuntiva Pterigion Pingüecula Nevo conjuntival

El pterigión constituye una hiperplasia fibrovascular de carácter benigno de la conjuntiva bulbar que invade la córnea,1-4 está clasificado dentro de las degeneraciones no involutivas o tumoraciones epiteliales benignas corneales.1,3,5 Se localiza en la conjuntiva bulbar cerca del limbo corneal en el área interpalpebral, a las 3 y 9 horas. Puede ser unipolar (solo afecta una parte) o bipolar, cuando afecta tanto la parte temporal como la nasal, es más frecuente en el lado nasal. También pueden ser unilaterales o bilaterales. Recibe su nombre por su aspecto de ala pequeña. Presenta una forma triangular, cuyo vértice mira hacia el área pupilar y cuya base se orienta hacia la carúncula en la localización nasal, y hacia el lado temporal en los localizados en el área temporal. Habitualmente muestra un crecimiento horizontal que puede llegar a afectar el eje visual.2,3,7 Actualmente, en los estudios sobre alteraciones ultraestructurales del pterigión, se considera una alteración inflamatoria y proliferativa de la superficie ocular.2 Se plantea que las células madre limbares se modifican con exposición crónica a la luz ultravioleta, por lo que hay una rotura de la barrera limbar que causa la invasión conjuntival del epitelio corneal. Otro mecanismo que se expone relacionado con las reacciones inmunológicas es una reacción de hipersensibilidad tipo I a elementos irritantes exógenos (polvo, viento, etc.) asociada a una inflamación local que causa un incremento en la producción de IgE.3

Un estudio reciente muestra muestra una asociación entre una película lagrimal inestable y el inicio de un pterigión. Referencias bibliográficas 1. Adamis AP, Stark T, Kenyon KR. The manegement of pterygium. Ophtamol Clin North Am. 1990;3(4):611 2. Klinworth GK. Chronic Actinic keratopathy, a condition associated with conjunctival elastosis (pingueculae) and typified by characteristic extracellular concretions. Am J Pathol. 1972;67:32. 3. Mac Kenzie FD, Hirst LW, Battistutta D. Risk analysis in the development of pterygia. Ophthalmology. 1992;99:1056-61. 4. Clear AS, Chirambo MC, Hutt MSR. Solar keratosis, ptert-gium, and squamous cell carcinoma of the conjunctiva in Malawi. Br J Ophthalmol. 1979;63:102-9.

70.- A 27-year-old woman has been sad for the last two weeks. She is fatigued a has a hard time concentrating at work. Just a few weeks earlier she was energetic an enthusiastic, and was able to work 10-12 hours a day with little sleep and go dancing at night. Her husband wants a divorce because he is tired of “these constant ups and downs”. The most accurate diagnosis is: a) b) c) d)

Borderline personality disorder Seasonal mood disorder Dissociative identity disorder Cyclothymic disorder

Criterios para el diagnóstico de F34.0 Trastorno ciclotímico (301.13) A. Presencia, durante al menos 2 años, de numerosos períodos de síntomas hipomaníacos y numerosos períodos de síntomas depresivo que no cumplen los criterios para un episodio depresivo mayor. Nota: En los niños y adolescentes la duración debe ser de al menos 1 año. B. Durante el período de más de 2 años (1 año en niños y adolescentes) la persona no ha dejado de presentar los síntomas del Criterio A durante un tiempo superior a los 2 meses.

C. Durante los primeros 2 años de la alteración no se ha presentado ningún episodio depresivo mayor, episodio maníaco o episodio mixto. Nota: Después de los 2 años iniciales del trastorno ciclotímico (1 año en los niños y adolescentes), puede haber episodios maníacos o mixtos superpuestos al trastorno ciclotímico (en cuyo caso se diagnostican ambos trastornos, el ciclotímico y el trastorno bipolar I) o episodios depresivos mayores (en cuyo caso se diagnostican ambos trastornos, el ciclotímico y el trastorno bipolar II). D. Los síntomas del Criterio A no se explican mejor por la presencia de un trastorno esquizoafectivo y no están superpuestos a una esquizofrenia, un trastorno esquizofreniforme, un trastorno delirante o un trastorno psicótico no especificado. E. Los síntomas no son debidos a los efectos fisiológicos directos de una sustancia (p. ej., una droga, un medicamento) o a una enfermedad médica (p. ej., hipertiroidismo). F. Los síntomas provocan malestar clínicamente significativo o deterioro social, laboral o de otras áreas importantes de la actividad del individuo.

71.- Masculino de 35 años de edad que acude a consulta externa refiere que desde hace más de 3 meses presenta una dermatosis diseminada a codos y rodillas con placas eritemato-escamosas. Sin antecedentes de importancia para el cuadro actual. ¿Cuál de los siguientes diagnósticos es el de mayor probabilidad? a) Dermatitis atópica b) Dermatitis seborreica c) Psoriasis d) Dermatitis de contacto La psoriasis es una enfermedad crónica que evoluciona en brotes de causa desconocida, que se caracteriza por placas eritemato escamosas en diferentes partes de la piel. Se presenta por igual en hombres y en mujeres, en todas las edades, predominando en jóvenes y más frecuentemente en personas de piel blanca (parece que la presencia de melanina protege contra la enfermedad. También es ampliamente reconocido que la psoriasis es una enfermedad familiar y hereditaria (diátesis psoriásica). Los sitios de predilección para que aparezcan las lesiones, son los salientes óseos como codos, rodillas, y la piel cabelluda así como la región sacrocoxígea.

Christophers E, Krueger G G. Psoriasis. En: Fitzpatrick TB, EISEN AZ, Wolff K. Dermatología en Medicina General Buenos Aires: Editorial Panamericana 1988. P. 585-591 Christophers E, Schubert C, Schröder J M. Psoriasis. Dermatología. 1992; 45. 72.- Masculino de 27 años que desde hace una semana inicia con erupción maculopapular que afecta fundamentalmente a tronco, muslos y palmas, de forma simétrica. Las lesiones son rojas claras. Refiere Adinamia, astenia, con dolor de garganta y cefalea leve, no refiere fiebre. Tras realizarle, entre otras pruebas, RPR y FTA-Abs, ambas son positivas en suero, se le diagnostica de sífilis. ¿Cuál de las siguientes afirmaciones es correcta? a) Si el paciente es VIH positivo la actitud no cambia. b) El tratamiento de elección es penicilina G benzatina. c) El tratamiento siempre debe incluir probenecid. d) Se trata de una sífilis primaria. La penicilina es el antibiótico de elección, con eficacia probada para cualquiera de los estadíos de la enfermedad. 1) Para la sífilis primaria se recomienda penicilina G benzatínica 2,4 MUI i/m en 1 sola dosis. Se prefiere repetir la dosis semanal de penicilina benzatínica (1 o 2 veces) cuando se presume que la personas no va a concurrir a los controles posteriores y en la embarazada. Con ello curan más de 95% de los pacientes. Después del tratamiento el VDRL se hace negativo en 1 año en casi todos los casos.

2) El mismo tratamiento también cura a la gran mayoría de los enfermos con sífilis secundaria, aunque algunos expertos recomiendan una segunda dosis con intervalo de 1 semana. El VDRL se negativiza en aproximadamente 2 años, pero en algunos pacientes las pruebas serológicas reagínicas siguen siendo reactivas, con niveles bajos de anticuerpos.

Para las personas alérgicas a la penicilina no embarazadas la alternativa es tetraciclina 2 g/d v/o fraccionados en 4 tomas diarias o doxiciclina 100 mg v/o c/12 horas, cualquera de ellas durante 14 días. Para personas que no toleran la tetraciclina, puede administrarse eritromicina 500 mg c/6 horas v/o por 14 días, aunque es menos eficaz y se han observado fracasos. Por eso cuando se usan drogas alternativas, el control posterior debe ser más estrecho. Si se sospecha mala adherencia al tratamiento o que el paciente no va a concurrir a los controles, se prefiere desensibilizarlo y tratado con penicilina G benzatínica. Otra alternativa es la ceftriaxona 1 g/d i/m o i/v por 10 días. 3) La sífilis latente precoz se trata de igual manera. 4) Para los pacientes con sífilis latente tardía, latente de duración no conocida y terciaria no neurosífilis, se recomiendan 7,2 MUI de penicilina G benzatínica i/m en 3 dosis con intervalo de 1 semana. En caso de alergia a la penicilina la alternativa es tetraciclina o doxiciclina en la forma dicha, por 4 semanas . Algunos expertos tratan a todos los que tienen sífilis cardiovascular con planes de neurosífilis. 5) No se recomienda penicilina G benzatínica para el tratamiento de la neurosífilis pues no alcanza niveles terapéuticos en LCR. El plan para neurosífilis tanto sintomática como asintomática es de penicilina G cristalina 12 a 20 MUI/d i/v en 6 dosis diarias, por 10 a 14 días. Se menciona la alternativa de ceftriaxona 1 g/d i/m o i/v por 14 días, pero se han observado fallas terapéuticas en pacientes infectados VIH con neurosífilis, sífilis latente o presunta sífilis latente. Aunque no se recomienda la ceftriaxona para el tratamiento de la neurosífilis, puede aceptarse como antibiótico de alternativa para pacientes con neurosífilis y sífilis secundaria concomitante. Wardropper AG, Snow M. Neurosyphilis and HIV infection. Int J STD AIDS. 1994; 5(2):146-8. O'Mahony C, Rodgers C.A., Mendelsohn S.S., Sissons G., et al. Rapidly progressive syphilis in early HIV infection. Int J STD AIDS. 1997; 8(4): 275-78. Hook EW, Marra CM. Acquired syphilis in adults. N Eng J Med. 1992; 326 (16):1060-68. Guidelines for treatment of sexually transmitted disease. MMWR 1998; 47(RR-1):1-118. Hutchinsom CM, Hook EW. Sífilis en adultos. Clin Med N. A.1990; 6: 1451-76. Musher DM. Syphilis. Gorbach, Bartlett, Blacklow. Infectious diseases. Ed.Saunders. 1992: 822-28.

Charles D Ch. Sífilis. Sanford J P, Tyrrell D A J, Weller T H, Wolff S M. Infecciones obstétricas y perinatales. Doyma. 1994:252-69. Giovangrandi Y, Costa JM, Malka D, Belein V. Maladies infectieuses au cours de la grossesse (II). Rev Prat (Paris).1995; 45:2215-37.

73.- Al explorar a un paciente masculino de 40 años de edad quien presenta una otoscopía sin alteraciones, una hipoacusia derecha, con Rinne negativo derecho y positivo izquierdo y Weber con lateralización al lado derecho. El diagnóstico más probable es: a) Otitis serosa. b) Neuronitis vestibular. c) Hipoacusia súbita. d) Otosclerosis. OTOSCLEROSIS INTRODUCCION: OSTEODISTROFIA DE LA CAPSULA OTICA DEL LABERINTO. CLINICA:

HIPOACUSIA: HIPOACUSIA PROGRESIVA TRANSMISIVA UNILATERAL , EN LA CUAL LA PACIENTE ENTIENDE MEJOR EN AMBIENTES RUIDOSOS.(PARACUSIA DE WILLIS) PROGRESIVA: AVANZA CON EL TIEMPO, EMBARAZOS. TRANSMISIVA EN INICIO. LUEGO PUEDE HACERSE NEUROSENSORIAL. UNILATERAL EN INICIO. LUEGO BILATERAL EN > DEL 75% PARACUSIA DE WEBER: NO OYE CON MASTICACION. VERTIGO : 10-20%. . SENSACION DE “BORRACHERA” OCASIONAL. ACUFENO: 75%. DE TONO GRAVE. MUY MOLESTOS. (ANSIOLITICOS) ALTS PSIQUICAS: IRRITABILIDAD, INTROVERSION, .HIPOCONDRIACOS,...NEUROSIS. EXPLORACION: OTOSCOPIA: NORMAL (MANCHA DE SCHWARTZE 10%) ACUMETRIA: RINNE NEGATIVO. WEBER LATERALIZADO AL LADO MAS AFECTO SCHWABACH : ALARGADO.(VIA OSEA PACIENTE Y EXPLOR) (TRIADA DE BEZOLD) PR.GELLE: NEGATIVO (DIAPASON EN FRENTE—PRESION CON PERA DE POLITZER EN CAE.) •



• •

• •

• • •

• •



• •

IMPEDANCIOMETRIA: CURVAS CENTRADAS Y EN OCASIONES ALGO APLANADAS. REFLEJO AUSENTE O INVERTIDO.(NO RECRUITMENT) TAC: SI VERTIGO. SE VE FOCO OTOESPONGIOTICO. AUDIOMETRIA: (+AUDIO VERBAL) ESTADIOS: FORMA TIPICA O DE POLITZER-SIEBENMANN: LA MAS FRECUENTE. PROGRESION LENTA. ♣

1) HIPOACUSIA DE TRANSMISION CON UMBRAL DE 20-40 Dbs,PEOR EN FRECUENCIAS GRAVES. 2) HIPOACUSIA MIXTA: TRANSMISION DE 40-60 DBS Y PERCEPCION DE 20-30DBS (ESCOTOMA EN 2000Hzs-MUESCA DE CARHART). 3) SORDERA CON GRAN COMPONENTE NEUROSENSORIAL DE 6080DBS OTOSCLEROSIS DE LERMOYEZ: HIPOACUSIA TRANSMISIVA PURA. OTOSCLEROSIS COCLEAR DE MANASSE: DESDE INICIO COMPONENTE NEUROSENSORIAL.PURA ♣ ♣

Bibliografía: • •







House J. Otosclerosis. Otolaryngol Clinics 1993;26(3):323-502. Jerger J. Clinical experience with impedance audiometry. Arch Otolaryngol 1970;92:311 Lempert J. Improvement in hearing in cases of otosclerosis: A new, one stage surgical technique. Arch Otolaryngol 1938; 28:42-97. Lippy WH, Schuring AG. Treatment of the inadvertently mobilized footplate. Otolaryngol Head Neck Surg 1973; 98:80-81. Meyer S. The effect of stapes surgery on high frequency hearing in patients with otosclerosis Am J Otol 1999; 20:36-40.

74.- Masculino de 4 años de edad con antecedentes de otitis media a los doce meses de edad, actualmente cursa con otalgia, otorrea e hipoacusia, acompañado de malestar general. A la exploración en conducto izquierdo se aprecia Hiperemia timpánica, hipervascularización y engrosamiento timpánico. El tratamiento de elección en esta patología es: a) b) c) d)

Antihistamínicos de forma exclusiva Solo medidas generales Drenaje, AINES y esteroides locales Antibióticos, antihistamínicos y AINES.

Otitis media que se acompaña con secreción y signos locales, sistémicos, o ambos. Instituto Nacional de la Nutrición. “Salvador Zubiran” Manual de terapéutica médica y procedimientos de urgencias. Cuarta edición. Pag. 443 – 450. Mac Graw-Hill Interamericana. México. La otitis media se define como la inflamación del oído medio que se acompaña de secreción y signos locales, sistémicos, o ambos, de enfermedad aguda. Esta enfermedad predomina en la infancia, sin embargo también existe en el adulto. Su distribución tiene una clara periodicidad estacional, sobre todo en otoño e invierno. Los agentes de mayor prevalencia en nuestro país son: Streptococcus pneumoniae, Haemophilus  influenzae, Morexella catarrhalis  estreptococo del grupo A y S. aureuse . Los antibióticos más aceptados en estos casos son la amoxicilina, la amoxicilina con clavulanato, la calritromicina y el trimetropim con slfametoxazol. Se acepta el uso de antihistamínicos y antiinflamatorios no esteroideos para disminuir la congestión de la mucosa para resolver la obstrucción de la trompa faríngotimpánica; en realidad no se ha demostrado efecto sobre la duración de los síntomas. El drenaje del derrame solo esta indicado cuando éste persiste por más de tres semanas. 75.- Masculino de 62 años, refiere disnea de moderados esfuerzos y se le ausculta un primer tono fuerte, chasquido de apertura y soplo diastólico con refuerzo presistólico y en el ECG presenta ondas P con signos de crecimiento de la aurícula izquierda. El diagnóstico más probable es: a) b) c) d)

Doble lesión mitral en ritmo sinusal. Estenosis mitral en ritmo sinusal. Insuficiencia aórtica en ritmo sinusal. Estenosis mitral en fibrilación auricular, probablemente severa.



Estenosis mitral:

Los hallazgos se concentran en el diástole: - Después del 2 ruido se ausculta un chasquido de apertura (momento en que se abre la válvula mitral) - Luego viene un soplo en decrescendo ("rodada mitral"): corresponde a la fase de llenado rápido. - Acontinuación, en pacientes en ritmo sinusal se puede escuchar un refuerzo presistólico, que es un soplo en crescendo  y que se escucha inmediatamente antes del primer ruido (se debe a la contracción de las aurículas).

- El primer ruido, tiende a ser de ayor intensidad (la explicación sería que cuando se comienzan a contraer los ventrículos al comienzo del sístole, los velos de la válvula mitral se encuentran en su máxima separación ya que la aurícula estaba recién contrayéndose, y esto, unido a las características de los velos, haría que el primer ruido sea más intenso; sería equivalente a un "portazo").

Estos ruidos y soplos se auscultan mejor en el ápex. La auscultación puede resultar más nítida si se pone a la persona en decúbito semilateral izquierdo y se le pide que bote el aire  y se mantenga un rato sin respirar. Convendría auscultar tanto con la membrana como con la campana del estetoscopio. Si la estenosis mitral ha generado hipertensión pulmonar secundaria, el cierre de la válvula pulmonar (P2) es más intenso y el segundo ruido se ausculta desdoblado porque se retrasa el cierre de la válvula pulmonar. También puede ocurrir que el ventrículo derecho se hipertrofie y sea palpable. Cuando el soplo de "estenosis mitral" se debe a la inflamación de los velos por una

enfermedad reumática activa recibe la denominación de soplo de Carey-Coombs. Manual de semiología .2007 Universidad Católica de Chile

76.- Al realizar un ensayo clínico controlado doble ciego para evaluar un nuevo antiinflamatorio, ¿Qué condición deben cumplir los participantes del estudio? a) Ni el observador ni los sujetos conocen cual grupo recibe el nuevo medicamento y cual el placebo b) Ni el grupo de estudio ni el grupo control conocen a los observadores c) Los sujetos del grupo control no conocen a los sujetos del grupo en estudio d) La asignación del tratamiento no es conocida por los pacientes

Para considerar el efecto de placebo y reducir los sesgos debido a las concepciones de los pacientes y los investigadores el estudio puede conducirse bajo un patrón ciego. En un estudio doble ciego, la asignación al tratamiento no es conocida por los pacientes ni por los médicos. Greenberg R. S; Epidemiología médica, Manual Moderno, 2ª. Ed. Pág. 116 

77.- A 55-year-old woman presents to the Emergency Department with 8 hours of severe left lower quadrant abdominal pain, focal peritonitis, and guarding on physical examination. She has nonspecific abdominal radiographs and a leukocytosis. Which of the following is indicated at this point? a) b) c) d)

IV antibiotics. CT scan. Colonoscopy. Surgery.

ABSCESO DIVERTICULAR: • La formación de un absceso diverticular complicado depende de la capacidad de los

tejidos pericólicos de controlar (localizar) la diseminación del proceso inflamatorio. • En general, los abscesos intra -abdominales se forman por: o Fuga anastomótica = 35% o Enfermedad diverticular = 23% • Signos/Síntomas

o fiebre+/- leucocitosis a pesar de antibióticos adecuados, tumoración dolorosa • Tratamiento

o Absceso pericólico pequeño - 90% responde a los antibióticos y manejo conservador. o Drenaje percutáneo de los abscesos (DPA) es el tratamiento de elección para las colecciones simples, bien definidas. o 100% de los abscesos uniloculares simples se resolvieron con DPA y antibióticoterapia. Schauer P, Ramos P, Ghiatas A, Sirinek K. Virulent diverticular disease in young obese men.Am J Surg; 164:443-8. Pubmed-Medline 78.- Se trata de femenio de 28 años con desarrollo de abceso puerperal en mama izquierda, si patológicos sistémicos de importancia para su actual padecimiento, la medida inicial más importante es: a) b) c) d)

Antibióticos. Drenaje quirúrgico del absceso Instituir una terapia con antiinflamatorios. Suspender la lactancia.

En el desarrollo del absceso puerperal hay inicialmente dolor, enrojecimiento, induración; en las fases tempranas de la celulitis los antibióticos son muy efectivos, pero una vez formado el absceso, al igual que en el absceso no relacionado con la lactancia, se impone el drenaje quirúrgico. El drenaje debe ser amplio, y en los abscesos no puerperales es necesario abrir los tabiques que con frecuencia se forman for man en la cavidad del absceso. Muchos autores preconizan el simple drenaje sin cubrimiento antibiótico. Si se usan antibióticos, éstos deben ser debidamente seleccionados de acuerdo con el germen, recordando que el más frecuente es el S taphylococcus aureus .

Beller FK, Galask RP. Infections of the breast. En: Infectious Diseases. Edited by SL Gorbach, JG Bartlett, NR Blacklow. WB Saunders Co. Philadelphia, 1992 Benson EA. Management of breast abscesses. World J Surg 23:753, 1989 Bundred NJ, Dixon JMJ, Lumsden AB, et al. Are the lesions of duct ectasia sterile? Br J Surg 72:844, 1985 Ingham HR, Freeman R, Wilson RG. Anaerobic breast abscesses. Lancet 1:165, 1979 Scholefield JH, Duncan JL, Rogers K. Review of hospital experience of breast abscesses. Br J Surg 74:469, 1987

79.- Se realiza una endoscopia digestiva a un paciente de 57 años, se reporta el diagnóstico de sospecha de “esófago de Barrett” ¿Cuál es entre las siguientes, la conducta más adecuada? a) Esperar el resultado de la biopsia antes de tomar decisiones, para corroborar y tener seguridad del diagnóstico b) Indicar tratamiento médico con inhibidor de la bomba de protones 40 mg día en ayuno durante 1 año y regresar a valoración. c) Prepararlo para la realización de funduplicatura. d) Realizar esofaguectomía. Esófago de Barret. La Enfermedad por Reflujo Gastroesofágico es una de las causas más frecuentes de consulta médica en la práctica diaria y esta se puede acompañar de Esofagitis por Reflujo (es la inflamación del esófago causada por el reflujo del contenido gástrico al esófago), La Esofagitis por Reflujo se diagnostica mediante Endoscopía, y este procedimiento permite la biopsia, la cual nos permite confirmar la presencia de Esófago de Barret, una complicación poco frecuente pero de gran relevancia clínica. El Esófago de Barret es una lesión premaligna, que predispone al Adenocarcinoma Esofágico, patología neoplásica que ha aumentado su incidencia en los últimos años, incluso desplazando al Carcinoma Epidermoide. DIAGNOSTICO. IPresentación Clínica. La presencia de EB  no provoca síntomas por sí mismo. Más bien, los síntomas son una consecuencia de la Enfermedad por Reflujo Gastro-Esofágico  (ERGE)  o sus complicaciones. La mayoría de los pacientes tienen antecedentes prolongados de síntomas de RGE como  pirosis y regurgitación (2, 6, 10). Otros pacientes, aproximadamente la tercera parte, se presentan oligo ó asintomáticos  (2, 8, 13), lo que sugiere una disminución de la sensibilidad al reflujo ácido por parte del epitelio de Barret. De hecho, más del 90% de los pacientes con EB  no buscan atención médica y el trastorno pasa inadvertido hasta que el proceso se complica por el desarrollo de cáncer, debutando con disfagia (por formación de estenosis) o con hemorragia digestiva (por ulceraciones profundas de la mucosa lesionada). (10) II- Fibroendoscopía Digestiva Alta con toma de Biopsia. La endoscopia con biopsias dirigidas constituye el patrón oro para el diagnóstico del EB (2, 6). Sin embargo, debido a la gran cantidad de pacientes con RGE y la baja frecuencia de EB , no parece costo-efectivo el realizar endoscopia a todos ellos (13). Se ha sugerido que los siguientes pacientes con RGE deben ser sometidos a una endoscopia (9): (9 ): · RGE complicado (disfagia, estenosis, úlcera, hemorragia); · RGE con esofagograma que muestre patrón pa trón reticular o seudomembranas; · RGE con sintomatología persistente a pesar del tratamiento; tr atamiento; · RGE asociado a esclerodermia.

Se sospecha EB  en la endoscopia por la presencia de epitelio color naranja a rojo, aterciopelado, de tipo gástrico; que contrasta con el color rosado blanquecino del epitelio esofágico normal (1, 2, 4, 8). La línea de transición entre ambos epitelios (cambio mucoso) puede ser regular (circunferencial) ó, más frecuentemente, irregular (en forma de lengüetas o islotes)(1, 2, 8,). La extensión de la metaplasia también es variable, pudiendo abarcar desde 2 cm de longitud a partir de la unión gastro-esofágica (UGE) o puede extenderse incluso hasta el esófago cervical (1). Para confirmar el diagnóstico debe obtenerse la biopsia. Las muestras deben tomarse, fundamentalmente, del límite de la UGE y hasta 1 ó 2 cm por debajo de la misma, ya que esta es la zona de mayor riesgo de desarrollar un Adenocarcinoma, y es en esta parte más proximal del segmento metaplásico donde se sitúa, principalmente, el epitelio columnar especializado (2). Por supuesto que deben tomarse muestras para biopsias de todo el segmento metaplásico y, especialmente, de aquellas zonas en las que se observe alguna alteración macroscópica. Microscópicamente pueden identificarse tres tipos de epitelio de Barret (1, 2, 4, 8) Metaplasia gástrica fúndica : es similar al epitelio del cuerpo o del fondo gástrico, y presenta células parietales y principales; Metaplasia gástrica cardiaca o transicional : como el epitelio del cardias gástrico, que exhibe acúmulos profundos de glándulas mucosas y criptas; c riptas; y Metaplasia tipo intestinal  o también llamado epitelio columnar especializado : que tiene características de la mucosa gástrica y de la intestinal, que presenta células caliciformes dispersas entre las células cilíndricas. BIBLIOGRAFIA 1. Badaloni AE. Enfermedad por reflujo gastroesofágico. En: Ferraina P, Oria A, ed. Cirugía de Mitchans, Vol 1. 5ª edición. Buenos Aires, Argentina. Editorial El Ateneo. 392-399. 2. Cucarella JF. Esófago de Barret. Gaceta Gaceta de la sociedad española de patología digestiva [en línea] 1999. [fecha de acceso 10 de noviembre de 2004] . URL. 3. Beers MH. Trastornos del esófago. Cáncer esofágico. En: El Manual Merck, 17ª edición. Edición electrónica en CD-ROM. Madrid, España: Ediciones Harcourt: 1999; sección 3, cap 20. 4. McGarrity TJ. Barret`s esophagus: the continuing conundrum. Surveillance should be confined to the surgically fit. BJM 2000; 321: 1238-1239 5. Heading R. ¿Es el seguimiento endoscópico del esófago de Barret una pérdida de tiempo y esfuerzo? XXX Congreso Chileno de Gastroenterología. Medwave. Año 4. Nº 2. [en línea] Edición Marzo 2004. [fecha de acceso 28 de diciembre de 2004]. URL. 6. Murra Saca JA. Esófago de Barret. Atlas de Video Endoscopia Gastrointestinal de El Salvador.[en línea] 2004 [fecha de acceso 10 de noviembre de 2004] . URL. URL. Disponible en: 7. Ayre AM, Benitez Fernandez A, Cocco JE, y col. col . Tratamiento del cáncer de esófago: revisión. Revista de Postgrado de la VI Cátedra de Medicina [en línea] 2003 [fecha de acceso: 15 de enero de 2005]; 126: 37-41. URL. Disponible en:

8.

Crawford JM. El tracto gastrointestinal. Esófago de Barret. En: Cotran, Kumar, Robbins, ed. Patología Estructural y Funcional. 5ª edición. Madrid, España: Ediciones McGraw-Hill-Interamericana. 845 9. Rodriguez A. Esófago de Barret. Boletín de la Escuela de Medicina. Universidad Católica de Chile. [en línea] 1998 [fecha de acceso: 28 de diciembre de 2004] Vol 27. Nº 1. 1998. URL. Disponible 80.- Lactante menor próximo a cumplir 2 meses, atendido en consulta para control de niño sano. Se refiere asintomático. E.F.: dentro de límites normales. Antecedente: esquema de vacunación completo para la edad. De acuerdo a la cartilla nacional de vacunación, que vacunas ya recibió: a) b) c) d)

Polio y bcg. Hepatitis b y bcg. Solo bcg. Polio y hepatitis b.

81.- Femenino de 34 años que inicia tratamiento con sulfato de magnesio por presentar eclampsia, se presentan signos de sobre dosificación, el antídoto que debe emplearse en esta paciente es: a) Gluconato cálcico b) Nitroprusiato. c) Simpaticomiméticos. d) Carbonato sódico.

NIVEL DE PRIMER CONTACTO (ATENCION PRIMARIA) Se debe instruir a todas las embarazadas que deben acudir inmediatamente a un centro de salud en cualquiera de los siguientes casos: • • • •

Edema que se desarrolla rápidamente (en pocos días) Cefalea severa y persistente Dolor en la región abdominal superior Visión borrosa

Se debe realizar la medición de la presión arterial y un análisis de orina para la detección de proteinuria a las mujeres que acudan a centros de salud presentando estos síntomas.

Convulsiones: Si se asiste a una mujer con eclampsia en un centro de atención primaria, 1. Deben mantenerse las vías respiratorias permeables. 2. Se debe colocar a la mujer de costado (posición decúbito lateral izquierda) para evitar la aspiración del vómito u otras secreciones. 3. Si es posible, se debe establecer una vía intravenosa. 4. Se debe administrar sulfato de magnesio. Monitoreo de la administración de sulfato de magnesio: Durante el tratamiento con sulfato de magnesio, se recomienda realizar un control cada 4 horas, como mínimo, para detectar la presencia de: Reflejo rotuliano, frecuencia respiratoria superior a 16 por minuto, volumen de orina >100 ml en las 4 horas previas.

- Sobredosis de sulfato de magnesio: Todo centro de salud que utilice sulfato de magnesio debe disponer de ampollas de gluconato de calcio (1 g) como antídoto para la sobredosis de dicho fármaco. •



Se sugiere medir la presión arterial y administrar antihipertensivos según corresponda. Convulsiones recurrentes: en caso de convulsiones recurrentes, se administran otros 2 a 4 g de sulfato de magnesio por vía IV en el lapso de 5 minutos, tanto para el régimen IM como el IV; la dosis se determina en función del peso de la paciente.

El sulfato de magnesio es un fármaco usado en el control de las convulsiones eclámpticas, para suprimir o controlar las contracciones uterinas sean estas espontáneas o inducidas, y como broncodilatador luego del uso de beta agonistas y agentes anticolinergicos. También tiene indicación como terapia de reemplazo en la deficiencia de magnesio, como laxante para reducir la absorción de tóxicos del tracto gastrointestinal. El sulfato de magnesio esta ganando popularidad como tratamiento de inicio en el manejo de algunas arritmias, particularmente en Torsades de Pointes, y en arritmias secundarias a sobredosis de antidepresivos tricíclicos o toxicidad digitálica. Esta también considerado clase Ila (probable beneficio) para la fibrilación ventricular refractaria y la taquicardia ventricular, luego de la administración de dosis de lidocaina y bretilio.

FARMACODINAMIA El sulfato de magnesio tiene la capacidad de alterar la excitabilidad de la fibra miometrial, afecta el acoplamiento excitación – contracción y el proceso mismo de contracción, inhibe la entrada de calcio al sarcoplasma y reduce la frecuencia de los potenciales de acción. Inhibe también la liberación de acetilcolina. Por ser estas acciones comunes en las fibras musculares se pueden ver afectadas también la musculatura voluntaria e incluso las fibras miocárdicas.(1)

Bibliografía: Graves C. Fármacos que contraen o relajan el útero. En: Hardman J, Limbird L, Molinoff P, Ruddon R, Goodman A, eds. Goodman & Gilman. Las Bases Farmacológicas de la Terapéutica. 9 ed. México DF: McGraw-Hill Interamericana; 1996. pp. 1012-3.

82.- Se trata de paciente de 34 años que cursa con 39 SDG; a la exploración física reflejos patelares hiperactivos, inquieta, se reportan cifras de TA 145/95, se realiza laboratorio que reporta proteinuria 2+,. El diagnóstico más probable es: a) Glomerulonefritis aguda b) Hipertensión esencial c) feocromocitoma d) Preeclampsia La hipertensión es la complicación médica más común del embarazo , aunque para algunos autores es la segunda complicación médica del embarazo sólo después de la anemia; es más frecuente en jóvenes durante el primer embarazo  y en nulíparas de mayor edad, hipertensas previas y diabéticas. En México, también es la complicación más frecuente del embarazo, la incidencia es de 47.3 por cada 1 000 nacimientos y es además, la primera causa de ingreso de pacientes embarazadas a las unidades de terapia intensiva (debido a hemorragia masiva, para recibir soporte hemodinámico), según la secretaría de salud (2001) la mortalidad por complicaciones del embarazo ocupa el 15º lugar en la mortalidad hospitalaria en general. Además, la tasa de preeclampsia se ha incrementado 40% en el periodo entre 1990 y 1999 y constituye hasta 40% de los partos prematuros iatrogénicos. .

Preeclampsia La preeclampsia es un síndrome clínico caracterizado por hipertensión con disfunción orgánica múltiple, proteinuria, edemas. Es definida como un incremento de al menos 140/90 mmHg después de la semana 20 de gestación, un incremento en la presión sanguínea diastólica de al menos 15 mmHg respecto a un nivel previo a la semana 20 combinado con proteinuria (> 300 mg en 24 horas). Las mediciones de la presión arterial citadas deben ser medidas al menos 2 ocasiones con por lo menos 6 horas de separación. La proteinuria puede ser una toma simple de orina al azar que indique al menos 30 mg/dL 3 ó ++ en dos muestras de orina 1 según el tipo de prueba. El criterio del incremento de 30 mmHg en la presión sistólica y/o 15 mmHg en la presión diastólica respecto a valores previos a la semana 20 de gestación ha sido eliminado por ser poco específico15

1.Myers JE, Baker PN. Hupertensive diseases and eclampsia. Curr Opin Obstet Gynecol 2002; 14: 119-125. 2. Tierney, McPhee, Papadakis. Diagnóstico clínico y tratamiento 2003. 38ª ed, México, Manual Moderno, 2003: 770-773. 3. Wilson MI, Goodwin TM, Pan VI, Ingles SA. Molecular epidemiology of preeclampsia. Obstet and Gynecol Survey 2003; 58(1):39-66.

4. Burrow GM. Complicaciones médicas durante el embarazo. 4ª ed, México, McGraw-Hill panamericana: 1996: 1-25. 5. Guyton AC, Hall JE. Embarazo y lactancia en: Tratado de fisiología médica, 10ª ed, México, McGraw-Hill Interamericana 2001: 1135-45. 6. Vaticon D. Fisiología de la fecundación, embarazo, parto y lactancia, en: Tresguerres JAF. Fisiología Humana. México, Interamericana McGraw-Hill, 1992: 1086-1109. 7. Pridjian G, Puschett JB. Preeclampisa. Part 1: Clinical and Pathophysiologic Considerations. Obstet and Gynecol Survey 2002; 57 (9): 598-618. 8. Pridjian G, Puschett JB. Preeclampisa. Part I1: Experimental and Genetic Considerations. Obstet and Gynecol Survey 2002; 57 (9): 619-40. 9. IMSS. Embarazo de alto riesgo. Guía diagnóstica terapéutica. Rev Med IMSS 1998; 36(1):45-60.

83.- Masculino de 10 años de edad que inició su padecimiento hace 24 hrs al presentar, posterior a la ingesta de pescado, evacuaciones diarreicas, dolor abdominal, náusea y vómito; acompañándose de cefalea, parestesias periorales y bradicardia. El día de hoy se agregó la presencia de ataxia y refiere que se quema al tocar el agua fría. ¿Cuál es el agente etiológico más probable? a)  Vibrio cholerae 

b) Ciguatera c) Agente Norwalk d)  StaphIlococcus aeurus  U.S. Food & Drug Administration - Center for Food Safety & Applied Nutrition Foodborne Pathogenic Microorganisms and Natural Toxins Handbook. La intoxicación por ciguatera se produce por ingerir pescados como dorado, barracuda, cubera, etc. contaminados con macroalgas con ciguatoxina-1. la evolución de la intoxicación es la siguiente: en las primeras 12 horas se presentan manifestaciones gastrointestinales como dolor abdominal, náusea, vómito y diarrea; posteriormente se agregan datos neurológicos como calambres, parestesias, mialgias, fatiga, ataxia, y disgeusia y finalmente alteraciones cardiovasculares (bradicardia, taquicardia o hipotensión.) Los datos principales son las parestesias periorales y la inversión de la sensación térmica, es decir sentir frío al tocar objetos calientes y viceversa. Además se puede presentar ceguera temporal, parálisis de los músculos faciales, oftalmoplejia, espasticidad, delirio, sensación de pérdida de los dientes, exacerbación del acné, sialorrea, caída del cabello, uñas y descamación de las piel.

84.- Masculino de 6 años acude al servicio de urgencias por salida de áscaris por el ano. A la exploración física presenta dificultad y sibilancia respiratoria, abdomen con hepatomegalia y distensión abdominal, se confirma el diagnóstico por USG. El tratamiento de elección en este paciente es: a) b) c) d)

Secnidazol vía oral Metronidazol vía oral Salbutamol Inhalado Albendazol vía oral

Tratamiento Se realizará tratamiento farmacológico por vía oral, siendo las drogas de elección el albendazol, el pamoato de pirantel y oxantel y la clásica piperacina La ascaridiasis intestinal es una patología ocasionada por un helminto que parasita el tubo digestivo del ser humano. Es la parasitosis más frecuente y cosmopolita de todas las helmintiasis humanas, se calcula más 1.5 billones de portadores en el mundo de los cuales 51 millones de afectados son niños. La gran mayoría de los casos cursa en forma asintomático o pueden producirse signos inespecíficos como dolor abdominal difuso o ir acompañado en los casos crónicos por signos de desnutrición, déficit de crecimiento y retardo del aprendizaje. Los efectos patológicos producidos por Áscaris  en el organismo humano, se presentan en varios sitios de acuerdo a la localización de las diversas formas evolutivas. Las larvas al pasar por el pulmón producen ruptura de los capilares y de la pared alveolar. Cuando ocurre en forma masiva da origen al síndrome de Löeffler que se caracteriza por lesiones múltiples de los alvéolos donde producen granulomas de cuerpo extraño, el cual se observa a los rayos X como opacidades diseminadas. Cuando el número de parásitos es grande puede ocasionar complicaciones tales como obstrucción intestinal, vólvulos, y perforación visceral con peritonitis, complicaciones que deben ser resueltas en forma quirúrgica. Las mayores complicaciones se presentan por las migraciones de Áscaris  adultos a diferentes sitios del organismo. La invasión del parásito al árbol biliar, puede causar ictericia obstructiva, colecistitis acalculosa, colangitis, pancreatitis, abscesos hepáticos. Se han descrito diversas modalidades de tratamiento, La migración masiva de parásitos hacia el árbol biliar se asocia con procesos infecciosos, para lo cual deben administrarse antibióticos aun en forma profiláctica. La administración de albendazol se considera el tratamiento antihelmíntico de elección por su mayor excreción biliar. En un bajo porcentaje (21.2%) se logra expulsar los parásitos de la vesícula biliar.

Si no se logra expulsarlo, si persiste la sintomatología o si el paciente empeora, es necesario someterlo a una laparotomía exploradora para realizar colecistectomía. Los pacientes pueden caer en una septicemia fatal, si no llevamos a cabo la terapéutica en forma adecuada, lo que hace que la evolución y el pronóstico empeore. La parasitosis es uno de los grandes problemas de la salud pública. La OMS la considera una de las principales causas de morbilidad; estrechamente ligada a la pobreza y relacionada con inadecuada higiene personal y de los alimentos crudos, falta de servicios sanitarios, falta de provisión de agua potable y contaminación fecal del ambiente. Infecta a personas de todas las edades, pero la sufren principalmente los niños, a lo que les causa trastornos del crecimiento y desarrollo. Desde hace años la OMS propone como solución, aunque sea parcial, el uso de tratamientos antihelmínticos masivos y reiterados, sin diagnóstico parasitológico previo individual, en aquellas comunidades que tengan una elevada prevalencia de geohelmintos (>50%).

Bibliografía: Cook GC, Zumla AI (edit). Manson´s Tropical Diseases. 21th ed. London: Elsevier Science; 2002 García LS. Diagnostic Medical Parasitology. 4th. Ed. Washington: American Society for Microbiology; 2002 Gill G. Lectura Notes on Tropical Medicine. 5th ed. Blackwell Scientific Pub; 2004 Mensa JM, Gatell MT, Jiménez de Anta G, Prats A, Dominguez-Gil A. Guía terapeútica antimicrobiana. 14ª ed. Barcelona: MASSON; 2004. Murray PR, Baron EJ (Edit.)Manual of clinical microbiology. 8th edit. Washington: ASM Press; 2003 Villa Luis F. Guía de Terapia Farmacológica. Medimecum. Barcelona: ADIS Internacional; 2005. 85.- Se trata de paciente femenino de 42 años diagnosticado con síndrome de Cushing al encontrar una ACTH muy baja o suprimida, la primera posibilidad de diagnóstico es:

a) b) c) d)

Enfermedad hipotálamo-hipofisiaria Hiperplasia suprarrenal congénita Adenoma suprarreanal Adenoma hipofisiario

Aproximadamente 20 a 25% de los pacientes con síndrome de Cushing tienen una neoplasia suprarrenal. En la mayoría de estos casos existe una producción autónoma de cortisol por la neoplasia, lo que conduce a descenso en los niveles de ACTH a rangos indetectables o menores a 2 pmol/L o 10 pg/ml, por retroalimentación negativa de la secreción de ACTH. Williams GH, Dluhy RG. Enfermedades de la corteza suprarrenal. En Jameson JL (ed): Harrison. Endocrinología. 1a ed. Madrid. MacGraw-Hill España, 2006: 126-127.

86.- Masculino de 68 años con antecedentes de dolor lumbar de 1 mes meses de evolución asi como malestar generalizado. Refiere cefaleas, visión borrosa, sed intensa y molestias digestivas. Con datos de deshidratación, TA de 170/100 mmHg. Presenta una hemoglobina de 9.5 g/dl, VSG 90, acidosis metabólica, creatinina 2.7 mg/dl, proteínas totales 10.2 con un pico monoclonal de IgG tipo l, y una calcemia de 15 mg/dl. Se inicia tratamiento de urgencia el cual incluye el uso de esteroides, difosfonatos, expansión con suero salino y por sus condiciones el uso del siguiente diurético. a) Furosemida. b) Clortalidona. c) Acetazolamida. d) Hidroclorotiazida. DESCRIPCION La furosemida es un diurético de asa de la familia de las sulfonamidas utilizado en el tratamiento del edema asociado a la insuficiencia cardíaca congestiva, cirrosis y enfermedad renal, incluyendo el síndrome nefrótico. También se utiliza en el tratamiento de la hipertensión ligera o moderada y como adyuvante en las crisis hipertensivas y edema pulmonar agudo. La furosemida es empleada, asimismo, para el tratamiento de la hipercalcemia. Pertenece al grupo de los diuréticos de alto techo. Mecanismo de acción: el mecanismo de acción de la furosemida no es bien conocido. La furosemida no se une a los grupos sulhidrilo de las proteínas renales como hace el ácido etacrínico, sino que parece ejercer su efecto diurético inhibiendo la resorción del sodio y del cloro en la porción ascendiente del asa de Henle. Estos efectos aumentan la excreción renal de sodio, cloruros y agua, resultando una notable diuresis. Adicionalmente, la furosemida aumenta la excreción de potasio, hidrógeno, calcio, magnesio, bicarbonato, amonio y fosfatos. In vitro, la furosemida inhibe la anhidrasa carbónica pudiendo ser este efecto el responsable de la eliminación del bicarbonato. La furosemida no es un antagonista de la aldosterona. Después de la administración de furosemida disminuyen las resistencias vasculares renales aumentando el flujo renal, occurriendo lo mismo en las resistencias periféricas, lo que se traduce en una reducción de la presión en el ventrículo izquierdo. Si inicialmente la furosemida tiene un efecto antihipertensivo debido a una reducción de la volemia aumentando la velocidad de filtración glomerular y reduciendo el gasto cardíaco, más tarde el gasto cardíaco puede volver a su valor inicial pero las resistencias periféricas permanecen bajas, lo que resulta en una reducción de la presión arterial.

Farmacocinética: La furosemida se administra por vía oral y parenteral. La absorción oral de este fármaco es bastante errática y es afectada por la comida, si bien esta no altera la respuesta diurética. La diuresis se inicia a los 30-60 minutos después de la administración oral y a los 5 minutos después de la administración intravenosa. El fármaco se une extensamente a las proteínas del plasma (95%), atraviesa la barrera placentaria y se excreta en la leche materna. La furosemida experimenta un mínimo metabolismo en el hígado eliminándose en su mayor parte en la orina. Aproximadamente el 20% de la dosis se excreta en las heces, si bien este porcentaje puede aumentar hasta el 98% en los pacientes con insuficiencia renal. La semi-vida plasmática es de 0.5 a 1 hora aunque aumenta significativamente en los neonatos y en los pacientes con insuficiencias renal o hepática en los que se deben reducir las dosis

CONTRAINDICACIONES: FUROSEMIDA está contraindicada en pacientes con anuria y en pacientes con antecedentes de hipersensibilidad al fármaco. La administración de FUROSEMIDA se debe interrumpir durante el tratamiento de la enfermedad renal progresiva severa si ocurre azoemia creciente y oliguria. BIBLIOGRAFÍA 1. Reynolds J, Martindale W. The Extra Pharmacopoeia. 31 ed. London: Royal Pharmaceutical Society; 1996. pp. 871. 2. Kradjan W. Congestive Heart Failure. In: Young L, Koda-Kimble M, eds. Applied Therapeutics: The clinical use of drugs. 6 ed. Vancouver: Edit. Applied Therapeutics Inc.; 1999. pp. 15-4.

87.- Al encontrarse realizando exploración física por dolor abdominal, usted identifica signo de Murphy positivo, esto se traduce en: a) Dolor a la palpación profunda en el punto cístico b) Dolor a la palpación en el punto cístico durante la inspiración c) Dolor a la palpación profunda en el punto cístico durante la espiración d) Dolor que interrumpe la inspiración a la palpación profunda en el punto cístico

Maniobra de Murphy, el enfermo sentado y el examinador colocado atrás engancha el punto cistico mientras el enfermo trata de realizar una inspiración profunda, Es positiva en la cocolecistitis aguda. Apuntes de Semiología Semiología médica y técnica exploratoria Escrito por Antonio Surós Batlló, Juan Surós Batlló 88.- Masculino de 39 años, obeso, no fumador, con asma desde hace año y medio, que empeora después de las comidas y se despierta por las noches con tos irritativa. Sigue tratamiento con broncodilatadores y esteroides. ¿Cuál sería la mejor prueba diagnóstica para aclarar el cuadro? a) b) c) d)

Endoscopia bronquial. Pruebas de provocación con alergenos. pH esofágico durante 24 horas. Radiografía esofagogastroduodenal

INDICACIONES DE LA pH-METRÍA La pHmetría intraesofágica de 24 horas es una prueba con una alta sensibilidad y especificidad para el diagnóstico de RGE En general, la pHmetría estará indicada en tres tipos de situaciones: a) Cuando existen síntomas sugestivos de RGE y la evolución no es favorable a pesar de instaurar el tratamiento correcto. b) Cuando quiera establecerse la relación entre RGE y síntomas extradigestivos. c) Como control de la eficacia del tratamiento, ya sea médico o quirúrgico. Desde un punto de vista práctico pueden diferenciarse las situaciones en las que puede o no ser de utilidad la realización de una pHmetría. 1. pHmetría innecesaria En general, no será necesario realizar una pHmetría para diagnosticar el RGE cuando el diagnóstico ya esté hecho mediante otra prueba o cuando sea evidente por la clínica.

1 Esofagitis péptica. La pHmetría no será necesaria para hacer el diagnóstico de RGE si la histología o la endoscopia han demostrado la existencia de una esofagitis péptica. 2. pHmetría poco útil. Existen una serie de situaciones en las que los datos ofrecidos por la pHmetría contribuyen poco para decidir el tratamiento o establecer el pronóstico del paciente. 1.- Estudio del paciente con disfagia. El tránsito digestivo, la endoscopia con toma de biopsias y la manometría esofágica suelen proporcionar información mucho más útil en este tipo de patología. 2.- Estudio de trastornos motores. Estaría indicada la manometría. Si el trazado muestra alteraciones sugestivas de esofagitis, ésta deberá confirmarse mediante endoscopia. 3.- Estudio del reflujo alcalino. La pHmetría convencional de un solo canal no proporciona información suficiente para el estudio del reflujo alcalino. La colocación de un electrodo gástrico amplia esta información, aunque hay que tener en cuenta que puede existir reflujo duodenogastroesofágico sin alcalinización gástrica ni esofágica.

3. pHmetría útil 3.1 Pacientes con pausas de apnea. Los episodios de apnea de origen periférico, producidos durante el sueño y de larga evolución son los que con más probabilidad pueden ser debidos a RGE y en los que más útil puede ser la pHmetría. No es suficiente con demostrar la existencia de reflujo, sino que debe establecerse la relación entre éste y la apnea mediante la realización de un registro pneumocardiográfico múltiple simultáneo. No será necesario realizar pHmetría en aquellos casos en los que la relación entre las pausas de apnea y los episodios de reflujo sea clínicamente evidente. 3.2 Episodios aparentemente amenazadores para la vida. Al igual que en el grupo anterior, la pHmetría deberá formar parte de un registro múltiple de frecuencia cardiaca, respiratoria y pulsioximetría para establecer la relación del reflujo con estos episodios. 3.3 Asma refractario al tratamiento. Más de la mitad de los asmáticos presentan RGE concomitante. Sin embargo, el reflujo puede ser tanto causa como consecuencia de la patología respiratoria, por lo que lo ideal es que la pHmetría pueda demostrar una relación temporal entre los episodios de reflujo y la aparición de las sibilancias. En los casos en que esto no sea posible, puede ser útil detectar un patrón de RGE consistente en episodios prolongados durante el periodo nocturno . No está claramente demostrada la necesidad de reflujo a nivel proximal para la producción de los síntomas respiratorios, por lo que puede ser suficiente con la realización de una pHmetría convencional con un solo electrodo esofágico.

3.4 Otros síntomas respiratorios. Puede utilizarse para el diagnóstico del RGE oculto en pacientes con tos crónica, neumonía recurrente o aspiraciones pulmonares, buscando la presencia de reflujos largos durante el sueño. Aunque lo ideal es demostrar la relación temporal del reflujo con la patología respiratoria, esto puede ser difícil debido al carácter intermitente de la misma . 3.5 Patología ORL. El RGE puede ser la causa de diversa sintomatología ORL, como estridor, laringitis e, incluso, sinusitis refractarias al tratamiento convencional. Es menos probable que otras patologías como la otitis recurrente, la disfonía o la papilomatosis laríngea sean debidas a reflujo. En estos casos sí estaría indicada la realización de pHmetría de doble canal, situándose el electrodo proximal inmediatamente por debajo del esfínter esofágico superior. 3.6 Control del tratamiento médico. Indicada para valorar la eficacia del tratamiento del RGE moderado-severo previamente diagnosticado mediante pHmetría. En los casos con mala evolución podrá realizarse tras 4-8 semanas de tratamiento. Si la evolución es favorable, la pHmetría de control puede diferirse 6-12 meses. En casos de RGE leve o de muy buena evolución no será imprescindible la realización de pHmetría de control previa al alta definitiva. 3.7 Control pre y postquirúrgico. Aunque la pHmetría no es el único criterio para indicar la cirugía, sí que puede mostrar datos que la aconsejen, como la persistencia de un reflujo importante tras el tratamiento, la existencia de reflujos nocturnos prolongados, etc. Por otra parte, la mejora del registro puede aconsejar el retraso de la cirugía. Aunque no existe acuerdo unánime sobre la indicación tras la cirugía, puede realizarse 3-6 meses después para comprobar la competencia del mecanismo antirreflujo. En aquellos pacientes con persistencia de los síntomas es necesario realizar pHmetría antes de valorar la reintervención. 89.- Masculino de 5 años es llevado al servicio de urgencias por dolor súbito hiperemia, y aumento de volumen en región interna de órbita derecha. Exploración Física: Se palpa masa bien delimitada dolorosa en el borde interno de la órbita. La medida terapéutica inicial en este paciente es: a) b) c) d)

Sólo drenaje. Tobramicina tópico y drenaje. Extirpación de saco lagrimal. Dicloxacilina y drenaje.

El tratamiento debe iniciarse antes de que se identifique el microorganismo causal. Tan pronto se obtienen cultivos de nariz, conjuntivas y sangre deben administrarse antibióticos. La terapéutica antibiótica inicial debe cubrir estafilococos, H influenzae y anaerobios. La mayor parte de los casos responde a los antibióticos. Aquellos en los cuales esto no es así, pueden requerir drenaje. Riordan P,Witcher j, Oftalmología general de Vaughan y Asbury, 17ª Ediciión, Pág. 262

90.- In a male patient in determining non-pathological short stature, the most common cause is: a) b) c) d)

Hypothyroidism Family Malnutrition Nephropathy

La talla constituye un indicador muy útil del estado de salud de un niño, por lo cual una desviación significativa del canal de crecimiento normal puede ser la primera manifestación de una alteración clínica de importancia. Junto con la talla de un niño, es importante evaluar su velocidad de crecimiento, parámetro más sensible ya que permite detectar cambios en el canal de crecimiento que habitualmente preceden a los cambios en la talla absoluta. La velocidad de crecimiento debe establecerse sobre períodos no menores a 6 meses para ser confiable, restando particular atención a las desviaciones significativas y no a aquellas menores, que pueden observarse en niños normales. Se define como talla baja aquella ubicada bajo el percentil 3 o bajo 22 desviaciones estándar de las curvas de NCHS. Estas curvas, de origen norteamericano, tienen utilidad para pacientes pertenecientes a estratos socioeconómicos medios y altos, pero tienen menos utilidad para evaluar a niños que pertenecen a estratos socioeconómicos bajos en nuestro país. En general ameritan estudio aquellos pacientes cuya talla se encuentra bajo el percentil 3, o cuya velocidad de crecimiento se ha deteriorado en forma significativa incluso antes de ubicarse 22 desviaciones estándar bajo la media. Las causas de talla baja se clasifican en: 1. Variantes normales: talla baja familiar y retraso constitucional. 2. Trastornos primarios del crecimiento como displasias esqueléticas, trastornos del desarrollo intrauterino y anormalidades cromosómicas.

3. Alteraciones del crecimiento secundarias a nutrición inadecuada, enfermedades crónicas (como síndrome de malabsorción, insuficiencia renal, alteraciones pulmonares o cardíacas),  y enfermedades endocrinológícas (como hipotiroidismo, alteraciones del eje somatotráfico, síndrome de Cushing, o raquitismo).

Causas de talla baja · Familiar ·Retardo constitucional ·Retardo del crecimiento intrauterino · Alteraciones nutricionales · Enfermedades sistémicas · Alteraciones cromosómicas · Displasias óseas · Deprivación emocional - maltrato infantil · Trastornos endocrinos

REFERENCIAS 1. Mahoney CP : Evaluating the child with short stature. Pediatr Clin North Am 1987; 34: 825-48. 2. Mericq V, y Cassorla F : Sistema hormona del crecimiento-efector y su rol en el crecimiento infantil. Rev Chil Pediatr 1997; 68: 27-37. 3. Rosenfeld R, Albertsson- Wikland K, Cassorla F , et al: Diagnostic controversy, the diagnosis of childhood growth hormone deficiency revisited. J Clin End 4. Codner E, Mericq V, Ugarte F , et al: Utilidad de la determinación del factor de crecimiento insulino símil tipo 1 y de su proteína ligante tipo 3 en el diagnóstico de la deficiencia de hormona de crecimiento en niños. Rev Méd Chile 1999; 127: 807-13.

91.- Femenino de 1 año y medio de edad, es llevada por sus padres a consulta, por que presenta lesiones rojas, con costras melicéricas en la cara, cuero cabelludo, área de pañal y extremidades, desde aproximadamente los 3 meses de edad. El evitar sustancias irritantes no ha funcionado en el tratamiento. La niña se rasca constantemente las áreas afectadas. Hay una historia familiar positiva para asma y fiebre. ¿Cuál es el diagnóstico más probable? a) b) c) d)

Celulitis Dermatitis de contacto Dermatitis atópica Liquen simple crónico

La dermatitis atopica (DA) (Wise-Sulzberger, 1993), llamada neurodermatitis diseminada, por las escuelas Europea, es un estado reaccional de la piel, intensamente pruriginosa, más frecuente en los niños, multifactorial, en la que combinan factores constitucionales y factores ambientales, por lo tanto de difícil tratamiento, muy frecuente en la consulta diaria del pediatra y del dermatólogo. DATOS EPIDEMIOLOGICOS La enfermedad originalmente conocida como prúrigo de Besnier y eccema constitucional, es ubicua, afecta a todas las razas y existe en todos los países. Se señala su existencia hasta en el 2% de la población general y en el 14% de los niños. La enfermedad se inicia antes del primer años de la vida en más del 60% de los casos, la curva desciende hacia los 12 años de la cifra de inicio es apenas del 5%-y es excepcional que la enfermedad se inicie en la edad adulta. La enfermedad sufre exacerbaciones en las temporadas de calor o de frío cuando hay sequedad de la atmósfera.

CUADRO CLÍNICO Tradicionalmente se han descrito tres etapas que en la actualidad ya no se presentan pues se suman unas a otras debido a los tratamientos que reciben los pacientes desde su inicio. Etapa de lactante. La enfermedad suele iniciarse en los primeros meses de vida, a veces casi desde el nacimiento. Las lesiones afectan la cara: mejillas, frente (respetando su centro); afectan también la piel cabelluda, los pliegues, las nalgas y el dorso de los pies. Las lesiones son de tipo eccematosos (piel llorosa): eritema, vesícular y costras melicéricas con costras hemáticas como signo del rascado. Estas lesiones altamente pruriginosas evolucionan por brotes y en general si el paciente no es yatrogénicamente tratado, tienen tendencia a involucionar al año de edad. Fase del escolar. Se inicia hacia los 3 a 7 años, cuando el niño empieza a ir a la escuela. Las lesiones son preferentemente flexurales: cuello, pliegues de codo y huecos poplíteos y la morfología corresponde a la de una dermatitis crónica: zonas de eritema y liquenificación (índice de rascado crónico) y costras hemáticas periódicamente sufren un proceso de eccematización sobre todo por los tratamientos indebidos. La enfermedad evoluciona por brotes, hay temporadas en que no existen lesiones aun cuando persiste a veces una piel xerótica (seca) y pigmentada con intenso prurito. Etapa del adulto. Se veía con poca frecuencia y ahora es habitual en las consultas diarias debido a los malos tratamientos. En este caso además de las zonas flexurales, se presenta lesiones periorbitarias; y peribucales y lesiones vesiculosas en las manos. Son lesiones tanto liquenificadas como eccematosas, muy pruriginosas, que alternan con periodos asintomáticos. Con el uso inmoderado de los corticoesteroides no hay diferenciación entre las etapas de la enfermedad y se ven casos que arrastran su enfermedad casi desde que nacen hasta la edad adulta.

DIAGNÓSTICO Es clínico y relativamente sencillo, aunque no toda dermatitis flexural es necesariamente de origen atópico. Deben tomarse en cuenta los antecedentes del propio enfermo y los familiares. En los lactantes debe diferenciarse de la dermatitis seborreica que afecta sobre todo la piel cabelluda y el centro de la cara. En ocasiones hay mezcla de las dos dermatitis: la atópica y la seborreica y es difícil diferenciarlas. La dermatitis del pañal predomina en los lactante; en las zonas glúteas y genitales, se presenta con eritema y lesiones vesículopapulosas muy pruriginosas. La pitiriasis alba corresponde a los clásicos “jiotes” de los niños: manchas hipocrómicas cubiertas de fina escama en la cara. Estos procesos pueden coincidir con manifestaciones de DA, pero nunca se ha demostrado la relación de causa efecto.

TRATAMIENTO Medidas generales. Es conveniente una explicación amplia al paciente y su familia señalando la naturaleza y evolución de la enfermedad y lo que se espera del tratamiento que vamos a indicar. Evitar el sol excesivo, el agua clorada de las albercas, el uso de jabón (indicar jabones neutros o sustitutos de jabón), prohibir el uso de pomadas y remedios caseros así como los cosméticos que irritan a la piel. Las dietas restrictivas han sido y siguen siendo un tema muy controversial, pues mientras los alergistras siguen insistiendo sin muchas bases en el beneficio de estas dietas, los más hemos comprobado su inutilidad. Existe un grupo muy limitado en que se puede demostrar que una dieta restrictiva de huevo, leche, fresas, etc. puede mejorar los brotes de DA y en tales casos (sólo en esos casos) se aconsejaría la supresión de tales medicamentos; en lo general se permite al paciente que coma de todo. En la actualidad se están limitando por los pediatrias ciertos alimentos como el huevo, las fresas, el plátano y la leche de vaca en el primer año de vida por ser muy alergénicos y se discute la utilidad de la leche materna en estos niños. Es necesario hacer ver que estos pacientes requieren de una atención más personal, que sienten la necesidad de cariño por lo que es de recomendarse que el niño sea atendido personalemente por la madre. Tratamiento tópico. Depende del estado de la piel; si está eccematosa debe secarse antes de aplicar cualquier pomada que será rechazada por la piel llorosa. El uso de fomentos con agua de manzanilla o suero fisiológico es útil. Los fometos con agua de végeto (subacetato de plomo) al 20% son potentes antiexudativos, pero deben limitarse a áreas restringidas y no usarse en niños. Estando la piel ya seca, se usan pasta inertes que llevan óxido de zinc y calamina en una base de vaselina y lanolina que son protectoras a la vez que antiprutiginosas. Si la piel está muy seca y liquenificada, los fomentos y baños serán emolientes, con almidón y aceites seguidos de cremas o pomadas más grasosas que llevan vaselina, coldcream y óxido de zinc. En el comercio existen numerosos preparados humectantes que ayudan a mantener el manto ácido grasa-agua, que se pierde en la DA. Si hay dermatitis por contacto o impétigo hay que tratar primero estas complicaciones con sulfato de cobre al 1: 1000 y pomadas con vioformo o mupirocín. En el caso de eritrodermia, los baños emolientes y el uso del petrolato (vaselina) será lo indicado. Cuando hay mucha liquenificación, pueden usarse cremas con alquitrán de hulla al 3% en base de coldcream por tiempo y zonas limitadas. Los corticoesteroides tópicos son los medicamentos más usados en esta enfermedad y muchas veces causa de las complicaciones que se presentan. Estos medicamentos no curan nada, solo engañan al paciente y al médico haciéndoles creer que la enfermedad va curando cuando sólo se oculta y modifica. Al pasar el efecto de estos medicamentos, invariablemente se presenta el rebote y más tarde la corticodependencia, haciendo a la enfermedad incontrolable. Nunca deben usarse los corticoesteroides fluorinados en niños y en la cara y zonas genitales o en los pliegues por su posibilidad de absorción.

La hidrocortisona es de baja potencia, hace menos daño, pero también es menos efectiva; algunos la recomiendan. Tratamiento sistémico. El uso de antihistamínicos sobre todo de la primera generación que son sedantes como la clorfeniramina y la hidroxicina ayudan a mejorar el prurito; el ketotifeno y la oxotamida por su acción dual: inhiben la producción de histamina por los mastocitos y bloquean los receptores Hl, también son de ayuda, al igual que los sedantes suaves tanto para el paciente como para la madre que está en perenne angustia que transmite al pequeño paciente. La talidomida ha mostrados ser de ayuda en casos de DA conticoestropeada, en su fase eritrodérmica, no tanto en los niños, a dosis de 100 mg al día. Los antibióticos tipo dicloxacilina, serán necesarios cuando haya infección o simplemente eccematización por el papel que tiene el estafilococo dorado. 1. Criterios Diagnóstico para Dermatitis Atópica (DA) , MedicalCriteria.com. Dr. Luciano Domínguez-Soto, Dr. Amado Saúl Cano, Dermatitis atópica o neurodermatitis diseminada, Dermatología, Parte C, Libro ) "

92.- El siguiente grupo de microorganismos patógenos son capaces de multiplicarse extracelularmente, pero que resisten la acción fagocitaria del macrófago, lo que ocaciona puedan persistir latentes, su erradicación depende de la activación de los macrófagos por los linfocitos T CD4 (T helper). ¿A que grupo nos referimos?

a) Entamoeba histolytica. b) Mycobacterium tuberculosis. c) Pseudomonas aeruginosa. d) Virus de la hepatitis C. La supervivencia del bacilo de Koch (BK) dentro del hospedador depende de la capacidad de los microorganismos para multiplicarse dentro de los macrófagos y monocitos. La inmunidad del hospedador a M. tuberculosis  depende de que ocurra una respuesta celular antimicobacteriana que active los macrófagos para que produzcan la muerte o restrinjan el crecimiento de los microorganismos intracelulares. La tuberculosis es la enfermedad micobacteriana humana clásica. La vía de infección por Mycobacterium tuberculosis  consiste en la inhalación de aire con partículas infectadas que alcanzan la vía aérea terminal. Una vez deglutidos por los macrófagos alveolares, los bacilos comienzan a multiplicarse libremente y acaban destruyendo las células fagocitarias. De esta manera, se produce un ciclo posterior de fagocitosis por los macrófagos y linfocitos, que emigran hacia el foco de infección; finalmente, se produce la destrucción celular. Los macrófagos infectados diseminan el proceso hacia los ganglios linfáticos locales en la fase inicial de la enfermedad, así como hacia el torrente circulatorio y otros tejidos (por ejemplo: médula ósea, bazo, riñones, huesos y sistema sis tema nervioso central

93.- Femenino de 45 años que tras 20 min. Posteriores al inicio de una transfusión transfusión presenta fiebre, ansiedad, dolor lumbar severo, nausea y enrojecimiento facial, por lo que usted sospecha: a) Reacción febril b) Reacción hemolítica aguda c) Toxicidad por citrato d) Hepatitis R e a cc i ó n Hemolítica Definición: Destrucción acelerada del eritrocito. De acuerdo a la causa puede ser inmune o no inmune, por el sitio de destrucción puede ser intra o extravascular y por el tiempo de aparición puede ser aguda o retardada. Incidencia Reacción hemolítica aguda: Las referencias internacionales reportan una incidencia de reacción hemolítica aguda de 1 en 6 000 en 30 000 unidades transfundidas, con una tasa de mortalidad de 1 en 500 000 a 1 en 1 000 000 de unidades. Del total de las reacciones hemolíticas agudas, el 6% resultan fatales. La FDA reporta que alrededor del 41% de las muertes por transfusión son causadas por incompatibilidad ABO, con una incidencia de mortalidad de 1 en 200 000 pacientes transfundidos. Reacción hemolítica retardada. La incidencia es de 1 en 2 500 a 1 en 4 000 unidades transfundidas. La mortalidad es de 1 en 3.85 millones de unidades y de 1 en 1.15 millones de pacientes transfundidos.

Fisiopatogenia La reacción hemolítica transfusional más grave se presenta cuando interactúan los eritrocitos transfundidos con los anticuerpos preformados en el receptor. La reacción antígenoanticuerpo puede no activar complemento de acuerdo a la inmunoglobulina implicada, lo que conduce a hemólisis intra o extravascular. En la hemólisis extravascular algunas citocinas con actividad inflamatoria y vasoactiva intervienen en la reacción como: Factor de necrosis tumoral alfa, Interleucina 1, 6, 8 y Proteína quimioatractante de macrófago (MCP), así como la liberación de sustancias tromboplásticas que explican el cuadro clínico característico de la reacción hemolítica transfusional. En la hemólisis extravascular el eritrocito sensibilizado es destruido por el sistema fagocítico mononuclear.

REACCION HEMOLITICA AGUDA Ó INMEDIATA DE TIPO INMUNE: Es la reacción ya descrita por incompatibilidad ABO y como consecuencia de ella al iniciar la transfusión en pocos minutos pueden ser lisados la mitad ó más de los de los hematíes transfundidos. La reacción hemolítica aguda se produce de forma brusca, tras la infusión de varios mililitros de sangre aparece un cuadro de fiebre, escalofrios, dolor lumbar, dolor opresivo  en torax , cefalea  nauseas con ó sin vomitos e incluso shock ,dependiendo de la cantidad y rapidez de la sangre transfundida. Los sintomas dependen de los pacientes pero la fiebre aparece en todos los casos. La reacción es inmediata al inicio de la transfusión En los enfermos anestesiados estas reacciones quedan enmascaradas y ser mínimas,por lo que la hemólisis puede debutar con taquicardia e hipotensión brusca trás el inicio de la transfusión,si persiste la transfusión puede aparecer oliguria,hemoglobinuria,CID ó fracaso renal postoperatorio debido a algún efecto efec to tóxico de la Hb ó a la situación de shock.

Diagnóstico Diferencial Contaminación Bacteriana del componente sanguíneo Hemólisis no inmune: - Mecánica - Térmica - Osmótica

Lecturas recomedadas: Rev Mex Med Tran, Vol. 3, Núm. 1, pp 18- 21 • Enero - Abril, 2010 Hospital de Pediatría CMN Siglo XXI Boletín La Calidad Abr May Jun 2004

94.- Femenino de 58 años, que acude al médico por cefalea y mareos. Exploración física: rubicundez facial. Exámenes de Laboratorio: Hto 62%, leucocitos 13.000/microl con fórmula normal; plaquetas 325.000/microl; saturación arterial de oxígeno 95%; fosfatasa alcalina granulocítica 150 (sobre una puntuación máxima de 200; valor de referencia: 30 a 80); B12 sérico 593 pg/ml (valores de referencia: 170 a 920 pg/ml); Masa eritrocitaria: 35 ml/kg. La patología responsable de los hallazgos de laboratorio en esta paciente es: a) b) c) d)

Eritrocitosis espuria. Vértigo de Ménière. Poliglobulia secundaria a hemangioma cerebeloso. Policitemia vera.

Trastorno mieloproliferativo de etiología desconocida, que se caracteriza por proliferación anormal de todos los elementos hematopoyéticos de la médula ósea  y por incremento absoluto de la masa de células rojas y del volumen total de sangre, y que se asocia frecuentemente con esplenomegalia, leucocitosis, y trombocitopenia. La hematopoyesis también es reactiva en sitios extramedulares (hígado y bazo). En ocasiones ocurre mielofibrosis. El curso clínico de la policitemia vera se complica con frecuencia con trombosis y posible transición a metaplasia mieloide con mielofibrosis o leucemia mieloide aguda. Dejada a su curso natural la supervivencia media es de 18 meses. Las flebotomías y el tratamiento citoreductor han reducido las complicaciones tromboticas y mejorado sustancialemente la superviviencia. El dilema terpaeútico que se plantea en esta entidad es la utilización aislada de la flebotomia para prevenir el riesgo de trombsosi o la combinacion con el tratamiento citorreductor, por ejemplo busulfan, con el riesgo, entonces de ocurrencia de mielofiboris o transformación maligna.( Polycythemia Vera Study Group ) .

Bibliografía: 1.

Berk PD, Goldberg JD, Silverstein MN, et al.: Increased incidence of acute leukemia in polycythemia vera associated with chlorambucil therapy. New England Journal of Medicine 304(8): 441-447. 2. Silverstein MN: Myeloproliferative disease. Current Hematology and Oncology 6: 163-184, 1988. 3. Demory JL, Dupriez B, Fenaux P, et al.: Cytogenetic studies and their prognostic significance in agnogenic myeloid metaplasia: a report on 47 cases. Blood 72(3): 855-859, 1988. 4. Barosi G: Myelofibrosis with myeloid metaplasia: diagnostic definition and prognostic classification for clinical studies and treatment guidelines. Journal of Clinical Oncology 17(9): 2954-2970, 1999. Tefferi A: Myelofibrosis with myeloid metaplasia. New England Journal of Medicine 342(17): 1255-1265, 2000.

95.- Masculino de 22 años de edad, que acude al servicio de cardiología, sin antecedentes patológicos y sin hábitos tóxicos inicia con cuadro de 8 días de evolución acompañado de fiebre y dolor centrotorácico intenso que aumenta con la inspiración y los movimientos respiratorios. En el ecocardiograma se objetiva un derrame pericárdico importante, sin signos de compromiso hemodinámico. ¿Cuál sería su primer diagnóstico? a) b) c) d)

Pericarditis aguda idiopática. Pericarditis tuberculosa. Taponamiento cardíaco. Pericarditis de origen autoinmune.

La pericarditis aguda (PA) es un síndrome clínico debido a una inflamación del pericardio que se caracteriza por dolor torácico, roce pericárdico y alteraciones electrocardiográficas evolutivas. Su incidencia y prevalencia son difíciles de determinar. Estudios realizados en autopsias muestran una prevalencia de alrededor de 1% en la población general, lo que sugiere que la presentación en ocasiones se hace de manera subclínica. Representa un 5% de los dolores torácicos no isquémicos que acuden a urgencias 1.

La causa más frecuente de PA es la idiopática y/o viral, que corresponden a casi el 80% de los casos, de hecho, los términos idiopático y viral suelen utilizarse de forma indistinta 2-5. Otras posibles causas son: Infecciosa (7%): Bacteriana: neumococo, estreptococo, estafilococo, neisseria, legionella. Tuberculosa. Vírica: coxsackie, influenzae, VIH, hepatitis, adenovirus, echovirus. Fúngica: histoplasmosis, coccidiomicosis, blastomicosis, candidiasis. Otras: sífilis, protozoos, parásitos (entamoeba histolytica, toxoplasma). Neoplasias (7%): primarias: mesotelioma, sarcoma, fibroma, lipoma secundarias (metástasis): pulmón, mama, linfoma, leucemia, carcinoides… La triada diagnóstica clásica es: dolor torácico, roce pericárdico y alteraciones electrocardiográficas que suele ser precedida de fiebre, mal estado general y mialgias (aunque en pacientes ancianos no suele presentarse la fiebre). Los síntomas principales son: Dolor torácico: dolor retroesternal o localizado en hemitórax izquierdo en la zona precordial, de características pleuríticas (aunque en ocasiones puede simular un cuadro isquémico), que se puede irradiar al cuello o al trapecio izquierdo. Puede acompañarse de sensación de falta de aire. Ocasionalmente se localiza en región epigástrica, simulando un abdomen agudo. Suele aumentar con la inspiración profunda, la tos, la deglución y la posición supina y mejorar con la incorporación a la posición de sentado. Roce pericárdico: es el hallazgo patognomónico de la exploración física en la PA. Corresponde al movimiento del corazón dentro del pericardio y se asemeja al ruido que produce la fricción de cuero, por roce de las hojas pericárdicas inflamadas. Se da en aproximadamente el 60-85% de los casos 7,8. Puede ser transitorio, monofásico, bifásico o trifásico, según la relación que tenga con los movimientos cardiacos durante la sístole auricular, sístole ventricular y el llenado ventricular rápido. Se ausculta mejor a nivel de mesocardio y en parte baja de borde esternal izquierdo, sobretodo al final de la espiración con el paciente inclinado hacia delante. Es independiente de la existencia de derrame. Cuando sólo tiene un componente se puede confundir con un soplo sistólico mitral o tricuspídeo. Es característico del roce pericárdico su evanescencia (por lo que las auscultaciones deben ser repetidas en varias ocasiones) y los cambios en sus características según la posición en que se realice la exploración.

Bibliografía: •









LeWinter MM, Kabbani S. Pericardial diseases. En: Braunwald´s heart disease. Douglas P. Zipes editor. 7th. Ed. Philadelphia: Elsevier; 2005. p.1757-1780 Friman G, Fohlman J. The epidemiology of viral heart disease. Scand J Infect Dis Suppl 1993; 88: 7–10. [Medline] Braunwald E. Enfermedades del pericardio. En: Harrison Principios de Medicina Interna. Dennos L. Koper editores. 16ª ed. México: Mc-Graw-Hill; 2005. p. 15541571 Maisch B, Seferovic PM, Ristic AD, Erbel R, Rienmüller R, Adler Y, et al, Grupo de Trabajo para el Diagnóstico y Tratamiento de las Enfermedades del Pericardio de la Sociedad Europea de Cardiología. Guía de Práctica Clínica para el diagnóstico y tratamiento de las enfermedades del pericardio. Versión resumida. Rev Esp Cardiol. 2004; 57:1090-114. [Medline] [Texto completo] Zayas R, Anguita M, Torres F, Gimenez D, Bergillos F, Ruiz M, Ciudad M, Gallardo A, Valles F. Incidence of specific etiology and role of methods for specific etiologic diagnosis of primary acute pericarditis. Am J Cardiol. 1995 Feb 15; 75(5):378-82.

96.- En nuestro país, la Secretaría de Salud ha lanzado una campaña para fomentar entre el personal de las instituciones gubernamentales, una rutina de ejercicios con duración de cinco minutos diarios, esta es una acción específica de: a) b) c) d)

Restauración de la salud Medicina preventiva Protección de la salud Promoción de la salud.

Las acciones de Promoción de la Salud pretenden fomentar la salud de los individuos y la colectividad, promoviendo que adopten estilos de vida saludables. Ello se consigue mediante intervenciones de educación sanitaria. Piedrola G. G; Medicina Preventiva y Salud Pública. Masson, 9ª ed. Pág. 15, 16.

97.- Al existir la sospecha de líquido libre en la cavidad peritoneal y después de analizar los hallazgos de manera conjunta de las placas de abdomen y en el ultrasonido abdominopélvico, ¿Dónde se deberán de buscar los cambios de densidad de manera inicial?

a) b) c) d)

Infradiafragmático Fondos de saco de Douglas y de Morrison Transcavidad de los epiplones Correderas parieto-cólicas

Pedrosa C, Casanova R. Diagnóstico por imagen . Mc Graw Hill 2001 p. 209, 210. La diseminación del líquido intraperitoneal está producida por la gravedad y la presión hidrostática secundaria al movimiento diafragmático.la pelvis es la parte más dependiente de la cavidad peritoneal, su capacidad es de 300 cc, a partir de lo cual el líquido asciende por las correderas parietocólicas. El líquido pélvico desplaza las asas intestinales que rellenan los espacios pélvicos, el íleon del lado derecho y el colon sigmoides en el izquierdo, lo que resulta de la parición de una densidad homogénea en al pelvis menor, en contra del patrón poco homogéneo de las asas intestinales que tienen gas y líquido. En la pelvis femenina el líquido se acumula en los recesos laterales de la vejiga y al útero, y en el hombre puede verse en la fosa vesicorrectal y los recesos pélvicos laterales. En el ultrasonido el diagnóstico de pequeñas cantidades de líquido debe hacerse fundamentalmente en el saco de Douglas, en la bolsa de Morrison y en el receso  yuxtaesplénico. 98.- Masculino de 12 años, acude al centro de urgencias 30 mins. Después de haber ingerido comprimidos de sulfato ferroso de 325 mg. La madre indica que faltan 20 comprimidos del frasco. Cada comprimido tiene 65 mg de hierro elemental. El niño ha vomitado una vez y su comportamiento parece totalmente normal. Los hallazgos de la exploración física no arrojan nada importante. Pesa 13 kg. ¿Cuál de los siguientes enunciados sobre el estado del niño es el correcto? a) b) c) d)

La dosis de hierro no debería causar secuelas clínicas. Ese episodio único de vómito probablemente no esta relacionado con la ingestión. Aunque ha vomitado una vez debería administrarse ipecacuana. Los catárticos son ineficaces en intoxicaciones.

En situaciones en las que no hay testigos de la ingestión debe presuponerse inicialmente el “peor escenario posible” para estimar la toxicidad potencial. La dosis máxima de hierro ingerida por este niño es de 65 mg de hierro elemental x 20 píldoras / 13 kg = 100 mg /kg que es una dosis potencialmente grave. Entre los síntomas causados por la ingestión grave de hierro están los vómitos por tanto en este caso el vómito debe considerarse relacionado con la ingestión. Ello implica un riesgo potencial de secuelas graves. En conjunto la ingestión debe valorarse como clínicamente importante. Como un solo episodio de vómitos no vacía el estómago de manera suficiente debe administrarse ipecacuana o alternativamente proceder a un lavado gástrico. El carbón activado no adsorbe el hierro y no es necesario. Los catárticos sirven de ayuda una vez que e ha vaciado el estómago. Los niveles de hierro sérico y de capacidad de enlace del hierro deben estimarse según las circunstancias. También es útil obtener una radiografía de abdomen por cuanto las píldoras son radioopacas y puede deducirse así algún indicio de su permanencia en el tracto GI después del tratamiento inicial. 1.- Montoya-Cabrera MA. Intoxicaciones y envenenamientos en niños. México, Intersistemas , 2000. 2.-Montoya CMA. Toxicología clínica. 2ª. Ed, México, Méndez Editores, 1997

99.- Femenino de 49 años, con antecedentes de obesidad, IAM hace 5 años así como tabaquismo positivo. Ingresa al servicio de traumatología por fractura de cadera, será intervenida quirúrgicamente para reemplazo total de la misma, el riego que presenta esta paciente es: a) b) c) d)

Paciente de riesgo bajo para trombosis venosa profunda Paciente de riesgo alto para trombosis venosa profunda Paciente con riesgo de trombosis arterial Paciente con riesgo muy bajo de TEP

La fisiopatología de la TVP se resume mediante la tríada de Virchow: estasis sanguíneo, daño endotelial e hipercoagulabilidad. Estas 3 circunstancias aisladamente o en asociación, intervienen en el desarrollo de un trombo. Los factores de riesgo enumerados a continuación aumentan la probabilidad de desarrollar trombosis mediante uno o más de los mecanismos de la tríada (Kahn S, 1998): 1. Cirugía mayor, especialmente la cirugía ortopédica, pero también los pacientes sometidos a cirugía abdominal, neurocirugía. 2. Neoplasias malignas, el riesgo aumenta en los pacientes que reciben quimioterapia activa. 3. Infarto Agudo de Miocardio. 4. Síndrome Nefrótico.

5. Ictus isquémico, preferentemente en el miembro hemipléjico. 6. Inmovilización prolongada. A mayor tiempo de inmovilización mayor riesgo, aunque se ha documentado una incidencia de TVP del 13% en pacientes encamados durante 8 días. 7. TVP o Embolismos Pulmonares (EP) previos. El riesgo se debe probablemente a la persistencia de obstrucción al flujo y/o daño en las válvulas venosas tras la TVP anterior. 8. Embarazo y postparto. 9. Anticoncepción oral y Terapia Hormonal Sustitutiva (THS). 10.Alteraciones congénitas o adquiridas que produzcan hipercoagulabilidad: o Deficiencia congénita de Proteína C, Proteína S y Antitrombina III. o Resistencia a la Proteína C activada: esta alteración se encuentra en el 5% de la población general y en el 20-40% de los enfermos con TVP. o Hiperhomocisteinemia o Disfibrinogenemia o Presencia de Anticuerpos Antifosfolípido Estos trastornos se asocian a TVP recurrentes, o en localizaciones atípicas o de aparición a edades tempranas (antes de los 45 años). Estos trastornos se asocian a TVP recurrentes, o en localizaciones atípicas o de aparición a edades tempranas (antes de los 45 años). A pesar de la naturaleza inespecífica de los signos y síntomas de la TVP aguda Wells y cols desarrollaron un modelo clínico objetivo que incorpora 8 características clínicas o factores de riesgo asignándoles 1 punto a cada una si están presentes y – 2 si existe la probabilidad de un diagnóstico alternativo, esto permite clasificar a los pacientes en baja probabilidad ≤0 puntos, probabilidad moderada 2 puntos y probabilidad alta ≥ 3 puntos con prevalencias

de TVP de 5%, 33% y 85% respectivamente. Este modelo de probabilidad previa es validado en múltiples ensayos además de que tiene una excelente reproducibilidad (Cuadro 1) (8,1,12).

Bibliografía: •







Ebell MH. Evaluation of the patient with suspected deep vein thrombosis. J Fam Pract. 2001 Feb;50(2):167-71 [PubMed] [Texto completo] Gabriel Botella F, Labiós Gómez M, Brasó Aznar JV. Trombosis venosa profunda: presente y futuro. Med Clin 2000; 114: 584-596. [PubMed] Gorman WP, Davis KR, Donnelly R. ABC of arterial and venous disease. Swollen limb1: General assessment and deep vein thrombosis. BMJ 2000; 320: 1453-1456. [PubMed] [Texto completo] Green L, Fay W, Harrison V, Kleaveland M, Wahl R, Wakefield T, Weg J, Williams D. Venous thromboembolism (VTE) [Internet]. Ann Arbor (MI): University of Michigan Health System 2004 [acceso 18/3/2007] Disponible en: http://cme.med.umich.edu/iCME/vte04/ [NGC]

100.- A 6-year-old child presents with flesh-colored papules on the hand that are not pruritic. Examination reveals lesions that are approximately 4 mm in diameter with central umbilication. A halo is seen around those lesions undergoing regression. Which of the following is the most likely diagnosis? a) b) c) d)

Verruca vulgaris Molluscum contagiosum Keratoacanthoma Herpetic whitlow

El Molusco Contagioso es el nombre de una infección viral del grupo de los Poxvirus. Es frecuente, transmisible, autoinoculable (uno mismo la puede trasmitir a diferentes partes del cuerpo), se puede curar sola y es benigna. Afecta principalmente a los niños menores de 10 años, adultos sexualmente activos y a pacientes inmunosuprimidos (con defensas bajas) como los pacientes con SIDA, ocurriendo en un 5 ó 18 por ciento. Afecta más a hombres que a mujeres, y su frecuencia aumenta también en climas tropicales y durante el verano. CLINICA El periodo de incubación de la infección es de 14 a 50 días, aunque hay datos de recién nacidos con lesiones al cabo de 7 días postparto. Las lesiones se inician como neoformaciones que miden generalmente de 2 a 6 mm, aunque pueden llegar a medir 3 cm(13,14), son hemisféricas, cupuliformes, lisas, del color de la piel o perladas, algunas (20%) tienen una umbilicación central(15); la base es levemente eritematosa y son de consistencia firme(13,14). Se localizan en cualquier parte del cuerpo e incluso pueden afectar mucosas, generalmente se agrupan en un área específica, pero pueden estar diseminadas en personas infectadas con el virus de la inmunodeficiencia humana adquirida, siendo un marcador de enfermedad avanzada (16). En el caso de los niños las lesiones se localizan normalmente en cara, tronco, brazos y piernas a diferencia de los adultos jóvenes en quienes el molusco contagioso se adquiere por transmisión sexual, las lesiones tienen predilección por genitales, abdomen y cara interna de los muslos; en otras series no hay diferencia (17). Las lesiones del molusco suelen aparecer entre los 14 días a 6 meses después de la exposición, hay datos de recién nacidos con lesiones al cabo de 7 días postparto. Se pueden propagar por auto inoculación, pero es típico que se resuelvan espontáneamente en pocos meses. Las lesiones son asintomáticas en la mayoría de los pacientes (13,18), aunque en el 10% de los casos puede haber prurito y desarrollarse una reacción eccematosa (14).

View more...

Comments

Copyright ©2017 KUPDF Inc.
SUPPORT KUPDF